Liczby wymierne

96
Podróże po Imperium Liczb Część 1 Liczby wymierne Andrzej Nowicki Olsztyn, Toruń, 2008

description

Liczby wymierne

Transcript of Liczby wymierne

Page 1: Liczby wymierne

Podróże po Imperium Liczb

Część 1

Liczby wymierne

Andrzej Nowicki

Olsztyn, Toruń, 2008

Page 2: Liczby wymierne

WYM - 15(584) -27.08.2008

Page 3: Liczby wymierne

Spis treści

Wstęp 1

1 Wstępne informacje o liczbach wymiernych 51.1 Równości z liczbami wymiernymi . . . . . . . . . . . . . . . . . . . . . . . . . . . . . . 51.2 Całkowitość pewnych liczb wymiernych . . . . . . . . . . . . . . . . . . . . . . . . . . 61.3 Przedstawianie liczb wymiernych w szczególnej postaci . . . . . . . . . . . . . . . . . 91.4 Podzbiory zbioru liczb wymiernych . . . . . . . . . . . . . . . . . . . . . . . . . . . . 91.5 Różne zadania o liczbach wymiernych . . . . . . . . . . . . . . . . . . . . . . . . . . . 10

2 Rozkłady jedynki na ułamki proste 122.1 Ogólne fakty o rozkładach jedynki . . . . . . . . . . . . . . . . . . . . . . . . . . . . . 122.2 Jedynka i dwa ułamki proste . . . . . . . . . . . . . . . . . . . . . . . . . . . . . . . . 132.3 Jedynka i trzy ułamki proste . . . . . . . . . . . . . . . . . . . . . . . . . . . . . . . . 132.4 Jedynka i cztery ułamki proste . . . . . . . . . . . . . . . . . . . . . . . . . . . . . . . 142.5 Jedynka i pięć ułamków prostych . . . . . . . . . . . . . . . . . . . . . . . . . . . . . . 142.6 Jedynka i sześć ułamków prostych . . . . . . . . . . . . . . . . . . . . . . . . . . . . . 142.7 Jedynka i więcej niż sześć ułamków prostych . . . . . . . . . . . . . . . . . . . . . . . 152.8 Różne fakty o rozkładach jedynki . . . . . . . . . . . . . . . . . . . . . . . . . . . . . 16

3 Rozkłady liczb wymiernych na ułamki proste 173.1 Rozkłady liczb naturalnych . . . . . . . . . . . . . . . . . . . . . . . . . . . . . . . . . 173.2 Rozkłady liczb wymiernych . . . . . . . . . . . . . . . . . . . . . . . . . . . . . . . . . 173.3 Sumy dwóch ułamków prostych . . . . . . . . . . . . . . . . . . . . . . . . . . . . . . 183.4 Równanie 1x +

1y =

1z . . . . . . . . . . . . . . . . . . . . . . . . . . . . . . . . . . . . 21

3.5 Równanie 1x +1y =

2z . . . . . . . . . . . . . . . . . . . . . . . . . . . . . . . . . . . . 23

3.6 Równanie 1x +1y =

1z +

1t . . . . . . . . . . . . . . . . . . . . . . . . . . . . . . . . . . 24

3.7 Sumy trzech ułamków prostych . . . . . . . . . . . . . . . . . . . . . . . . . . . . . . . 243.8 Zadania różne . . . . . . . . . . . . . . . . . . . . . . . . . . . . . . . . . . . . . . . . 28

4 Odwrotności wyrazów pewnych ciągów 294.1 Niecałkowitość sumy odwrotności wyrazów ciągu . . . . . . . . . . . . . . . . . . . . . 294.2 Odwrotności wyrazów ciągu arytmetycznego . . . . . . . . . . . . . . . . . . . . . . . 294.3 Odwrotności kolejnych liczb naturalnych . . . . . . . . . . . . . . . . . . . . . . . . . 304.4 Naprzemienne sumy ułamków prostych . . . . . . . . . . . . . . . . . . . . . . . . . . 324.5 Odwrotności liczb pierwszych . . . . . . . . . . . . . . . . . . . . . . . . . . . . . . . . 334.6 Odwrotności liczb potęgowych . . . . . . . . . . . . . . . . . . . . . . . . . . . . . . . 334.7 Odwrotności liczb kwadratowych . . . . . . . . . . . . . . . . . . . . . . . . . . . . . . 344.8 Odwrotności liczb trójkątnych . . . . . . . . . . . . . . . . . . . . . . . . . . . . . . . 354.9 Odwrotności sześcianów . . . . . . . . . . . . . . . . . . . . . . . . . . . . . . . . . . . 364.10 Granice . . . . . . . . . . . . . . . . . . . . . . . . . . . . . . . . . . . . . . . . . . . . 37

5 Rozwinięcia dziesiętne liczb wymiernych 395.1 Tablice rozwinięć dziesiętnych pewnych liczb wymiernych . . . . . . . . . . . . . . . . 395.2 Okresy rozwinięć dziesiętnych liczb wymiernych . . . . . . . . . . . . . . . . . . . . . 415.3 Różne zadania o rozwinięciach dziesiętnych liczb wymiernych . . . . . . . . . . . . . . 42

i

Page 4: Liczby wymierne

6 Przystawanie modulo m dla liczb wymiernych 436.1 Definicje . . . . . . . . . . . . . . . . . . . . . . . . . . . . . . . . . . . . . . . . . . . 436.2 Przystawanie i mianowniki . . . . . . . . . . . . . . . . . . . . . . . . . . . . . . . . . 446.3 Przystawanie i dodawanie . . . . . . . . . . . . . . . . . . . . . . . . . . . . . . . . . . 456.4 Przystawanie jako relacja równoważności . . . . . . . . . . . . . . . . . . . . . . . . . 456.5 Przystawanie i mnożenie . . . . . . . . . . . . . . . . . . . . . . . . . . . . . . . . . . 466.6 Przystawanie modulo 2 . . . . . . . . . . . . . . . . . . . . . . . . . . . . . . . . . . . 466.7 Przystawanie modulo pk . . . . . . . . . . . . . . . . . . . . . . . . . . . . . . . . . . 47

7 Podzielność dla liczb wymiernych 497.1 Rozkład kanoniczny liczb wymiernych . . . . . . . . . . . . . . . . . . . . . . . . . . . 497.2 Relacja podzielności w Q∗ . . . . . . . . . . . . . . . . . . . . . . . . . . . . . . . . . 497.3 Nwd i nww dla liczb wymiernych. Definicje i przykłady . . . . . . . . . . . . . . . . . 507.4 Nwd i nww dla liczb wymiernych. Własności . . . . . . . . . . . . . . . . . . . . . . . 517.5 Względnie pierwsze liczby wymierne . . . . . . . . . . . . . . . . . . . . . . . . . . . . 52

8 Twierdzenie Wolstenholme i jego uogólnienia 538.1 Współczynniki Ai . . . . . . . . . . . . . . . . . . . . . . . . . . . . . . . . . . . . . . 538.2 Współczynniki Ai dla liczb pierwszych . . . . . . . . . . . . . . . . . . . . . . . . . . 548.3 Zastosowania dla liczb pierwszych i iloczynów . . . . . . . . . . . . . . . . . . . . . . 558.4 Sumy odwrotności iloczynów . . . . . . . . . . . . . . . . . . . . . . . . . . . . . . . . 578.5 Odwrotności liczb względnie pierwszych: podstawowe fakty . . . . . . . . . . . . . . . 588.6 Odwrotności liczb względnie pierwszych: Twierdzenia Gessela . . . . . . . . . . . . . 608.7 Twierdzenie Wolstenholme i inne twierdzenia . . . . . . . . . . . . . . . . . . . . . . . 618.8 Różne zadania . . . . . . . . . . . . . . . . . . . . . . . . . . . . . . . . . . . . . . . . 64

9 Liczby postaci x1/x2 + x2/x3 + · · ·+ xs/x1 669.1 Podstawowe własności zbiorów Bs i As . . . . . . . . . . . . . . . . . . . . . . . . . . 669.2 Zbiór B2 . . . . . . . . . . . . . . . . . . . . . . . . . . . . . . . . . . . . . . . . . . . 699.3 Zbiór B3 i liczby (a3 + b3 + c3)/abc . . . . . . . . . . . . . . . . . . . . . . . . . . . . 709.4 Nieskończoność zbioru A3 . . . . . . . . . . . . . . . . . . . . . . . . . . . . . . . . . . 729.5 Przykłady liczb naturalnych należących do A3 . . . . . . . . . . . . . . . . . . . . . . 739.6 Występowanie danej liczby w rozkładach liczb ze zbioru A3 . . . . . . . . . . . . . . . 779.7 Zbiór B3 . . . . . . . . . . . . . . . . . . . . . . . . . . . . . . . . . . . . . . . . . . . 799.8 Liczby postaci x/y + y/z + z/x, gdzie x, y, z są liczbami całkowitymi . . . . . . . . . 839.9 Zbiór A4 . . . . . . . . . . . . . . . . . . . . . . . . . . . . . . . . . . . . . . . . . . . 85

Spis cytowanej literatury 88

Skorowidz 91

ii

Page 5: Liczby wymierne

Wstęp

Głównym tematem prezentowanej serii książek są liczby i ich przeróżne własności. Autorod najmłodszych lat zbierał wszelkie fakty i ciekawostki dotyczące najpierw liczb całkowitychi wielomianów o współczynnikach całkowitych, a następnie dotyczące również liczb wymier-nych, rzeczywistych, zespolonych oraz wielomianów nad tymi zbiorami liczbowymi. Nazbie-rało się sporo interesującego materiału, którego wybrane fragmenty będą tu przedstawione.

Materiał pochodzi z wielu różnych źródeł. Są tu zadania i problemy, które znajdziemy wpopularnych czasopismach matematycznych. Wśród tych czasopism jest wychodzące od 1894roku (przeważnie 10 numerów w roku) The American Mathematical Monthly. Są wśród tychczasopism również: angielskie czasopismo Mathematical Gazette, , kanadyjskie Crux Mathe-maticorum, rosyjskie Kwant, chińskie Matematical Excalibur, itp. Godnymi uwagi są równieżpolskie czasopisma popularno-naukowe: Delta, czasopismo dla nauczycieli Matematyka orazinne.

Istotną rolę w prezentowanym materiale odegrały zadania z olimpiad i konkursów mate-matycznych całego świata. Każdego roku pojawiają się opracowania, książki oraz artykułydotyczące zadań z różnych zawodów matematycznych. Wspomnijmy tylko o prestiżowychseriach książek z zawodów International Mathematical Olympiad oraz Putnam MathematicalCompetition. Sporo oryginalnych zadań znajduje się w opracowaniach dotyczących olimpiadmatematycznych w Rosji lub w państwach byłego Związku Radzieckiego. Polska również mawartościowe serie tego rodzaju książek.

Zebrany materiał pochodzi również z różnych starych oraz współczesnych podręcznikówi książek z teorii liczb. Wykorzystano liczne książki popularno-naukowe oraz prace naukowepublikowane w różnych czasopismach specjalistycznych. Są tu też pewne teksty pochodzącez internetu.

Większość prezentowanych faktów ma swoje odnośniki do odpowiedniej literatury. Odnoś-niki te wskazują tylko wybrane miejsca, w których można znaleźć albo informacje o danymzagadnieniu, albo rozwiązanie zadania, albo opowiedni dowód. Bardzo często omawiany tematjest powtarzany w różnych pozycjach literatury i często trudno jest wskazać oryginalne źródła.Jeśli przy danym zagadnieniu nie ma żadnego odnośnika do literatury, to oznacza to, że alboomawiany fakt jest oczywisty i powszechnie znany, albo jest to własny wymysł autora.

Elementarna teoria liczb jest wspaniałym źródłem tematów zachęcających do pisaniawłasnych programów komputerowych, dzięki którym można dokładniej poznać badane prob-lemy. Można wykorzystać znane komputerowe pakiety matematyczne: MuPad, Mathematica,CoCoA, Derive, Maple i inne. W prezentowanej serii książek znajdziemy sporo wyników itabel uzyskanych głównie dzięki pakietowi Maple.

We wszystkich książkach z serii ”Podróże po Imperium Liczb” stosować będziemy jedno-lite oznaczenia. Zakładamy, że zero nie jest liczbą naturalną i zbiór {1, 2, 3, . . . }, wszystkichliczb naturalnych, oznaczamy przez N. Przez N0 oznaczamy zbiór wszystkich nieujemnychliczb całkowitych, czyli zbiór N wzbogacony o zero. Zbiory liczb całkowitych, wymiernych,rzeczywistych i zespolonych oznaczamy odpowiednio przez Z, Q, R oraz C. Zbiór wszystkichliczb pierwszych oznaczamy przez P.

1

Page 6: Liczby wymierne

Największy wspólny dzielnik liczb całkowitych a1, . . . , an oznaczamy przez nwd(a1, . . . , an)lub, w przypadkach gdy to nie prowadzi do nieporozumienia, przez (a1, . . . , an). Natomiastnajmniejszą wspólną wielokrotność tych liczb oznaczamy przez nww(a1, . . . , an) lub [a1, . . . , an].Zapis a | b oznacza, że liczba a dzieli liczbę b. Piszemy a - b w przypadku, gdy a nie dzielib. Część całkowitą liczby rzeczywistej x oznaczamy przez [x]. Jeśli m jest liczbą natural-ną, to ϕ(m) jest liczbą wszystkich liczb naturalnych mniejszych lub równych m i względniepierwszych z liczbą m.

Pewne zamieszczone tutaj fakty przedstawione są wraz z ich dowodami. Początek dowoduoznaczono przez D.. Pojawiają się również symbole R., U., W. oraz O. informujące odpo-wiednio o początku rozwiązania, uwagi, wskazówki i odpowiedzi. Wszystkie tego rodzajuteksty zakończone są symbolem �. Skrót ”Odp.” również oznacza odpowiedź.

Spis cytowanej literatury znajduje się na końcu tej książki. W pewnych podrozdziałachpodano również literaturę dodatkową lub uzupełniającą. Informuje o tym symbol F.

2

Page 7: Liczby wymierne

o o o o o

Pierwsza książka z serii ”Podróże po Imperium Liczb” poświęcona jest liczbom wymier-nym, czyli liczbom postaci ab , gdzie a i b są liczbami całkowitymi, przy czym b jest różne odzera.Książka ta składa się z dziewięciu rozdziałów. Trzy z tych rozdziałów (rozdziały 2, 3 i

4) są o liczbach wymiernych postaci 1n , gdzie n jest liczbą naturalną. Takie liczby nazywa-ją się ułamkami prostymi lub ułamkami egipskimi. W rozdziałach tych badane są główniezagadnienia związane z rozkładami liczb wymiernych na skończoną sumę ułamków prostych.Ułamkami prostymi zajmujemy się również w rozdziale 8. W 1862 roku J. Wolstenholme

udowodnił, że jeśli p > 5 jest liczbą pierwszą, to licznik ułamkaab = 1 +

12 +

13 + · · ·+

1p−1

jest podzielny przez p2. Przedstawiamy dwa różne dowody tego faktu. W rozdziale 8 podajemyprzeróżne warianty twierdzenia Wolstenholme oraz zajmujemy się problemami stowarzyszo-nymi z tym twierdzeniem. Główną motywacją do powstania tego rozdziału była piękna pracaI. M. Gessela, opublikowana w 1998 roku w The American Mathematical Monthly. Rozdziały6 i 7 pełnią funkcję pomocniczą. Przedstawiamy w nich zagadnienia potrzebne do zrozumieniapewnych dowodów z rozdziału 8.Każda liczba rzeczywista ma swoje dokładnie jedno nieskończone rozwinięcie dziesiętne.

Dana liczba rzeczywista jest wymierna wtedy i tylko wtedy, gdy jej nieskończone rozwinięciedziesiętne jest od pewnego miejsca okresowe. Pewnym problemom związanym z tym zagad-nieniem poświęcony jest rozdział 5.W ostatnim rozdziale zajmujemy się dodatnimi liczbami wymiernymi postaci

x1x2+ x2x3 + · · ·+

xs−1xs+ xsx1 ,

gdzie x1, . . . , xs są liczbami naturalnymi. Godnym uwagi jest fakt, że jeśli s = 12, to każ-da liczba naturalna większa od 11 ma powyższą postać. Udowodnił to w 2000 roku A. V.Bondarenko.Załóżmy teraz, że s = 3 i rozpatrzmy zbiór wszystkich liczb naturalnych n takich, że

n = xy +yz +

zx ,

dla pewnych liczb naturalnych x, y, z. Oznaczmy ten zbiór przez A3. Zauważmy, że liczby 3,5 i 6 należą do zbioru A3. Mamy bowiem:

3 = 11 +11 +

11 , 5 =

12 +

24 +

41 , 6 =

212 +

129 +

92 .

Czy liczba 4 należy do zbioru A3? Odpowiedź na to pytanie przez długi czas nie była znana.Rozstrzygnął ten problem dopiero w 2000 roku wspomniany wyżej Bondarenko. Udowodniłon, że 4 do tego zbioru nie należy.Można udowodnić, że do zbioru A3 należy nieskończenie wiele liczb naturalnych. Można

również udowodnić (i to nie jest aż tak bardzo skomplikowane), że zbiór A3 pokrywa się zezbiorem wszytkich liczb naturalnych postaci

a3+b3+c3abc , gdzie a, b, c ∈ N.

Czytelnika zainteresowanego tego rodzaju zagadnieniami zapraszamy do ostatniego rozdzia-łu.

3

Page 8: Liczby wymierne
Page 9: Liczby wymierne

����������������������������������������������������

1 Wstępne informacje o liczbach wymiernych����������������������������������������������������

Każdą liczbę postaci ab , gdzie a i b są liczbami całkowitymi oraz b 6= 0, nazywamy liczbąwymierną. Zbiór wszystkich liczb wymiernych oznaczamy przez Q.

oooooooooooooooooooooooooooooooooooooooooooooooooooooooooooooooooooooo1.1 Równości z liczbami wymiernymioooooooooooooooooooooooooooooooooooooooooooooooooooooooooooooooooooooo

1.1.1. 9−256+10 =96 −

2510 ,

121−6455+40 =

12155 −

6440 ,

8−502+5 =

82 −

505 . ([Kw] 9/72 21).

1.1.2. 14 =1664 =

166664 =

16666664 = · · · . ([Kw] 2/88 42).

1.1.3. 15 =1995 =

199995 =

19999995 = · · · . ([Kw] 2/88 42).

1.1.4. 48 =4998 =

499998 =

49999998 = · · · . ([Kw] 2/88 42).

1.1.5. 2653 =286583 =

28865883 =

2888658883 = · · · . ([KoM] Gy1959).

1.1.6. Zapis a1a2 . . . as oznacza liczbę naturalną, której kolejnymi cyframi są odpowiednioa1, a2, . . . , as. Jeśli a, b, c są takimi cyframi, że ab/bc = a/c, to

a/c = ab/bc = abb/bbc = abbb/bbbc = · · · .

([Fom] 14/64).

1.1.7 (N. Anning). Dany jest ułamek 1010 1 01011100 1 0011 zapisany w dowolnym systemie numeracji.Jeśli w liczniku i mianowniku środkową cyfrę 1 zastąpimy dowolną nieparzystą liczbą nastę-pujących po sobie jedynek, to ułamek ten nie zmieni wartości. ([S64] 159).

1.1.8. (d−b)(d−c)(a−b)(a−c) +(d−c)(d−a)(b−c)(b−a) +

(d−a)(d−b)(c−a)(c−b) = 1. ([BaL] 145).

1.1.9. 14

1·3 +243·5 +

345·7 + · · ·+

n4

(2n−1)(2n+1) =n(n+1)(n2+n+1)6(2n+1) dla n ∈ N. ([Mat] 3/52 49).

1.1.10. 23−123+1 ·

33−133+1 · · · · ·

1003−11003+1 =

33675050 . ([Fom] 18/91).

1.1.11. Jeśli x = b−c1+ab , y =

c−a1+ca , z =

a−b1+ab , to x+ y + z = xyz. ([BaL] 146).

1.1.12. Jeśli x = a−ba+b , y =b−cb+c , z =

c−ac+a , to (1+x)(1+y)(1+ z) = (1−x)(1−y)(1− z).

([BaL] 154).

5

Page 10: Liczby wymierne

6 Liczby wymierne. 1. Wstępne informacje o liczbach wymiernych

1.1.13. Niech x = a2+b2−c22ab , y =

a2+c2−b22ac , z =

b2+c2−a22bc , gdzie a, b, c ∈ N. Jeśli x+y+z =

1, to dwie z liczb x, y, z są równe 1, a pozostała −1. ([OM] Leningrad 1982).

1.1.14. Jeśli 1a +1b +

1c =

1a+b+c , to

1a2n+1 +

1b2n+1 +

1c2n+1 =

1(a+b+c)2n+1 . ([Oss] G75.2-5).

1.1.15. Jeśli a1b1 =a2b2= a3b3 oraz (p1, p2, p3) 6= (0, 0, 0), to(

a1b1

)n= p1a

n1+p2a

n2+p3a

n3

p1bn1+p2bn2+p3b

n3

dla wszystkich n ∈ N. ([OM] Kanada 1969).

oooooooooooooooooooooooooooooooooooooooooooooooooooooooooooooooooooooo1.2 Całkowitość pewnych liczb wymiernychoooooooooooooooooooooooooooooooooooooooooooooooooooooooooooooooooooooo

1.2.1. Niech a, b ∈ Q. Jeśli a+ b ∈ Z i ab ∈ Z, to a, b ∈ Z.

1.2.2. Niech a, b, c ∈ Q. Jeśli a+ b+ c ∈ Z, ab+ bc+ ca ∈ Z i abc ∈ Z, to a, b, c ∈ Z.

1.2.3.Wyznaczyć wszystkie trójki (x, y, z) liczb wymiernych dodatnich, dla których wszystkieliczby x+ y + z, 1x +

1y +

1z , xyz są naturalne. ([OM] Polska 1993/1994).

R. Jeśli (x, y, z) jest taką trójką, to x, y, z są liczbami naturalnymi. Wszystkie trójki (x, y, z)takie, że x > y > z: (1, 1, 1), (3, 3, 3), (2, 2, 1), (6, 3, 2), (4, 4, 2). �

1.2.4. Niech x = a2−1b+1 , y =

b2−1a+1 , gdzie a, b ∈ N. Jeśli x+ y jest liczbą całkowitą, to liczby

x i y też są całkowite. ([OM] St.Petersburg 1993, [Fom] 17/93).

1.2.5. Każda liczba postaci 15n5+ 13n

3+ 715n, gdzie n ∈ N, jest całkowita. ([OM] Australia 1994).

1.2.6. Dla każdej liczby naturalnej n liczba(4− 21

) (4− 22

) (4− 23

)· · ·(4− 2n

)jest całkowita. ([OM] Czechy-Słowacja 1998/1999).

1.2.7. Niech n ∈ N. Liczby 21n−34 i 15n+26 nie mogą być jednocześnie całkowite. ([M-sj] 463).

1.2.8. Każdą liczbę naturalną można przedstawić w postaci ab+1a+b , gdzie a, b są liczbami na-turalnymi. ([OM] Moskwa 1996/1997, [OM] Mołdawia 2001).

D. Niech (a, b) = (2n− 1, 2n+ 1) lub (n+ 1, n2 + n− 1). Wtedy ab+1a+b = n. �

1.2.9. Każdą liczbę naturalną większą od 1 i nie będącą postaci 2n + 2 można przedstawić wpostaci ab +

a+1b+1 , gdzie a, b ∈ N. ([OM] Moskwa 2000/2001).

Page 11: Liczby wymierne

Liczby wymierne. 1. Wstępne informacje o liczbach wymiernych 7

1.2.10. Niech n ∈ N. Znaleźć liczbę wszystkich par (x, y) liczb naturalnych takich, że n =xyx+y . Przykłady: 1 =

2·22+2 , 2 =

3·63+6 =

4·44+4 =

6·36+3 . ([Putn] 1960).

R. Problem sprowadza się do opisu liczby rozwiązań naturalnych równania (x− n)(y − n) = n2.Zachodzi jeden z przypadków: (x− n < 0, y − n < 0) lub (x− n > 0, y − n > 0).Jeśli x − n < 0 i y − n < 0, to 1 6 x < n i 1 6 y < n, stąd −n < x − n < n i −n < y − n < n,

czyli |x− n| < n i |y − n| < n. W tym przypadku mamy sprzeczność: n2 = |x− n||y − n| < n2.Niech x−n > 0 i y−n > 0. Niech (a, b) będzie dowolną parą liczb naturalnych takich, że ab = n2.

Przyjmijmy: x := a + n, y := b + n. Wtedy (x − n)(y − n) = ab = n2. Każda więc taka para (a, b)wyznacza naturalne rozwiązanie rozpatrywanego równania. Takich par jest oczywiście tyle ile jestnaturalnych podzielników liczby n2.Odpowiedź. Liczba wszystkich takich naturalnych par jest równa τ(n2), gdzie τ(n2) jest liczbą

wszystkich podzielników naturalnych liczby n2. Jeśli a jest podzielnikiem naturalnym liczby n2, to(x, y) = (n+ a, n+ n2/a) jest naturalnym rozwiązaniem. Każde rozwiązanie jest tej postaci. �

1.2.11. Każdą liczbę naturalną większą od 1 można przedstawić w postaci a2+bab+1 , gdzie a, b są

liczbami naturalnymi. ([OM] Moskwa 2000/2001).

1.2.12. Niech a, n ∈ N, (a, n) 6= (1, 1). Równanie x2+y2axy+1 = n

2 ma nieskończenie wiele natu-ralnych rozwiązań. ([Crux] 1990 s.172 z.1556).

1.2.13. Jeśli a 6= b ∈ Z, n ∈ N, to liczba 22n−1(a2n+b2n)−(a+b)2n

(a−b)2 jest całkowita. ([OMm] 1997/1998).

1.2.14. Liczba postaci 2a2−1b2+2 , gdzie a, b ∈ Z, nie jest całkowita. ([IMO] Longlist 1992).

1.2.15. Liczba postaci a2+b2a2−b2 , gdzie a, b ∈ N, a 6= b, nie jest całkowita. ([KoM] Gy1959).

1.2.16. Niech x, y ∈ C, x 6= y, an = xn−ynx−y . Jeśli jakieś cztery kolejne wyrazy ciągu (an) są

liczbami całkowitymi, to wszystkie wyrazy tego ciągu są liczbami całkowitymi.([OM] Rumunia 2002).

1.2.17. Niech a, b będą liczbami naturalnymi i niech

xn =(a+ 12

)n+(b+ 12

)n.

W ciągu (xn) jest tylko skończenie wiele liczb całkowitych. (Newman problem 30).

1.2.18. Jeśli liczba (m+3)n+1

3m jest całkowita, to jest nieparzysta. ([IMO] 1967).

1.2.19. Jeśli liczba m2+n2+1mn jest całkowita, to jest równa 3. ([LeH] A5).

1.2.20. Jeśli liczba m2+n2+6mn jest całkowita, to jest sześcianem liczby całkowitej.

([OM] Estonia 1995/1996, [Crux] 2002 s.74).

Page 12: Liczby wymierne

8 Liczby wymierne. 1. Wstępne informacje o liczbach wymiernych

1.2.21. Istnieje nieskończenie wiele par (n,m) liczb naturalnych takich, że 1 < n < m iliczba m

2+n2−1mn jest całkowita. ([Crux] z.1746).

1.2.22 ([Crux] 2001 z.2534 s.276-279).

Oznaczmy za(x, y) =x2+y2+axy . Niech A będzie zbiorem tych wszystkich liczba całkowitych

a, dla których liczba za(x, y) jest całkowita dla nieskończenie wielu par (x, y) liczb natural-nych. Jeśli a ∈ A, to przez E(a) oznaczać będziemy zbiór wszystkich liczb całkowitych postaciza(x, y), x, y ∈ N.(1) Niech a ∈ Z. Jeśli istnieją liczby naturalne x, y takie, że liczba za(x, y) jest całkowita,

to takich par (x, y) ∈ N2 jest nieskończenie wiele.(2) Zbór A jest nieskończony. Każda liczba postaci −d2, gdzie d ∈ N, należy do A. Mamy

bowiem z−d2(λd, d) = λ dla wszystkich λ ∈ N.(3) Liczba 0 należy do A i E(0) = {2}.(4) Jeśli a = −d2, gdzie d ∈ N, to a ∈ A i zbiór E(a) jest nieskończony; jest nawet

równy N. Wynika to z (2).(5) Jeśli a ∈ A i a nie jest postaci −d2, gdzie d ∈ N, to zbiór E(a) jest skończony.(6) Niech a ∈ N0. Niech za(x, y) = β, gdzie x, y, β ∈ N. Wtedy β 6 a+ 2.

1.2.23. Jeśli x, y, z są parami różnymi liczbami całkowitymi i n jest liczbą naturalną, to liczba

xn

(x−y)(x−z) +yn

(y−x)(y−z) +zn

(z−x)(z−y)

jest całkowita. ([Kurs] 175(1959), [Bryn] 1.1).

1.2.24. Znaleźć wszystkie pary (m,n) liczb naturalnych, dla których n3+1mn−1 jest liczbą całko-

witą. Odp. (2, 2), (2, 1), (1, 2), (3, 1), (1, 3), (5, 2), (2, 5), (5, 3), (3, 5). ([IMO] 1994).

1.2.25. Niech a, b ∈ Z. Jeśli a2

2ab2−b3+1 jest liczbą całkowitą, to (a, b) = (2n, 1) lub (n, 2n) lub(8n4 − n, 2n), gdzie n ∈ N. ([IMO] Shortlist 2003).

1.2.26. Znaleźć wszystkie pary (x, y) liczb naturalnych, dla których liczby x+1y ,y+1x są na-

turalne. Odp. (3, 2), (2, 3), (1, 1), (2, 1), (1, 2). ([OM] Polska 1994/1995).

1.2.27. Niech a = (x+y+z)2

xyz , gdzie x, y, z ∈ N. Jeśli a jest liczbą całkowitą, to a = 1, 2,3, 4, 5, 6, 8 lub 9 (a nie może być siódemką). ([OM] Mongolia 2000).

1.2.28. Jeśli p jest liczbą pierwszą i n ∈ N, to liczba

11+1(p−1) +

21+2(p−1) + · · ·+

n1+n(p−1)

nie jest całkowita. ([Mon] 96(8)(1989) E3249).

Page 13: Liczby wymierne

Liczby wymierne. 1. Wstępne informacje o liczbach wymiernych 9

oooooooooooooooooooooooooooooooooooooooooooooooooooooooooooooooooooooo1.3 Przedstawianie liczb wymiernych w szczególnej postacioooooooooooooooooooooooooooooooooooooooooooooooooooooooooooooooooooooo

1.3.1. Dowolny ułamek nieskracalny p/q, gdzie p, q są liczbami naturalnymi, przy czym qjest nieparzyste, można przedstawić w postaci n

2k−1 , dla pewnych liczb naturalnych n i k.([Balt] 1995).

1.3.2. Każda liczba wymierna z odcinka (0, 1) o nieparzystym mianowniku jest postaci{xyz

x2+y2+z2

},

dla pewnych liczb naturalnych x, y, z, gdzie {a} oznacza część ułamkową liczby a.([KoM] 1997(7) N 146).

1.3.3. Niech a ∈ N. Każda liczba wymierna z odcinka (0, 1) jest skończonym iloczynem liczbpostaci n(n+3)

(n+1)(n+2) , gdzie n > a. ([Mon] 98(2)(1991) E3347).

1.3.4. Każda liczba wymierna w 6= 1 ma nieskończenie wiele przedstawień w postaci w =x+1y+1 , gdzie x i y są względnie pierwszymi liczbami całkowitymi. ([Wm] 7 37).

1.3.5. Niech m ∈ N. Każdą liczbę wymierną w > 1 można przedstawić w postaci

w =(1 + 1k

) (1 + 1

k+1

)· · ·(1 + 1

k+s

),

gdzie k jest liczbą naturalną > m oraz s jest nieujemną liczbą całkowitą.([Mat] 1/58 60, [S64] 199).

1.3.6. Każda liczba wymierna dodatnia w ma jednoznaczne przedstawienie w postaci

w = a1 + a22! +a33! + · · ·+

akk! ,

gdzie a1, a2, . . . ak są nieujemnymi liczbami całkowitymi takimi, że a2 < 2, a3 < 3, . . . , ak < koraz ak 6= 0. ([Mat] 1/80 62).

oooooooooooooooooooooooooooooooooooooooooooooooooooooooooooooooooooooo1.4 Podzbiory zbioru liczb wymiernychoooooooooooooooooooooooooooooooooooooooooooooooooooooooooooooooooooooo

1.4.1. Znaleźć wszystkie podzbiory S ⊆ Q spełniające następujące warunki:(1) jeśli a, b ∈ S, to a+ b ∈ S;(2) jeśli a jest niezerową liczbą wymierną, to dokładnie jedna z liczb a i −a należy do S.

([Bryn] 2.1).

Page 14: Liczby wymierne

10 Liczby wymierne. 1. Wstępne informacje o liczbach wymiernych

O. Są cztery takie podzbiory; zbiory wszystkich liczb wymiernych: dodatnich, nieujemnych,ujemnych, niedodatnich. �

1.4.2. Niech S będzie podzbiorem zbioru liczb wymiernych zawierającym 12 i spełniającym

warunekx ∈ S =⇒ 1

x+1 ∈ S ixx+1 ∈ S.

Wtedy każda liczba wymierna z przedziału (0, 1) należy do S. ([OM] W.Brytania 2005).

D. (Indukcja ze względu na mianowniki ułamków). Niech n > 3 i załóżmy, że każdy ułamek abtaki, że b < n oraz 1 6 a < b, należy do S. Rozważmy ułamek pn , gdzie 1 6 p < n. Niech q = n − p.Oczywiście q < n. Jeśli p = q, to n = 2p i wtedy pn =

12 ∈ S. Jeśli p < q, to (na mocy założenia)

x = pq ∈ S i stądpn =

xx+1 ∈ S. Jeśli p > q, to x =

qp ∈ S i stąd

pn =

1x+1 ∈ S. �

oooooooooooooooooooooooooooooooooooooooooooooooooooooooooooooooooooooo1.5 Różne zadania o liczbach wymiernychoooooooooooooooooooooooooooooooooooooooooooooooooooooooooooooooooooooo

1.5.1. Jeżeli liczba naturalna nie jest k-tą potęgą liczby naturalnej, to nie jest też k-tą potęgąliczby wymiernej. ([S50] s.6).

1.5.2. Niech a, b ∈ R, a + b = 1. Jeśli liczby a3 i b3 są wymierne, to a i b też są liczbamiwymiernymi. ([OM] Polska 1994/1995).

1.5.3. Istnieje nieskończenie wiele par (x, y) liczb wymiernych takich, że x 6= y oraz√x2 + y3

i√x3 + y2 są liczbami wymiernymi. ([OM] Niemcy 2003/2004).

1.5.4. Niech S = Qr{−1, 0, 1}, f : S → S, f(x) = x− 1x . Wtedy∞⋂n=1fn(S) = ∅. ([Putn] 2001).

1.5.5. Jedyną funkcją f : Q→ Q taką, że f(1) = 2 oraz

f(xy) = f(x)f(y)− f(x+ y) + 1 dla x, y ∈ Q,

jest funkcja f(x) = x+ 1. ([Bryn] 6.1).

1.5.6. Funkcja f : Q×Q→ Q spełnia następujące własności:(1) f(a, b) = f(b, a),

(2) f(f(a, b), c) = f(a, f(b, c)),

(3) f(0, 0) = 0,

(4) f(a+ c, b+ c) = f(a, b) + c,

dla wszystkich a, b, c ∈ Q. Wykazać, że f(a, b) = max(a, b) dla wszystkich a, b ∈ Q lubf(a, b) = min(a, b) dla wszystkich a, b ∈ Q. (H. Derksen 1997).

Page 15: Liczby wymierne

Liczby wymierne 1. Wstępne informacje o liczbach wymiernych 11

1.5.7. Podać przykład podpierścienia ciała Q różnego od Z i Q.

1.5.8. Niech S ⊆ N będzie zbiorem multyplikatywnym. (tzn. 1 ∈ S oraz a, b ∈ S ⇒ ab ∈ S).Niech

ZS ={x ∈ Q; ∃a∈Z∃s∈S x =

a

s

}.

Wykazać, że:

(1) ZS jest podpierścieniem ciała Q;(2) Każdy podpierścień ciała Q jest postaci ZS dla pewnego multyplikatywnego podzbioru

S zbioru N.

1.5.9. Znaleźć najmniejszy podpierścień ciała Q zawierający ułamki 1/2 i 1/3.

1.5.10. Jeśli a, b ∈ Z, b 6= 0, to przez Z[a/b] oznaczamy najmniejszy podpierścień ciała Qzawierający liczbę a/b.

(1) Wykazać, że Z[2/7] = Z[3/7].(2) Jeśli a, b ∈ Z, b 6= 0 oraz nwd(a, b) = 1, to Z[a/b] = Z[1/b].

F W. N. Wagutien, O ułamkach Farey’a, [Kw] 8/75 33-39.N. J. Wilenkin, Z historii ułamków, [Kw] 5/87 34-36.P. W. Śniady, Teoria liczb i geometria, (o ułamkach Farey’a), [Dlt] 4/95 1-3.J. Wróblewski, Własności przystawania liczb wymiernych i zespolonych, [Mat] 2/80 113-116.

Page 16: Liczby wymierne

����������������������������������������������������

2 Rozkłady jedynki na ułamki proste����������������������������������������������������

Ułamkiem prostym nazywamy każdą dodatnią liczbę wymierną postaci 1n , gdzie n jestliczbą naturalną.

oooooooooooooooooooooooooooooooooooooooooooooooooooooooooooooooooooooo2.1 Ogólne fakty o rozkładach jedynkioooooooooooooooooooooooooooooooooooooooooooooooooooooooooooooooooooooo

2.1.1. Dla dowolnej liczby naturalnej s równanie

1x1+ 1x2+ · · ·+ 1

xs= 1

ma co najmniej jedno naturalne rozwiązanie i naturalnych rozwiązań jest skończenie wiele.([S64] 144).

2.1.2. Jeśli s > 3 jest liczbą naturalną, to równanie 1x1+1x2+· · ·+ 1xs = 1 posiada co najmniej

jedno naturalne rozwiązanie takie, że x1 < x2 < · · · < xs. Jeśli liczbę takich rozwiązańoznaczymy przez ls, to ls+1 > ls. Przykłady: l3 = 1, l4 = 6, l5 = 72, l6 = 2320. ([S64] 145, Maple).

Pierwsza część powyższej tezy wynika natychmiast z następującego faktu, którego łatwo udowodnićmetodą indukcji matematycznej.

2.1.3. Rozważmy ciąg (an), liczb naturalnych zdefiniowanych rekurencyjnie następująco:

a1 = 2, an+1 = a1a2 · · · an + 1, dla n ∈ N.

Dla każdej liczby naturalnej n zachodzi równość

1a1+1a2+ · · ·+ 1

an+

1a1a2 · · · an

= 1.

([MM] 34(1)(1960) z.397).

2.1.4. Jeśli (an) jest ciągiem takim, że a1 = 2 i an+1 = a1a2 · · · an + 1, dla n ∈ N, to1a1+ 1a2 + · · ·+

1an< 1. ([Fom] 38/86, wynika z 2.1.3).

2.1.5. Dla każdej liczby naturalnej n liczbę 1 można przedstawić jako sumę parami różnychułamków prostych o mianownikach podzielnych przez n. ([Cmj] 9(1)(1978) s.43).

2.1.6. Rozkłady jedynki na sumę n parami różnych ułamków prostych dla n > 3:

(1) 1 = 121 +

122 + · · ·+

12n−2 +

13·2n−3 +

16·2n−3 .

(2) Niech r1 = 1, rn+1 = rn(rn + 1). Wtedy 1 = 1r1+1+ · · ·+ 1

rn−1+1+ 1rn. ([Dlt] 4/84 16).

12

Page 17: Liczby wymierne

Liczby wymierne. 2. Rozkłady jedynki na ułamki proste 13

2.1.7. Jeśli x1 < x2 < · · · < xs są liczbami naturalnymi takimi, że 1x1 +1x2+ · · ·+ 1xs = 1, to

xs < s2s−1. ([Cmj] 28(1)(1997) s.71).

2.1.8. Jeśli x1 < x2 < · · · < xs są liczbami naturalnymi takimi, że 1x1 +1x2+ · · ·+ 1xs = 1, to

xs < 2s!. ([Ko04] 166).

2.1.9. Niech x1, x2, . . . , xs, xs+1 będą dodatnimi liczbami rzeczywistymi takimi, że

11+x1+ 11+x2+ · · ·+ 1

1+xs+1= 1.

Wtedy x1x2 · · ·xs+1 > ss+1. ([Crux] 2000 s.167).

oooooooooooooooooooooooooooooooooooooooooooooooooooooooooooooooooooooo2.2 Jedynka i dwa ułamki prosteoooooooooooooooooooooooooooooooooooooooooooooooooooooooooooooooooooooo

2.2.1. Jedynym rozwiązaniem naturalnym równania 1x +1y = 1 jest para (x, y) = (2, 2).

2.2.2. Jeśli x, y są liczbami naturalnymi takimi, że 1x +1y < 1, to

1x +

1y 6 5

6 .

D. Załóżmy, że x 6 y są liczbami naturalnymi takimi, że 1x +1y < 1. Wtedy jest oczywiste, że

x > 2 i y > 3. Mamy więc: 1x +1y 6 1

2 +13 =

56 . �

oooooooooooooooooooooooooooooooooooooooooooooooooooooooooooooooooooooo2.3 Jedynka i trzy ułamki prosteoooooooooooooooooooooooooooooooooooooooooooooooooooooooooooooooooooooo

2.3.1. Znaleźć wszystkie trójki (x, y, z) liczb naturalnych takich, że x 6 y 6 z oraz

1x +

1y +

1z = 1.

Odp. (2, 4, 4), (2, 3, 6), (3, 3, 3). ([GaT] 1/48, [Kw] 1/88 44).

2.3.2. Nie istnieją liczby naturalne a i b takie, że 1a2 +1ab +

1b2 = 1. ([WyKM] 150-52).

2.3.3. Jedynymi całkowitymi rozwiązaniami równania 1x2 +1xy +

1x2 = 1 są pary (1,−1) i

(−1, 1). ([Br80] 46).

2.3.4. Jeśli x, y, z są liczbami naturalnymi takimi, że 1x +1y +

1z < 1, to

1x +

1y +

1z 6 41

42 .([Fom] 15/86).

Page 18: Liczby wymierne

14 Liczby wymierne. 2. Rozkłady jedynki na ułamki proste

oooooooooooooooooooooooooooooooooooooooooooooooooooooooooooooooooooooo2.4 Jedynka i cztery ułamki prosteoooooooooooooooooooooooooooooooooooooooooooooooooooooooooooooooooooooo

2.4.1. Równanie 1x +1y +

1z +

1t = 1 ma dokładnie 14 naturalnych rozwiązań takich, że

x 6 y 6 z 6 t. Rozwiązaniami tymi są:

(2, 3, 7, 42), (2, 3, 8, 24), (2, 3, 9, 18), (2, 3, 10, 15), (2, 3, 12, 12), (2, 4, 5, 20), (2, 4, 6, 12), (2, 4, 8, 8),(2, 5, 5, 10), (2, 6, 6, 6), (3, 3, 4, 12), (3, 3, 6, 6), (3, 4, 4, 6), (4, 4, 4, 4).

Wśród nich jest dokładnie 6 takich rozwiązań, w których liczby x, y, z, t są parami różne.([S64] 143, Maple).

2.4.2. Znaleźć wszystkie liczby naturalne x, y, z spełniające równość 1x +1y +

1z +

1xyz = 1.

Odp. {x, y, z} = {2, 3, 7}. ([Mat] 2/1985 z.1134).

2.4.3. Trójka (1, 2, 4) (wraz z jej permutacjami) jest jedynym rozwiązaniem naturalnym rów-nania 1xy +

1yz +

1zx +

1xyz = 1. ([Bedn] 37).

oooooooooooooooooooooooooooooooooooooooooooooooooooooooooooooooooooooo2.5 Jedynka i pięć ułamków prostychoooooooooooooooooooooooooooooooooooooooooooooooooooooooooooooooooooooo

2.5.1. 1 = 13 +14 +

15 +

16 +

120 . ([S59] 237).

2.5.2. 1 = 12 +13 +

17 +

178 +

191 , 1 =

12 +

15 +

16 +

110 +

130 , 1 =

12 +

15 +

16 +

112 +

120 . (Maple).

2.5.3. Równanie 1x1 +1x2+ 1x3 +

1x4+ 1x5 = 1 ma dokładnie 147 naturalnych rozwiązań takich,

że x1 6 x2 · · · 6 x5. Wśród nich są dokładnie 72 takie rozwiązania, w których liczby x1, . . . , x5są parami różne. (Maple).

oooooooooooooooooooooooooooooooooooooooooooooooooooooooooooooooooooooo2.6 Jedynka i sześć ułamków prostychoooooooooooooooooooooooooooooooooooooooooooooooooooooooooooooooooooooo

2.6.1. 1 = 13 +14 +

16 +

17 +

112 +

142 . ([S59] 237).

2.6.2. 1 = 12 +13 +

110 +

124 +

172 +

190 , 1 =

13 +

14 +

16 +

110 +

112 +

115 . (Maple).

2.6.3. Równanie 1x1 +1x2+ · · ·+ 1x6 = 1 ma dokładnie 3462 naturalnych rozwiązań takich, że

x1 6 x2 · · · 6 x6. Wśród nich jest dokładnie 2320 takich rozwiązań, w których liczby x1, . . . , x6są parami różne. (Maple).

2.6.4. Jeśli x1, . . . , x6 są liczbami naturalnymi takimi, że 1x1 + · · ·+1x6= 1, to co najmniej

jedna z liczb x1, . . . , x6 jest parzysta. ([G-if] 210).

Page 19: Liczby wymierne

Liczby wymierne. 2. Rozkłady jedynki na ułamki proste 15

oooooooooooooooooooooooooooooooooooooooooooooooooooooooooooooooooooooo2.7 Jedynka i więcej niż sześć ułamków prostychoooooooooooooooooooooooooooooooooooooooooooooooooooooooooooooooooooooo

2.7.1 (Maple). Jedynka i 7 ułamków prostych:

1 = 13 +14 +

18 +

110 +

112 +

115 +

124 ; 1 =

13 +

14 +

19 +

110 +

112 +

115 +

118 .

2.7.2 (Maple). Jedynka i 8 ułamków prostych:

1 = 13 +14 +

16 +

18 +

124 +

130 +

136 +

145 ; 1 =

13 +

14 +

16 +

112 +

120 +

121 +

128 +

130 .

2.7.3. Równanie 1x +1y +

1z +

5xyz = 1, gdzie x 6 y 6 z, ma dokładnie dwa rozwiązania

naturalne (2, 3, 11) i (2, 5, 5). ([Mat] 4/2001 z.1513).

2.7.4. Niech s ∈ N i niech x1, . . . , xs będą takimi liczbami naturalnymi, że 1x1 + · · ·+1xs= 1.

Jeśli liczba s jest parzysta, to co najmniej jedna z liczb x1, . . . , xs jest również parzysta.

D. Niech s będzie liczbą parzystą i przypuśćmy, że wszystkie liczby x1, . . . , xs są nieparzyste.Oznaczmy: m = x1x2 · · ·xs oraz mi = mxi dla i = 1, . . . , s. Wtedy wszytkie liczby m1, . . . ,ms oraz msą nieparzyste i mamy równośćm = m1+· · ·+ms. Po lewej stronie tej równości jest liczba nieparzysta,a po prawej stronie mamy liczbę parzystą. �

2.7.5. Niech s ∈ N i niech x1, . . . , xs będą takimi liczbami naturalnymi, że 1x1 + · · ·+1xs= 1.

Jeśli s 6 8, to co najmniej jedna z liczb x1, . . . , xs jest parzysta. Dla nieparzystych s większychlub równych 9 już tak nie musi być. ([MM] 24(1)(1950) z.51).

2.7.6. Jedynka i 9 parami różnych ułamków prostych o nieparzystych mianownikach.

(1) 1 = 13 +15 +

17 +

19 +

111 +

115 +

135 +

145 +

1231 . (P.N.Nagara [MM] 24(1)(1950), [Dlt] 3/83 17).

(2) 1 = 13 +15 +

17 +

19 +

111 +

115 +

133 +

145 +

1385 . (O.H.Hoke [MM] 24(1)(1950)).

(3) 1 = 13 +15 +

17 +

19 +

111 +

115 +

121 +

1165 +

1693 . (P.N.Nagara [MM] 24(1)(1950)).

2.7.7. Liczba wszystkich ciągów (x1, . . . , x10) liczb naturalnych takich, że 1x1 + · · ·+1x10= 1,

jest nieparzysta. ([Putn] 1997, [Zw] 2000).

2.7.8. Jedynka i 11 parami różnych ułamków prostych o nieparzystych mianownikach.

(1) 1 = 13 +17 +

121 +

127 +

135 +

145 +

163 +

1105 +

1135 +

1189 +

1315 . (F.L.Miksa [MM] 24(1)(1950)).

(2) 1 = 13 +15 +

17 +

19 +

115 +

121 +

127 +

135 +

145 +

1105 +

1945 . ([S59] 237).

2.7.9. 1 = 13 +15 +

17 +

19 +

115 +

121 +

135 +

145 +

163 +

175 +

1105 +

1135 +

1675 . ([S59] 237).

Page 20: Liczby wymierne

16 Liczby wymierne 2. Rozkłady jedynki na ułamki proste

oooooooooooooooooooooooooooooooooooooooooooooooooooooooooooooooooooooo2.8 Różne fakty o rozkładach jedynkioooooooooooooooooooooooooooooooooooooooooooooooooooooooooooooooooooooo

2.8.1. Niech A będzie zbiorem wszystkich liczb naturalnych n, dla których istnieją liczbynaturalne a1, . . . , ak (niekoniecznie różne) takie, że a1 + · · ·+ ak = n i 1

a1+ · · ·+ 1

ak= 1.

Wykazać, że:

(1) każda liczba kwadratowa należy do A;(2) jeśli n,m ∈ A, to nm ∈ A;(3) jeśli n ∈ A, to 2n+ 2 ∈ A;(4) jeśli n ∈ A, to 2n+ 9 ∈ A;(5) 1, 4, 9, 10, 11, 16, 17, 18, 20, 22 ∈ A;(6) jeśli n > 24, to n ∈ A. ([OM] USA 1978, [Kw] 2/80 37, [IMO] Longlist 1992).

2.8.2. Niech A będzie zbiorem wszystkich liczb naturalnych n, dla których istnieją liczbynaturalne a1, . . . , ak (niekoniecznie różne) takie, że a1 + · · ·+ ak = n i 1

a1+ · · ·+ 1

ak= 1.

Wiedząc, że wszystkie liczby naturalne z przedziału [33, 73] należą do zbioru A wykazać, żekażda liczba naturalna większa od 32 należy do zbioru A. ([OM] USA 1978).

D. ([San4j]). Najpierw wykażemy, że jeśli n ∈ A, to 2n + 8 ∈ A oraz 2n + 9 ∈ A. Niech n =a1 + a2 + · · ·+ ak i 1a1 + · · ·+

1ak= 1. Wtedy 2n+ 8 = 2(a1 + a2 + · · ·+ ak) + 4 + 4 oraz

12a1+ 12a2+ · · · 12ak +

14 +

14 =

12 +

14 +

14 = 1.

Ponadto, 2n + 9 = 2(a1 + a2 + · · · + ak) + 3 + 6 oraz 12a1+ 12a2+ · · · 12ak +

13 +

16 =

12 +

13 +

16 = 1.

Zatem

(∗) n ∈ A =⇒ 2n+ 8 ∈ A i 2n+ 9 ∈ A.

Niech P (n) oznacza zdanie ”wszystkie liczby naturalne od n do 2n+7 należą do zbioru A”. Z założeniawiemy, że zdanie P (33) jest prawdziwe. Z (∗) natomiast wynika, że jeśli zdanie P (n) jest prawdziwe,to zdanie P (n+ 1) jest również prawdziwe. Teza wynika zatem na mocy indukcji matematycznej. �

F O. T. Iżboldin, L. D. Kurlandczyk, Rozbicie jedynki, [Kw] 7/87 48-52.

Page 21: Liczby wymierne

����������������������������������������������������

3 Rozkłady liczb wymiernych na ułamki proste����������������������������������������������������

oooooooooooooooooooooooooooooooooooooooooooooooooooooooooooooooooooooo3.1 Rozkłady liczb naturalnychoooooooooooooooooooooooooooooooooooooooooooooooooooooooooooooooooooooo

3.1.1. 2 = 12 +13 +

14 +

15 +

16 +

17 +

18 +

19 +

110 +

115 +

1230 +

157960 . ([Mat] 3/56 5, [S59] 236).

3.1.2. 2 = 12+13+14+15+16+18+

110+

112+

115+

120+

124+

130+

140+

160+

1120 . ([Mat] 3/56 5, [S59] 236).

3.1.3. Istnieją parami różne liczby naturalne a1, . . . , a1974 takie, że 1a1 + · · · +1a1974

= 2.([WyKM] 637-74).

D. Startujemy od równości 2 = 11+12+13+16 i dla ostatniego ułamka kilkakrotnie wykorzystujemy

równość 16k =

112k +

120k +

130k . �

3.1.4. Niech x1, x2, . . . , xn będą takimi liczbami naturalnymi, że żadna z nich nie jest począ-tkowym fragmentem żadnej innej (na przykład 12 jest początkowym fragmentem liczb 12, 125lub 12405). Zachodzi wtedy nierówność: 1x1 +

1x2+ · · ·+ 1

xn< 3. ([Balt] 2000).

3.1.5. Każda liczba naturalna jest skończoną sumą parami różnych ułamków prostych o nie-parzystych mianownikach. ([S57a] 75).

3.1.6. Każda liczba naturalna jest skończoną sumą parami różnych ułamków prostych o bez-kwadratowych mianownikach. ([Cmj] 9(1)(1978) s.43).

3.1.7. Znaleźć wszystkie trójki x 6 y 6 z liczb naturalnych, dla których 1x +1y +

1z jest liczbą

naturalną. Odp. (1, 1, 1), (1, 2, 2), (2, 3, 6), (2, 4, 4), (3, 3, 3). ([Jedr] B.2).

3.1.8. Istnieje taka permutacja (xn) zbioru wszystkich liczb naturalnych, że dla każdej liczbynaturalnej m istnieje n takie, że m = 1

x1+ 1x2+ · · ·+ 1

xn. ([OM] Norwegia 1995).

oooooooooooooooooooooooooooooooooooooooooooooooooooooooooooooooooooooo3.2 Rozkłady liczb wymiernychoooooooooooooooooooooooooooooooooooooooooooooooooooooooooooooooooooooo

3.2.1. Każda liczba wymierna z przedziału (0, 1) jest skończoną sumą parami różnych ułam-ków prostych. ([Mat] 6/56 59, [Br80] 27, [B-rs] 259).

3.2.2. Każda liczba wymierna jest skończoną sumą parami różnych ułamków prostych.([S57a] 46, [Mon] NT-227).

17

Page 22: Liczby wymierne

18 Liczby wymierne. 3. Rozkłady liczb wymiernych na ułamki proste

3.2.3. Każdą liczbę wymierną w z przedziału (0, 1) można przedstawić w postaci

w = 1q1+ 1q2 + · · ·+

1qr,

gdzie q1, . . . , qr są liczbami naturalnymi takimi, że q1 < q2 < · · · < qr oraz qk dzieli qk+1 dlak = 1, . . . , r − 1. ([Kw] 2/71 29 M24).

U. Przedstawienie takie nie musi być jednoznaczne. Mamy np. 38 =14 +

18 =

13 +

124 . �

3.2.4. Niech k ∈ N i niech w ∈ Q. Równanie w = 1x1+ 1x2 + · · ·+

1xkposiada tylko skończenie

wiele rozwiązań naturalnych. ([Isaa] s.48).

3.2.5. 22k+1 =

12k+1 +

12k+3 +

1(k+2)(2k+1) +

1(k+2)(2k+1)(2k+3) . ([S59] 238).

3.2.6. 22k+1 =

12k+1 +

12k+3 +

1(k+1)(2k+3) +

1(k+1)(2k+1)(2k+3) . ([S59] 238).

3.2.7. Niech n > 2 będzie liczbą naturalną. Rozważmy ułamki postaci 1ab , gdzie a i b sąwzględnie pierwszymi liczbami naturalnymi takimi, że

a < b 6 n, a+ b > n.

Suma wszystkich takich ułamków jest równa 12 . ([Ibe] 1996, [Crux] 2002 s.215).

3.2.8. Dla każdej liczby naturalnej n > 6 istnieją liczby naturalne x1, . . . , xn takie, że 19971998 =1x1+ 1x2+ · · ·+ 1

xn. Ponadto, w każdym takim rozkładzie istnieją dwie liczby xi, xj, które nie

są względnie pierwsze. Dla n = 6 mamy: 19971998 =12 +

13 +

19 +

137 +

137 +

1999 , ([OM] Szwecja 1998).

oooooooooooooooooooooooooooooooooooooooooooooooooooooooooooooooooooooo3.3 Sumy dwóch ułamków prostychoooooooooooooooooooooooooooooooooooooooooooooooooooooooooooooooooooooo

3.3.1 (A. Schinzel). Niech m,n ∈ N. Liczba wymierna mn jest sumą dwóch ułamków prostychwtedy i tylko wtedy, gdy istnieją liczby naturalne a, b takie, że a dzieli n, b dzieli n i a+ bdzieli m. ([S57a] 6).

3.3.2. Niech m i n będą względnie pierwszymi liczbami naturalnymi. Liczba wymierna mnjest sumą dwóch ułamków prostych wtedy i tylko wtedy, gdy istnieją względnie pierwsze liczbynaturalne a, b takie, że a dzieli n, b dzieli n i a+ b dzieli m. ([S57a] 7).

3.3.3. Niech m i n będą względnie pierwszymi liczbami naturalnymi. Liczba wymierna mn jestsumą dwóch ułamków prostych o względnie pierwszych mianownikach wtedy i tylko wtedy, gdym2 − 4n jest liczbą kwadratową. ([S57a] 14).

Page 23: Liczby wymierne

Liczby wymierne. 3. Rozkłady liczb wymiernych na ułamki proste 19

3.3.4. Każda liczba wymierna z przedziału (56 , 1) nie jest sumą dwóch ułamków prostych.(Wynika to z 2.2.2).

3.3.5. Żadna liczba wymierna postaci 37n nie jest sumą dwóch ułamków prostych. ([Mat] 2/56 8).

3.3.6. Pewne pary (a, b), dla których równanie 1x+1y =

ab nie posiada naturalnych rozwiązań.

(3, 7), (3, 13), (3, 19), (3, 31), (3, 37), (3, 43), (3, 49),(4, 5), (4, 13), (4, 17), (4, 25), (4, 29), (4, 37), (4, 41),(5, 7), (5, 11), (5, 13), (5, 17), (5, 22), (5, 23), (5, 31), (5, 33), (5, 37), (5, 41), (5, 43), (5, 47),(6, 7), (6, 13), (6, 19), (6, 31), (6, 37), (6, 43), (6, 49),(7, 8), (7, 9), (7, 11), (7, 15), (7, 16), (7, 17), (7, 19), (7, 22), (7, 23), (7, 25), (7, 29), (7, 31), (7, 32),(7, 37), (7, 43), (7, 44), (7, 46), (7, 47),(8, 9), (8, 11), (8, 13), (8, 17), (8, 19), (8, 25), (8, 27), (8, 29), (8, 33), (8, 37), (8, 41), (8, 43),(9, 10), (9, 11), (9, 13), (9, 19), (9, 22), (9, 23), (9, 25), (9, 29), (9, 31), (9, 37), (9, 38), (9, 41),(9, 43), (9, 46), (9, 47), (9, 49). (Maple).

3.3.7. Pewne pary (a, b), dla których równanie 1x+1y =

ab posiada dokładnie jedno rozwiązanie

naturalne (x, y) takie, że x 6 y.

(1, 1),(3, 4), (3, 5), (3, 11), (3, 17), (3, 23), (3, 25), (3, 29), (3, 41), (3, 47),(4, 7), (4, 9), (4, 11), (4, 19), (4, 23), (4, 31), (4, 43), (4, 47), (4, 49),(5, 6), (5, 8), (5, 9), (5, 14), (5, 16), (5, 19), (5, 21), (5, 26), (5, 27), (5, 29), (5, 32), (5, 34), (5, 38),(5, 39), (5, 46), (5, 49),(6, 11), (6, 17), (6, 23), (6, 25), (6, 29), (6, 41), (6, 47),(7, 10), (7, 13), (7, 18), (7, 26), (7, 27), (7, 33), (7, 34), (7, 38), (7, 39), (7, 41), (7, 45), (7, 50),(8, 21), (8, 23), (8, 31), (8, 35), (8, 47), (8, 49),(9, 16), (9, 17), (9, 20), (9, 26), (9, 28), (9, 32), (9, 34), (9, 35), (9, 44), (9, 50). (Maple).

3.3.8. Pewne pary (a, b), dla których równanie 1x+1y =

ab posiada dokładnie dwa rozwiązania

naturalne (x, y) takie, że x 6 y.

(1, 2), (1, 3), (1, 5), (1, 7), (1, 11), (1, 13), (1, 17), (1, 19), (1, 23), (1, 29), (1, 31), (1, 37), (1, 41), (1, 43),(1, 47),(2, 3), (2, 5), (2, 7), (2, 11), (2, 13), (2, 17), (2, 19), (2, 23), (2, 29), (2, 31), (2, 37), (2, 41), (2, 43), (2, 47),(3, 8), (3, 10), (3, 16), (3, 22), (3, 34), (3, 46),(4, 21), (4, 27), (4, 33),(5, 12), (5, 18), (5, 28),(7, 12), (7, 20), (7, 24), (7, 30), (7, 36), (7, 40),(8, 15), (8, 39), (8, 45). (Maple).

3.3.9. Pewne pary (a, b), dla których równanie 1x+1y =

ab posiada dokładnie trzy rozwiązania

naturalne (x, y) takie, że x 6 y.

(1, 4), (1, 9), (1, 25), (1, 49),

Page 24: Liczby wymierne

20 Liczby wymierne. 3. Rozkłady liczb wymiernych na ułamki proste

(2, 9), (2, 25), (2, 49),(3, 14), (3, 26), (3, 28), (3, 32), (3, 35), (3, 38),(4, 15), (4, 35), (4, 39), (4, 45),(5, 24), (5, 36), (5, 42), (5, 44), (5, 48),(6, 35),(7, 48). (Maple).

3.3.10. Pewne pary (a, b), dla których równanie 1x +1y =

ab posiada dokładnie cztery rozwią-

zania naturalne (x, y) takie, że x 6 y: (1, 8), (1, 27), (2, 27), (3, 20), (3, 44), (3, 50). (Maple).

3.3.11. Pewne pary (a, b), dla których równanie 1x +1y =

ab posiada dokładnie pięć rozwiązań

naturalnych (x, y) takich, że x 6 y.

(1, 6), (1, 10), (1, 14), (1, 15), (1, 16), (1, 21), (1, 22), (1, 26), (1, 33), (1, 34), (1, 35), (1, 38), (1, 39), (1, 46),(2, 15), (2, 21), (2, 33), (2, 35), (2, 39),(3, 40). (Maple).

3.3.12. Pewne pary (a, b), dla których równanie 1x+1y =

ab posiada dokładnie sześć rozwiązań

naturalnych (x, y) takich, że x 6 y: (1, 32), (3, 56), (3, 70), (3, 100), (5, 84). (Maple).

3.3.13. Pary (a, b), dla których równanie 1x +1y =

ab posiada dokładnie 7 rozwiązań natural-

nych (x, y) takich, że x 6 y: (1, 64), (3, 80). Są to wszystkie takie względnie pierwsze parydla a 6 b 6 100. (Maple).

3.3.14. Pary (a, b), dla których równanie 1x +1y =

ab posiada dokładnie 8 rozwiązań natural-

nych (x, y) takich, że x 6 y:

(1, 12), (1, 18), (1, 20), (1, 28), (1, 44), (1, 45),(1, 50), (1, 52), (1, 63), (1, 68), (1, 75), (1, 76), (1, 92), (1, 98), (1, 99),(2, 45), (2, 63), (2, 75), (2, 99).

Są to wszystkie takie względnie pierwsze pary dla a 6 b 6 100. (Maple).

3.3.15. Pary (a, b), dla których równanie 1x +1y =

ab posiada dokładnie 11 rozwiązań na-

turalnych (x, y) takich, że x 6 y: (1, 24), (1, 40), (1, 54), (1, 56), (1, 88). Są to wszystkie takiewzględnie pierwsze pary dla a 6 b 6 100. (Maple).

3.3.16. Pary (a, b), dla których równanie 1x +1y =

ab posiada dokładnie 13 rozwiązań natu-

ralnych (x, y) takich, że x 6 y: (1, 36), (1, 100). Są to wszystkie takie względnie pierwsze parydla a 6 b 6 100. (Maple).

3.3.17. Pary (a, b), dla których równanie 1x+1y =

ab posiada dokładnie 14 rozwiązań natural-

nych (x, y) takich, że x 6 y: (1, 30), (1, 42), (1, 48), (1, 66), (1, 70), (1, 78), (1, 80). Są to wszystkietakie względnie pierwsze pary dla a 6 b 6 100. (Maple).

Page 25: Liczby wymierne

Liczby wymierne. 3. Rozkłady liczb wymiernych na ułamki proste 21

3.3.18. Równanie 1x +1y =

196 posiada dokładnie 17 rozwiązań naturalnych (x, y) takich, że

x 6 y. Równanie 1x +1y =

172 posiada dokładnie 18 takich rozwiązań. (Maple).

3.3.19. Pary (a, b), dla których równanie 1x +1y =

ab posiada dokładnie 23 rozwiązania na-

turalne (x, y) takie, że x 6 y: (1, 60), (1, 84), (1, 90). Są to wszystkie takie względnie pierwszepary dla a 6 b 6 100. Dla takich (a, b) liczba 23 jest maksymalna. (Maple).

3.3.20. Jeśli r jest jedną z liczb 9, 10, 12, 15, 16, 19, 20, 21, 22, to nie ma takich względniepierwszych liczb naturalnych 1 6 a 6 b 6 100, że równanie 1x +

1y =

ab posiada dokładnie r

rozwiązań naturalnych x 6 y. (Maple).

oooooooooooooooooooooooooooooooooooooooooooooooooooooooooooooooooooooo3.4 Równanie 1

x+ 1y= 1z

oooooooooooooooooooooooooooooooooooooooooooooooooooooooooooooooooooooo

3.4.1. 1n =1n+1 +

1n(n+1) ,

1n2 =

1n(n+1) +

1n2(n+1) ,

1n3 =

1n(n2+1) +

1n3(n2+1) .

3.4.2. 12n =1

2n+2i +1

2n+22n−i , dla i = 0, 1, . . . , n.

3.4.3. 1pn =1

pn+pi +1

pn+p2n−i , dla i = 0, 1, . . . , n.

3.4.4. Równania postaci 1n =1x +

1y i ich wszystkie naturalne rozwiązania (x, y), x 6 y, dla

pewnych n. Liczbę rozwiązań oznaczono przez r.(1) n = 2; (3, 6), (4, 4); r = 2.(2) n = 3; (4, 12), (6, 6); r = 2.

(3) n = 4; (5, 20), (6, 12), (8, 8); r = 3.(4) n = 5; (6, 30), (10, 10); r = 2.(5) n = 6; (7, 42), (8, 24), (9, 18), (10, 15), (12, 12); r = 5.(6) n = 7; (8, 56), (14, 14); r = 2.(7) n = 8; (9, 72), (10, 40), (12, 24), (16, 16); r = 4.

(8) n = 9; (10, 90), (12, 36), (18, 18); r = 3.(9) n = 10; (11, 110), (12, 60), (14, 35), (15, 30), (20, 20); r = 5.(10) n = 25; (26, 650), (30, 150), (50, 50); r = 3.(11) n = 50; (51, 2550), (52, 1300), (54, 675), (55, 550), (60, 300), (70, 175), (75, 150),

(100, 100); r = 8.

(12) n = 75; (76, 5700), (78, 1950), (80, 1200), (84, 700), (90, 450), (100, 300), (120, 200),(150, 150); r = 8.

(13) n = 100; (101, 10100), (102, 5100), (104, 2600), (105, 2100), (108, 1350), (110, 1100),(116, 725), (120, 600), (125, 500), (140, 350), (150, 300), (180, 225), (200, 200); r = 13.

(14) n = 200; (201, 40200), (202, 20200), (204, 10200), (205, 8200), (208, 5200), (210, 4200),(216, 2700), (220, 2200), (225, 1800), (232, 1450), (240, 1200), (250, 1000), (264, 825), (280, 700),(300, 600), (325, 520), (360, 450), (400, 400); r = 18. (Maple).

Page 26: Liczby wymierne

22 Liczby wymierne. 3. Rozkłady liczb wymiernych na ułamki proste

3.4.5. Niech n ∈ N i niech x, y ∈ N. Następujące warunki są równoważne.(1) Para (x, y) jest rozwiązaniem równania 1x +

1y =

1n .

(2) Para (x, y) jest postaci (n+ a, n+ b), gdzie a jest podzielnikiem naturalnym liczby n2

oraz b = n2

a .

D. Wynika to z tego, że równość 1x +1y =

1n jest równoważna równości (x− n)(y − n) = n

2. �

3.4.6. Niech n ∈ N i niech τ(n2) oznacza liczbę naturalnych podzielników liczby n2.(1) Liczba naturalnych rozwiązań (x, y) równania 1x +

1y =

1n jest równa τ(n

2).

(2) Liczba naturalnych rozwiązań (x, y) równania 1x +1y =

1n takich, że x 6 y, jest równa

τ(n2)+12 . (Wynika to z 3.4.5).

3.4.7. Dla każdej liczby naturalnej n liczba naturalnych rozwiązań (x, y) równania 1x+1y =

1n

jest nieparzysta. (Wynika to z 3.4.6).

3.4.8. Jeśli p jest liczbą pierwszą, to równanie 1x +1y =

1p ma dokładnie trzy naturalne

rozwiązania, mianowicie: (x, y) = (p+ 1, p+ p2), (2p, 2p), (p+ p2, p+ 1). (Wynika to z 3.4.5).

3.4.9. Niech n ∈ N. Następujące warunki są równoważne.(1) Równanie 1x +

1y =

1n ma dokładnie trzy naturalne rozwiązania (x, y) (przy czym

rozwiązania (x, y) i (y, x), gdzie x 6= y, uważamy za różne).(2) Liczba n jest pierwsza. ([Dlt] 6/83 8, [GaT] 4/73, wynika to z 3.4.6).

3.4.10. Jeśli p > 2 jest liczbą naturalną, to następujące dwa warunki są równoważne:

(1) p jest liczbą pierwszą;

(2) równanie 1x =1p +

1y ma dokładnie jedno rozwiązanie w zbiorze liczb naturalnych.

([OMm] 1997/98).

3.4.11. Niech n ∈ N. Jeśli liczba naturalnych rozwiązań (x, y) równania 1x+1y =

1n jest liczbą

pierwszą, to n jest potęgą liczby pierwszej o wykładniku naturalnym. Implikacja odwrotna niemusi zachodzić. Dla potęgi n = 27 liczba rozwiązań jest równa 15.

D. Niech n = pα11 · · · pαss będzie rozkładem kanonicznym liczby n. Wiemy z 3.4.6(1), że liczbanaturalnych rozwiązań równania 1x +

1y =

1n jest równa τ(n

2), czyli jest równa liczbie r = (2α1 +1) · · · (2αs + 1). Jeśli r jest liczbą pierwszą, to s = 1 i stąd n = pα11 . �

3.4.12. Niech n ∈ N. Jeśli równanie 1x +1y =

1n ma dokładnie 2005 rozwiązań naturalnych

(x, y), to n jest liczbą kwadratową. ([OM] W.Brytania 2005).

Page 27: Liczby wymierne

Liczby wymierne. 3. Rozkłady liczb wymiernych na ułamki proste 23

D. Niech n = pα11 · · · pαss będzie rozkładem kanonicznym liczby n. Wiemy z 3.4.6(1), że liczbanaturalnych rozwiązań równania 1x +

1y =

1n jest równa τ(n

2), czyli jest równa liczbie r = (2α1 +1) · · · (2αs + 1). Załóżmy, że r = 2005 = 5 · 401 (401 jest liczbą pierwszą). Wtedy s = 2, (α1, α2) =(2, 200) lub (200, 2). Zatem n =

(p1p1002

)2lub n =

(p1001 p2

)2, czyli n jest liczbą kwadratową. �

3.4.13.Wszystkie rozwiązania w liczbach naturalnych równania 1x +1y =

1z otrzymujemy ze

wzorów:x = m(m+ n)a, y = n(m+ n)a, z = mna,

gdzie m,n, a ∈ N, nwd(m,n) = 1. ([Mat] 4/49 57, 4/55 71, [S59] 85).

3.4.14. Niech n ∈ N. Nie istnieje żadne rozwiązanie naturalne (x, y) równania 1x +1y =

1n ,

w którym liczby x i y są względnie pierwsze.

3.4.15. Jeśli liczby naturalne x, y, z, których największy wspólny dzielnik jest równy 1, speł-niają równanie 1x +

1y =

1z , to x+ y jest liczbą kwadratową. ([OM] 37/2).

3.4.16. Jeśli parami względnie pierwsze liczby naturalne x, y, z spełniają równanie 1x+1y =

1z ,

to liczby x+ y, x− z, y − z są kwadratowe. ([Tri] 146, [UsaT]).

3.4.17. Niech x, y, z będą liczbami naturalnymi spełniającymi równanie 1x +1y =

1z i niech

d = nwd(x, y, z). Wtedy liczby dxyz i (x− z)d są kwadratowe. ([OM] W.Brytania 1998).

F J. Sandor, On the equation 1x +1y =

1z in integers, [Sand] 57-58.

oooooooooooooooooooooooooooooooooooooooooooooooooooooooooooooooooooooo3.5 Równanie 1

x+ 1y= 2z

oooooooooooooooooooooooooooooooooooooooooooooooooooooooooooooooooooooo

3.5.1. Równania postaci 2n =1x +

1y i ich wszystkie naturalne rozwiązania (x, y), x 6 y, dla

pewnych n. Liczbę rozwiązań oznaczono przez r.

(1) n = 3; (2, 6), (3, 3); r = 2.

(2) n = 5; (3, 15), (5, 5); r = 2.

(3) n = 7; (4, 28), (7, 7); r = 2.

(4) n = 9; (5, 45), (6, 18), (9, 9); r = 3.

(5) n = 11; (6, 66), (11, 11); r = 2.

(6) n = 13; (7, 91), (13, 13); r = 2.

(7) n = 15; (8, 120), (9, 45), (10, 30), (12, 20), (15, 15); r = 5.

(8) n = 17; (9, 153), (17, 17); r = 2.

(9) n = 19; (10, 190), (19, 19); r = 2.

(10) n = 21; (11, 231), (12, 84), (14, 42), (15, 35), (21, 21); r = 5.

(11) n = 23; (12, 276), (23, 23); r = 2.

(12) n = 25; (13, 325), (15, 75), (25, 25); r = 3. (Maple).

Page 28: Liczby wymierne

24 Liczby wymierne. 3. Rozkłady liczb wymiernych na ułamki proste

3.5.2. 24k+1 =

12k+1 +

1(2k+1)(4k+1) . ([Mat] 3/56 7, [S59] 238).

3.5.3. Jeśli p > 2 jest liczbą pierwszą, to ułamek 2p ma dokładnie jedno przedstawienie wpostaci 1a +

1b , gdzie a, b ∈ N, a < b. ([Kurs] 103(1931), [Szn] 7.103).

W. 2ab = p(a+ b). �oooooooooooooooooooooooooooooooooooooooooooooooooooooooooooooooooooooo3.6 Równanie 1

x+ 1y= 1z+ 1t

oooooooooooooooooooooooooooooooooooooooooooooooooooooooooooooooooooooo

3.6.1. 12n +1

2n(2n+1) =12n+1 +

1n(2n+1) . ([Mat] 4/72 251).

3.6.2. Nie ma takich czterech liczb pierwszych a < b < c < d, że 1a+1d =

1b+1c . ([Mat] 4/72 251).

oooooooooooooooooooooooooooooooooooooooooooooooooooooooooooooooooooooo3.7 Sumy trzech ułamków prostychoooooooooooooooooooooooooooooooooooooooooooooooooooooooooooooooooooooo

3.7.1. 1n =1n+2 +

1n(n+1) +

1(n+1)(n+2) . ([S59] 86).

3.7.2. 12k+1 =

13k+2 +

16k+3 +

1(3k+2)(6k+3) . ([MM] 49(1)(1976) s.34).

3.7.3. 14k−1 =

14k+1 +

1(2k+1)(4k−1) +

1(2k+1)(4k−1)(4k+2) . ([S59] 238).

3.7.4. 14k+1 =

14k+3 +

1(2k+1)(4k+3) +

1(2k+1)(4k+1)(4k+3) . ([S59] 238).

3.7.5. 43k−1 =

1k +

13k−1 +

1k(3k−1) . ([Mat] 2/56 10).

3.7.6. 4(4t+3)k−1 =

1(t+1)k +

1(t+1)((4t+3)k−1) +

1(t+1)k((4t+3)k−1) . ([Mat] 2/56 10).

3.7.7. 44k+3 =

1k+2 +

1(k+1)(k+2) +

1(k+1)(4k+3) . ([Mat] 2/56 10).

3.7.8. 47k+3 =

12k+1 +

12(7k+3) +

12(2k+1)(7k+3) . ([Mat] 2/56 11).

3.7.9. 47k−1 =

12k +

12(7k−1) +

12k(7k−1) . ([Mat] 2/56 11).

3.7.10. 54k−1 =

1k +

14k−1 +

1k(4k−1) . ([Mat] 2/56 12).

3.7.11. 55k−1 =

12k +

12k +

12k(10k−1) . ([Mat] 2/56 12).

3.7.12. 55k+3 =

1k+1 +

1(k+1)(5k+3) +

1(k+1)(5k+3) . ([Mat] 2/56 11).

3.7.13. 510k+7 =

12(k+1) +

1(k+1)(10k+7) +

12(k+1)(10k+7) . ([Mat] 2/56 12).

3.7.14. 65k−1 =

1k +

15k−1 +

1k(5k−1) . ([Mat] 2/56 13).

Page 29: Liczby wymierne

Liczby wymierne. 3. Rozkłady liczb wymiernych na ułamki proste 25

3.7.15. 66k−1 =

12k +

12k +

1k(6k−1) . ([Mat] 2/56 13).

3.7.16. Znaleźć wszystkie liczby naturalne x 6 y 6 z spełniające równość 1x +1y +

1z =

12 .

O ([S57a] 34, [MG] 84(500)(2000) s.213, Maple). Jest 10 rozwiązań: (3, 7, 42), (3, 8, 24), (3, 9, 18),(3, 10, 15), (3, 12, 12), (4, 5, 20), (4, 6, 12), (4, 8, 8), (5, 10, 10), (6, 6, 6). �

F W. Sierpiński, O równaniu 1x +1y +

1z =

12 . Posadzkowanie, [S57a] 31-42.

3.7.17. Znaleźć wszystkie liczby naturalne x 6 y 6 z spełniające równość 1x +1y +

1z =

45 .

Odp. (2, 5, 10), (2, 4, 20).

3.7.18. Znaleźć wszystkie liczby naturalne x 6 y 6 z spełniające równość 1x +1y +

1z =

67 .

Odp. (2, 3, 14). ([Mat] 2/56 17).

3.7.19. Pewne równania postaci ab =1x+1y+1z i ich wszystkie naturalne rozwiązania (x, y, z),

x 6 y 6 z. Liczbę rozwiązań oznaczono przez r.

(1) (a, b) = (1, 3); (4, 13, 156), (4, 14, 84), (4, 15, 60), (4, 16, 48), (4, 18, 36), (4, 20, 30), (4, 21, 28), (4, 24, 24),(5, 8, 120), (5, 9, 45), (5, 10, 30), (5, 12, 20), (5, 15, 15), (6, 7, 42), (6, 8, 24), (6, 9, 18), (6, 10, 15), (6, 12, 12), (7, 7, 21),

(8, 8, 12), (9, 9, 9); r = 21.

(2) (a, b) = (2, 3); (2, 7, 42), (2, 8, 24), (2, 9, 18), (2, 10, 15), (2, 12, 12), (3, 4, 12), (3, 6, 6), (4, 4, 6); r = 8.

(3) (a, b) = (1, 4); (5, 21, 420), (5, 22, 220), (5, 24, 120), (5, 25, 100), (5, 28, 70), (5, 30, 60),(5, 36, 45), (5, 40, 40), (6, 13, 156), (6, 14, 84), (6, 15, 60), (6, 16, 48), (6, 18, 36), (6, 20, 30), (6, 21, 28),

(6, 24, 24), (7, 10, 140), (7, 12, 42), (7, 14, 28), (8, 9, 72), (8, 10, 40), (8, 12, 24), (8, 16, 16), (9, 9, 36),

(9, 12, 18), (10, 10, 20), (10, 12, 15), (12, 12, 12); r = 28.

(4) (a, b) = (3, 4); (2, 5, 20), (2, 6, 12), (2, 8, 8), (3, 3, 12), (3, 4, 6), (4, 4, 4); r = 6.

(5) (a, b) = (1, 5); (6, 31, 930), (6, 32, 480), (6, 33, 330), (6, 34, 255), (6, 35, 210), (6, 36, 180),(6, 39, 130), (6, 40, 120), (6, 42, 105), (6, 45, 90), (6, 48, 80), (6, 50, 75), (6, 55, 66), (6, 60, 60), (7, 18, 630),

(7, 20, 140), (7, 21, 105), (7, 30, 42), (7, 35, 35), (8, 14, 280), (8, 15, 120), (8, 16, 80), (8, 20, 40), (8, 24, 30),

(9, 12, 180), (9, 15, 45), (9, 18, 30), (10, 11, 110), (10, 12, 60), (10, 14, 35), (10, 15, 30), (10, 20, 20),

(11, 11, 55), (12, 12, 30), (12, 15, 20), (15, 15, 15); r = 36.

(6) (a, b) = (2, 5); (3, 16, 240), (3, 18, 90), (3, 20, 60), (3, 24, 40), (3, 30, 30), (4, 7, 140),(4, 8, 40), (4, 10, 20), (4, 12, 15), (5, 6, 30), (5, 10, 10), (6, 6, 15); r = 12.

(7) (a, b) = (3, 5); (2, 11, 110), (2, 12, 60), (2, 14, 35), (2, 15, 30), (2, 20, 20), (3, 4, 60),(3, 5, 15), (3, 6, 10), (4, 4, 10), (5, 5, 5); r = 10.

(8) (a, b) = (4, 5); (2, 4, 20), (2, 5, 10); r = 2.

(9) (a, b) = (1, 6); (7, 43, 1806), (7, 44, 924), (7, 45, 630), (7, 46, 483), (7, 48, 336), (7, 49, 294),(7, 51, 238), (7, 54, 189), (7, 56, 168), (7, 60, 140), (7, 63, 126), (7, 70, 105), (7, 78, 91), (7, 84, 84),

(8, 25, 600), (8, 26, 312), (8, 27, 216), (8, 28, 168), (8, 30, 120), (8, 32, 96), (8, 33, 88), (8, 36, 72),

(8, 40, 60), (8, 42, 56), (8, 48, 48), (9, 19, 342), (9, 20, 180), (9, 21, 126), (9, 22, 99), (9, 24, 72),

(9, 27, 54), (9, 30, 45), (9, 36, 36), (10, 16, 240), (10, 18, 90), (10, 20, 60), (10, 24, 40), (10, 30, 30),

(11, 14, 231), (11, 15, 110), (11, 22, 33), (12, 13, 156), (12, 14, 84), (12, 15, 60), (12, 16, 48), (12, 18, 36),

(12, 20, 30), (12, 21, 28), (12, 24, 24), (13, 13, 78), (14, 14, 42), (14, 15, 35), (14, 21, 21), (15, 15, 30),

(15, 20, 20), (16, 16, 24), (18, 18, 18); r = 57.

Page 30: Liczby wymierne

26 Liczby wymierne. 3. Rozkłady liczb wymiernych na ułamki proste

(10) (a, b) = (5, 6); (2, 4, 12), (2, 6, 6), (3, 3, 6), (3, 4, 4); r = 4.

(11) (a, b) = (1, 7); (8, 57, 3192), (8, 58, 1624), (8, 60, 840), (8, 63, 504), (8, 64, 448), (8, 70, 280),(8, 72, 252), (8, 84, 168), (8, 88, 154), (8, 105, 120), (8, 112, 112), (9, 32, 2016), (9, 33, 693), (9, 35, 315),

(9, 36, 252), (9, 42, 126), (9, 45, 105), (9, 56, 72), (9, 63, 63), (10, 24, 840), (10, 25, 350), (10, 28, 140),

(10, 30, 105), (10, 35, 70), (10, 40, 56), (11, 21, 231), (11, 22, 154), (12, 17, 1428), (12, 18, 252), (12, 20, 105),

(12, 21, 84), (12, 24, 56), (12, 28, 42), (14, 15, 210), (14, 16, 112), (14, 18, 63), (14, 21, 42), (14, 28, 28),

(15, 15, 105), (15, 21, 35), (16, 16, 56), (21, 21, 21); r = 42.

(12) (a, b) = (2, 7); (4, 29, 812), (4, 30, 420), (4, 32, 224), (4, 35, 140), (4, 36, 126), (4, 42, 84),(4, 44, 77), (4, 56, 56), (5, 12, 420), (5, 14, 70), (5, 20, 28), (6, 9, 126), (6, 12, 28), (6, 14, 21),

(7, 8, 56), (7, 14, 14), (8, 8, 28); r = 17.

(13) (a, b) = (3, 7); (3, 11, 231), (3, 12, 84), (3, 14, 42), (3, 15, 35), (3, 21, 21), (4, 6, 84),(4, 7, 28), (5, 5, 35), (7, 7, 7); r = 9.

(14) (a, b) = (4, 7); (2, 15, 210), (2, 16, 112), (2, 18, 63), (2, 21, 42), (2, 28, 28), (3, 6, 14),(4, 4, 14); r = 7.

(15) (a, b) = (5, 7); (2, 5, 70), (2, 6, 21), (2, 7, 14), (3, 3, 21); r = 4.

(16) (a, b) = (6, 7); (2, 3, 42); r = 1. (Maple).

3.7.20. Liczba 811 nie jest sumą trzech ułamków prostych. ([Mat] 2/56 18).

3.7.21. Żadna liczba wymierna z przedziału (4142 , 1) nie jest sumą trzech ułamków prostych.(Wynika to z 2.3.4).

3.7.22.Wszystkie pary (a, b) względnie pierwszych liczb naturalnych takich, że 1 6 a 6 b 630 i równanie 1x +

1y +

1z =

ab nie posiada naturalnych rozwiązań.

(8, 11), (8, 17), (9, 11), (9, 19), (10, 11), (12, 13), (12, 25), (12, 29), (13, 14), (14, 17), (14, 19), (14, 29),(15, 16), (15, 17), (15, 19), (15, 23), (16, 17), (16, 23), (17, 19), (17, 23), (18, 19), (18, 23), (18, 29), (19, 23),(19, 26), (19, 29), (20, 21), (20, 23), (20, 27), (20, 29), (21, 22), (21, 23), (21, 26), (21, 29), (22, 23), (22, 25),(22, 27), (23, 25), (23, 26), (23, 29), (24, 25), (24, 29), (25, 26), (25, 27), (25, 29), (26, 27), (26, 29), (27, 28),(27, 29), (28, 29), (29, 30). Jest 51 takich par. (Maple).

3.7.23.Wszystkie pary (a, b) względnie pierwszych liczb naturalnych takich, że 1 6 a 6 b 630 i równanie 1x +

1y +

1z =

ab posiada dokładnie jedno naturalne rozwiązanie x 6 y 6 z.

(6, 7), (6, 13), (8, 9), (9, 29), (10, 13), (10, 23), (11, 13), (11, 17), (11, 23), (12, 17), (12, 19), (13, 16),(13, 17), (13, 29), (14, 15), (14, 23), (16, 19), (16, 21), (16, 25), (17, 18), (17, 22), (18, 25), (19, 20), (19, 21),(19, 22), (19, 25), (21, 25), (22, 29), (23, 24), (23, 27), (23, 28), (25, 28). Są 32 takie pary. (Maple).

3.7.24.Wszystkie pary (a, b) względnie pierwszych liczb naturalnych takich, że 1 6 a 6 b 630 i równanie 1x +

1y +

1z =

ab posiada dokładnie 2 naturalne rozwiązania x 6 y 6 z.

(4, 5), (6, 19), (7, 8), (7, 11), (7, 29), (8, 19), (9, 10), (9, 13), (9, 23), (11, 14), (13, 19), (13, 23), (13, 27),(16, 29), (17, 20), (17, 21), (17, 25), (17, 26), (17, 27), (17, 29). Jest 20 takich par. (Maple).

Page 31: Liczby wymierne

Liczby wymierne. 3. Rozkłady liczb wymiernych na ułamki proste 27

3.7.25.Wszystkie pary (a, b) względnie pierwszych liczb naturalnych takich, że 1 6 a 6 b 630 i równanie 1x +

1y +

1z =

ab posiada dokładnie 3 naturalne rozwiązania x 6 y 6 z.

(1, 1), (5, 11), (7, 9), (7, 23), (8, 13), (8, 25), (10, 17), (11, 12), (11, 16), (11, 19), (11, 25), (11, 26), (13, 15),(15, 22), (19, 24), (19, 27), (19, 28). Jest 17 takich par. (Maple).

3.7.26.Wszystkie pary (a, b) względnie pierwszych liczb naturalnych takich, że 1 6 a 6 b 630 i równanie 1x +

1y +

1z =

ab posiada dokładnie 4 naturalne rozwiązania x 6 y 6 z.

(4, 13), (4, 17), (5, 6), (5, 7), (7, 17), (8, 29), (9, 14), (9, 22), (10, 21), (11, 15), (11, 29), (13, 18), (14, 25),(15, 26), (16, 27), (17, 28), (23, 30). Jest 17 takich par. (Maple).

3.7.27.Wszystkie pary (a, b) względnie pierwszych liczb naturalnych takich, że 1 6 a 6 b 630 i równanie 1x +

1y +

1z =

ab posiada dokładnie 5 naturalnych rozwiązań x 6 y 6 z.

(7, 10), (7, 16), (7, 22), (11, 27), (13, 21), (13, 22), (13, 28), (17, 24). Jest 8 takich par. (Maple).

3.7.28.Wszystkie pary (a, b) względnie pierwszych liczb naturalnych takich, że 1 6 a 6 b 630 i równanie 1x +

1y +

1z =

ab posiada dokładnie 6 naturalnych rozwiązań x 6 y 6 z.

(3, 4), (4, 9), (5, 13), (5, 17), (7, 19), (8, 27), (11, 18), (13, 20). Jest 8 takich par. (Maple).

3.7.29.Wszystkie pary (a, b) względnie pierwszych liczb naturalnych takich, że 1 6 a 6 b 630 i równanie 1x +

1y +

1z =

ab posiada dokładnie 7 naturalnych rozwiązań x 6 y 6 z.

(4, 7), (4, 29), (5, 8), (6, 11), (6, 25), (8, 23), (9, 16), (9, 17), (10, 19), (10, 29), (12, 23), (15, 29), (19, 30).Jest 13 takich par. (Maple).

3.7.30.Wszystkie pary (a, b) względnie pierwszych liczb naturalnych takich, że 1 6 a 6 b 630 i równanie 1x +

1y +

1z =

ab posiada dokładnie 8 naturalnych rozwiązań x 6 y 6 z.

(2, 3), (6, 17), (7, 13), (10, 27), (11, 24). Jest 5 takich par. (Maple).

3.7.31.Wszystkie pary (a, b) względnie pierwszych liczb naturalnych takich, że 1 6 a 6 b 630 i równanie 1x +

1y +

1z =

ab posiada dokładnie 9 naturalnych rozwiązań x 6 y 6 z.

(3, 7), (4, 11), (5, 23), (6, 29), (7, 15), (9, 20), (9, 25), (17, 30). Jest 8 takich par. (Maple).

3.7.32.Wszystkie pary (a, b) względnie pierwszych liczb naturalnych takich, że 1 6 a 6 b 630 i równanie 1x +

1y +

1z =

ab posiada dokładnie 10 naturalnych rozwiązań x 6 y 6 z.

(1, 2), (3, 5), (4, 25), (8, 21), (9, 28), (13, 25), (13, 30). Jest 7 takich par. (Maple).

Page 32: Liczby wymierne

28 Liczby wymierne 3. Rozkłady liczb wymiernych na ułamki proste

3.7.33. Liczby r naturalnych rozwiązań x 6 y 6 z równania 1x +1y +

1z =

ab dla pewnych

względnie pierwszych liczb naturalnych 1 6 a 6 b 6 30.(1) r = 128, (a, b) = (1, 16).(2) r = 136, (a, b) = (1, 14).(3) r = 156, (a, b) = (1, 25).(4) r = 157, (a, b) = (1, 22).(5) r = 160, (a, b) = (1, 12).(6) r = 174, (a, b) = (1, 26).(7) r = 196, (a, b) = (1, 15).(8) r = 211, (a, b) = (1, 18).(9) r = 230, (a, b) = (1, 27).(10) r = 245, (a, b) = (1, 21).(11) r = 292, (a, b) = (1, 20).(12) r = 340, (a, b) = (1, 28).(13) r = 366, (a, b) = (1, 24).(14) r = 497, (a, b) = (1, 30). (Maple).

3.7.34. Znaleźć wszystkie naturalne rozwiązania równania 1x +1y +

1z =

1t . ([S59] 87).

3.7.35. Jeśli n > 3 jest nieparzystą liczbą naturalną niepodzielną przez 3, to istnieją różneliczby nieparzyste a, b, c takie, że 3n =

1a +

1b +

1c . ([Mon] 2000(1) s.62).

F T. R. Hagedorn, A proof of a conjecture on Egyptian fractions, [Mon] 1007(1)(2000) 62-63.oooooooooooooooooooooooooooooooooooooooooooooooooooooooooooooooooooooo3.8 Zadania różneoooooooooooooooooooooooooooooooooooooooooooooooooooooooooooooooooooooo

3.8.1.(1− 122

) (1− 132

) (1− 142

)· · ·(1− 1

n2

)= n+12n . ([GeG] 15).

3.8.2. Jeśli(1 + 1a

) (1 + 1b

) (1 + 1c

)= 2, gdzie a 6 b 6 c są liczbami naturalnymi, to

(a, b, c) = (3, 4, 5), (3, 3, 8), (2, 6, 7), (2, 5, 9) lub (2, 4, 15). ([OM] W.Brytania 1995).

3.8.3. Istnieją cztery liczby całkowite a, b, c, d takie, że |a|, |b|, |c|, |d| > 106 oraz 1a+1b+1c+1d =

1abcd . ([OM] Rosja 2006).

3.8.4. Niech d > 0 i niech Sn(d) ={s = 1

x1+ 1x2+ · · ·+ 1

xn; x1, . . . , xn ∈ N, s < d

}, dla

n ∈ N. Zbiór Sn(d) posiada maksymalny element. ([OM] Węgry-Izrael 2003).

F A. Bonning i inni, Writing a rational number in Egyptian forms, [MG] 86(2002), 432-436.R. Cohen, Egyptian fraction expansions, [MM] 46(2)(1973) 76-80.R. K. Guy, Egyptian fractions, [Gy04] 252-262.R. K. Guy, A reciprocal diophantine equations, [Gy04] 309.W. Narkiewicz, Egyptian fractions, [Nar86] 42-58.J. Sandor, On the diophantine equation 1x1 +

1x2+ · · ·+ 1

xn= 1xn+1, [Sand] 65-67.

W. Sierpiński, O rozkładach liczb wymiernych. . . , [Mat] 2/56 8-19, [Mat] 3/56 1-8.H. A. Simmons, On a cyclo-symmetric Diophantine equation, [Mon] 36(3)(1929) 148-155.

Page 33: Liczby wymierne

����������������������������������������������������

4 Odwrotności wyrazów pewnych ciągów����������������������������������������������������

oooooooooooooooooooooooooooooooooooooooooooooooooooooooooooooooooooooo4.1 Niecałkowitość sumy odwrotności wyrazów ciąguoooooooooooooooooooooooooooooooooooooooooooooooooooooooooooooooooooooo

4.1.1. Liczba 11 +12 +

13 + · · ·+

1n , dla n > 2, nigdy nie jest całkowita. ([Mon] 41(1)(1934) E46).

4.1.2. Liczba 1n +1n+1 + · · ·+

1n+k , gdzie n, k ∈ N, nigdy nie jest całkowita.

([S50] 12, [Wino] 24, [Br80] 28, [DoC] 139).

4.1.3. Liczba ± 1n ±1n+1 ± · · · ±

1n+k , gdzie n, k ∈ N, nigdy nie jest całkowita. ([Maza] s.12).

4.1.4. Liczba 13 +15 + · · ·+

12n+1 nigdy nie jest całkowita. ([Wino] 24,[DoC] 140).

4.1.5. Jeśli a ∈ N, to liczba 11+a+

11+2a+· · ·+

11+na nigdy nie jest całkowita. ([IMO] Longlist 1979).

4.1.6. Jeśli rosnący ciąg a1, a2, . . . , an jest arytmetyczny o wyrazach naturalnych i n > 2, toliczba 1a1 +

1a2+ · · ·+ 1

annie jest całkowita. ([Mon] 96(8)(1989) 741-742).

4.1.7. Niech m > n > 2 będą liczbami naturalnymi. Niech {a1, . . . , as} będzie zbiorem wszyst-kich liczb naturalnych mniejszych lub równych m i względnie pierwszych z n. Wtedy liczba1a1+ 1a2 + · · ·+

1asnie jest całkowita. ([Mon] 79(10)(1972) E2327).

4.1.8. Niech a1, . . . , an, gdzie n > 2, będą parami różnymi liczbami naturalnymi. Załóżmy,że dokładnie jedna z nich jest liczbą parzystą, a pozostałe są liczbami nieparzystymi. Wtedyliczba 1a1 +

1a2+ · · ·+ 1as nie jest całkowita. ([MM] 56(5)(1983)).

D. Przypuśćmy, że 1a1 +1a2+ · · ·+ 1

as= k jest liczbą całkowitą. Oznaczmy: m = a1a2 · · · an oraz

mi = maidla i = 1, . . . , n. Wtedy m1, . . . ,mn są liczbami całkowitymi i mamy równość m1 + m2 +

· · ·+mn = km. Prawa strona tej równości jest parzysta. Natomiast lewa strona jest nieparzysta. �

oooooooooooooooooooooooooooooooooooooooooooooooooooooooooooooooooooooo4.2 Odwrotności wyrazów ciągu arytmetycznegooooooooooooooooooooooooooooooooooooooooooooooooooooooooooooooooooooooo

4.2.1. Jeśli (an) jest ciągiem arytmetycznym o niezerowych wyrazach, to

1a1a2+ 1a2a3+ · · ·+ 1

anan+1= na1an+1

. ([Kw] 2/71 38).

29

Page 34: Liczby wymierne

30 Liczby wymierne. 4. Odwrotności wyrazów pewnych ciągów

4.2.2. Niech (an) będzie ciągiem arytmetycznym o wyrazach dodatnich i różnicy r > 0. Wtedy

1r ln

an+1a1< 1a1+ 1a2 + · · ·+

1an

6 1a1+ 1r ln

ana1. ([Bedn] 186-7).

4.2.3. Jeśli cn = 11 +13 +

15 + · · ·+

12n−1 dla n ∈ N, to

12c2n + (cn − c1)2 + (cn − c2)2 + · · ·+ (cn − cn−1)2 = n2 . ([Kw] 7/81 25).

4.2.4. Jeśli 2r jest największą potęgą dwójki dzielącą liczbę naturalną n, to 22r jest największąpotęgą dwójki dzielącą licznik liczby wymiernej 11 +

13 +

15 + · · ·+

12n−1 . ([Mon] 67(9)(1960) 924-925).

oooooooooooooooooooooooooooooooooooooooooooooooooooooooooooooooooooooo

4.3 Odwrotności kolejnych liczb naturalnychoooooooooooooooooooooooooooooooooooooooooooooooooooooooooooooooooooooo

4.3.1. Rozwinięcie dziesiętne liczby 1n +1n+1 +

1n+2 nie ma czystego okresu. ([Mat] 4/50 64).

4.3.2. 1kn +1kn+1 +

1kn+2 + · · ·+

1kn+n−1 > n

(n

√k+1k − 1

), dla n, k ∈ N. ([OM] Izrael 1995).

4.3.3. 1n+1 +1n+2 + · · ·+

12n >

1324 , dla n > 2. ([BaL] 463, [Szn] 1.70, [G-if] 102).

4.3.4. 1n+1 +1n+2 + · · ·+

12n >

35 , dla n > 3. ([G-if] 204).

D. 13 +14 =

1424 >

1324 ,

14 +

15 +

16 =

3760 >

35 . Krok indukcyjny:

1k+2 +

1k+3 + · · ·+

12k+2 =

(1k+1 +

1k+2 + · · ·+

12k

)+(12k+1 +

12k+2 −

1k+1

)=(1k+1 +

1k+2 + · · ·+

12k

)+ 1(2k+1)(2k+2)

> 1k+1 +

1k+2 + · · ·+

12k

> 35 . �

4.3.5. 1n +1n+1 + · · ·+

1n2 > 1. ([Kurs] 125(1938)).

4.3.6. 1 < 1n+1 +

1n+2 + · · ·+

13n+1 < 2. ([BoL] 51 s.55, [Szn] 1.70).

Page 35: Liczby wymierne

Liczby wymierne. 4. Odwrotności wyrazów pewnych ciągów 31

4.3.7. Jeśli n jest liczbą naturalną, to przez h(n) oznaczamy sumę 1 + 12 +13 + · · · +

1n .

Przyjmujemy dodatkowo, że h(0) = 0.

(1) n(n+ 1)1n < n+ h(n), dla n > 2. ([Putn] 1975).

(2) (n− 1)n−1n−1 < n− h(n), dla n > 3. ([Putn] 1975).

(3) n(n√n+ 1− 1

)< h(n) < n

(1− 1

n√n

)+ 1. ([Kw] 8/77 45).

(4) n2 < h (2

n − 1) < n, dla n > 2. ([BoL] 40 s.55).(5) Jeśli h(n) > m, to h(3n) > m+ 1. ([OM] Litwa 1993).(6) [h ([en])] = n, gdzie [x] oznacza część całkowitą liczby x. ([Bedn] 188).(7) n+ h(1) + h(2) + · · ·+ h(n− 1) = nh(n). ([OM] Kanada, [Kw] 7/89 76).

(8)n−1∑k=0(h(n)− h(k))2 = 2n− h(n). ([Kw] 9/90 25).

(9) 3h(1) + 5h(2) + 7h(3) + · · ·+ (2n+ 1)h(n) = (n+ 1)2h(n)− 12n(n+ 1). ([TT] 2003).

(10) h(n)2 > 2(12h(2) +

13h(3) + · · ·+

1nh(n)

), dla n > 2. ([OM] Mołdawia 1998).

4.3.8.∑ 1k > ϕ(m)

m

n∑k=1

1k , gdzie sumowanie po lewej stronie przebiega wszystkie liczby natu-

ralne k takie, że 1 6 k 6 n i nwd(k,m) = 1. Przez ϕ(m) oznaczamy liczbę wszystkich liczb na-turalnych mniejszych lub równych m i względnie pierwszych z m. ([IMO] 1978, [KoM] 2000(5) A240).

4.3.9. Czy z ciągu 1, 12 ,13 ,14 , . . . można wybrać ciąg składający się z

(1) 5,(2) n,(3) nieskończenie wielu wyrazów, w którym każdy wyraz, począwszy od trzeciego, jest

różnicą dwóch wyrazów poprzednich ? ([Kw] 5/95, M1486).R. (2). Niech n ∈ N. Rozpatrzmy n-wyrazowy ciąg Fibonacciego: 1, 2, 3, 5, 8, . . . , un−1, un. Za-

piszmy wszystkie wyrazy w odwrotnej kolejności i podzielmy je przez ich najmniejszą wspólną wielo-krotność. Otrzymany ciąg spełnia warunki zadania.(1). Dla przykładu dla n = 5 otrzymujemy: nww(1, 2, 3, 5, 8) = 120, 8120 =

115 ,

5120 =

124 ,

3120 =

140 ,

2120 =

160 ,

1120 =

1120 . Mamy zatem pięciowyrazowy ciąg spełniający warunki zadania:

115,124,140,160,1120.

Sprawdzamy: 115 −124 =

140 ,

124 −

140 =

160 ,

140 −

160 =

1120 .

(3). Wykażemy teraz, że nie istnieje żaden nieskończony ciąg spełniający warunki zadania. Przy-puśćmy, że taki ciąg istnieje. Oznaczmy go przez (an = 1

bn). Wówczas:

a3 = a1 − a2 =1b1− 1b2=b2 − b1b1b2

=m3b1b2,

gdzie m3 < b1b2. Stąd dalej otrzymujemy:

a4 = a2 − a3 =1b2− m3b1b2=b1 −m3b1b2

=m4b1b2,

gdzie m4 < b1b2. Na mocy indukcji stwierdzamy, że każdy wyraz ciągu (an) jest postaci mnb1b2 , gdzie mnjest liczbą naturalną mniejszą od b1b2. Takich wyrazów jest, wbrew przypuszczeniu, tylko skończeniewiele. �

Page 36: Liczby wymierne

32 Liczby wymierne. 4. Odwrotności wyrazów pewnych ciągów

4.3.10. Czy w ciągu 11 ,12 ,13 , . . . istnieje 5 wyrazów tworzących ciąg arytmetyczny? Jaki jest

najdłuższy podciąg arytmetyczny w tym ciągu? ([TT] 1982).

O. Istnieje taki ciąg arytmetyczny dowolnej skończonej długości. Niech m = nww(1, 2, 3, . . . , n).Wtedy 1m ,

2m ,3m , . . . ,

nm jest takim ciągiem arytmetycznym długości n. �

4.3.11. Niech n ∈ N i niech 11+12+13+· · ·+

1n =

anbn, gdzie liczby an i bn są względnie pierwsze.

Istnieje tylko 19 liczb naturalnych n takich, że 5 - bn. Są to: 1, 2, 3, 4, 20, 21, 22, 23, 24, 100,101, 102, 103, 104, 120, 121, 122, 123, 124. ([Putn] 1997).

4.3.12. Na tablicy wypisano liczby 11 ,12 ,13 , . . . ,

11998 ,

11999 . Wybierzmy dwie z nich, powiedzmy

a i b, i zamiast nich napiszmy liczbę a+ b+ab. Powtarzamy to tak długo, aż otrzymamy tylkojedną liczbę na tablicy. Czy jest możliwe by tą jedyną liczbą była liczba 2000 ?([OM] Słowenia 1999).

R. Oznaczmy: a⊕ b = a+ b+ab. Wtedy a⊕ b = b⊕a oraz (a⊕ b)⊕ c = a⊕ (b⊕ c). Można zatemwybierać dwie liczby w dowolnej kolejności i zawsze pozostanie na końcu ta sama liczba. Ponieważ1⊕ 12 = 2, 2⊕

13 = 3, . . . , 1998⊕

11999 = 1999, więc końcową liczbą jest 1999. �

4.3.13. Zbiór wszystkich liczb wymiernych postaci 1n+1 +

1n+2 + · · · +

1n+m , gdzie n,m ∈ N,

jest gęsty w zbiorze [0,∞). ([Cmj] 28(5)(1997) s.409).

F R. P. Boas Jr., J. W. Wrench Jr., Partial sums of the harmonic series, [Mon] 78(8)(1971) 864-870.

oooooooooooooooooooooooooooooooooooooooooooooooooooooooooooooooooooooo4.4 Naprzemienne sumy ułamków prostychoooooooooooooooooooooooooooooooooooooooooooooooooooooooooooooooooooooo

4.4.1. 1− 12 +13 −

14 + · · · −

12n =

1n+1 +

1n+2 + · · ·+

12n . ([BoL] 10 s.52).

4.4.2. Jeśli n jest parzyste, to 1− 12 +13 −

14 + · · ·−

1n = 2

(1n+2 +

1n+4 + · · ·+

12n

). ([Br80] 29).

4.4.3. Jeśli ab = 1 −12 +

13 −

14 + · · · −

11318 +

11319 , gdzie a, b ∈ N, to a jest podzielne przez

1979. ([Kw] 8/80 34, [Br83] 10).

4.4.4. Niech p będzie liczbą pierwszą postaci 3k+2. Niech a, b będą takimi liczbami natural-nymi, że ab = 1−

12 +

13 −

14 + · · · −

12k +

12k+1 . Wtedy a jest podzielne przez p. ([Br83] 33).

4.4.5. Jeśli p > 2 jest liczbą pierwszą, to 1− 12 +13 − · · · −

1p−1 =

a(p−1)! , gdzie a jest resztą z

dzielenia liczby (2− 2p)/p przez p. ([Wino] 72).

Page 37: Liczby wymierne

Liczby wymierne. 4. Odwrotności wyrazów pewnych ciągów 33

4.4.6. Każda liczba wymierna pq ∈ (0, 1] ma jednoznaczne przedstawienie postaci

pq =

1a1− 1a1a2+ 1a1a2a3

− · · ·+ (−1)k−1 1a1a2···ak ,

gdzie a1, . . . , ak ∈ N, 1 6 a1 < a2 < · · · < ak−1 < ak − 1. Na przykład: 38 =12 −

12·4 ,

1142 =

13 −

13·4 +

13·4·7 . ([Mon] 1999(3) 243).

oooooooooooooooooooooooooooooooooooooooooooooooooooooooooooooooooooooo4.5 Odwrotności liczb pierwszychoooooooooooooooooooooooooooooooooooooooooooooooooooooooooooooooooooooo

4.5.1. Liczba 1 nie jest sumą odwrotności parami różnych liczb pierwszych. ([Szu87] 38).

4.5.2. Nie istnieją parami różne liczby pierwsze p1, . . . , pn takie, że 1p1 + · · ·+1pnjest liczbą

całkowitą. ([MM] 56(5)(1983) Q687, [Mat] 1/1996 s.45).

D. Niech p1 < p2 < · · · < pn, gdzie n > 2, będą takimi liczbami pierwszymi, że 1p1 + · · ·+1pn= k

jest liczbą całkowitą. Oznaczmy: m = p1p2 · · · pn oraz mi = mpi dla i = 1, . . . , n. Wtedy m1, . . . ,mnsą liczbami całkowitymi i mamy równość m1 +m2 + · · · +mn = km. Prawa strona tej równości jestpodzielna przez p1. Natomiast lewa strona nie jest podzielna przez p1. �

4.5.3. Znaleźć wszystkie liczby pierwsze p 6 q 6 r spełniające równość 1p +1q +

1r =

1n dla

pewnej liczby naturalnej n. Odp. Jest tylko jedno rozwiązanie: p = q = r = 3.([OM] Norwegia 1994).

oooooooooooooooooooooooooooooooooooooooooooooooooooooooooooooooooooooo4.6 Odwrotności liczb potęgowychoooooooooooooooooooooooooooooooooooooooooooooooooooooooooooooooooooooo

4.6.1. Dla każdej liczby naturalnej m przy dostatecznie wielkim naturalnym s równanie 1xm1+

· · ·+ 1xms= 1 ma rozwiązanie w liczbach naturalnych. ([S64] 149).

4.6.2. Niech p, q, r będą liczbami naturalnymi takimi, że nwd(p, r) = nwd(q, r) = 1. Wtedyrównanie 1xp +

1yq =

1zr posiada nieskończenie wiele rozwiązań naturalnych.

D. Z założeń wynika, że liczby pq i r są względnie pierwsze. Istnieje zatem nieskończenie wielepar (m,n) liczb naturalnych takich, że npq − mr = 1. Dla każdej takiej pary (m,n) otrzymujemynaturalne rozwiązanie (x, y, z) = (2nq, 2np, 2m). �

Page 38: Liczby wymierne

34 Liczby wymierne. 4. Odwrotności wyrazów pewnych ciągów

oooooooooooooooooooooooooooooooooooooooooooooooooooooooooooooooooooooo4.7 Odwrotności liczb kwadratowychoooooooooooooooooooooooooooooooooooooooooooooooooooooooooooooooooooooo

4.7.1. Jedynym naturalnym rozwiązaniem równania 1x2 +

1y2 +

1z2 +

1t2 = 1 jest czwórka

(2, 2, 2, 2). ([Mat] 1/58 64, [S64] 147, [B-zm] 19).

4.7.2. Jeśli s > 1, to nie istnieją parami różne liczby naturalne x1, . . . , xn takie, że 1x21+ · · ·+

1x2s= 1. ([S64] 148b, [B-zm] 20).

4.7.3. Jeśli s = 1, 4 lub s > 6, to równanie 1x21+ · · · + 1

x2s= 1 ma rozwiązanie w liczbach

naturalnych. ([S64] 148).

4.7.4. Liczba 112 +122 + · · ·+

1n2 , gdzie n > 1, nigdy nie jest całkowita. ([OM] Litwa 1993).

D. 1 < 112 +

122 + · · ·+

1n2 < 1+

11·2+

12·3+ · · ·+

1(n−1)n = 1+

(1− 12

)+(12 −

13

)+ · · ·+

(1n−1 −

1n

)=

2− 1n < 2. �

4.7.5. 122 +132 + · · ·+

1n2 <

23 . ([Kw] 8/78 47).

4.7.6. 112 +122 +

132 + · · ·+

1n2 < 2−

1n . ([Crux] 1998 s.170).

4.7.7. 112 +122 +

132 + · · ·+

1n2 >

3n2n+1 . ([Crux] z.108).

4.7.8. m(n+1)(n+m+1) <

1(n+1)2 +

1(n+2)2 + · · ·+

1(n+m)2 <

mn(n+m) . ([Siw] 75).

4.7.9. 132 +152 + · · ·+

1(2n+1)2 <

14 . ([BoL] 41 s.55, [Dlt] 1/77 6).

4.7.10. Jeśli (an) jest ciągiem arytmetycznym o wyrazach dodatnich i różnicy r > 0, to1a21+ 1a22+ · · ·+ 1

a2n< 1a21+ 1ra1dla n ∈ N. ([Bedn] 103).

4.7.11. 1122 =1152 +

1202 . ([S54] 81).

4.7.12. 1362 =1452 +

1602 . ([Wm] 7 223, [S64] s.108).

4.7.13. 1602 =1652 +

11562 . ([S54] 81).

4.7.14. Równanie 1x2 +1y2 =

1z2 ma nieskończenie wiele rozwiązań naturalnych. ([Mat] 4/55 73).

Page 39: Liczby wymierne

Liczby wymierne. 4. Odwrotności wyrazów pewnych ciągów 35

D. x = a(b4 − c4), y = 2abc(b2 + c2), z = 2abc(b2 − c2), a, b, c ∈ N, b > c. �

4.7.15. 162 =172 +

1142 +

1212 . ([Wm] 7 223, [S64] s.108).

4.7.16. 12 =122 +

132 +

142 +

162 +

172 +

192 +

1122 +

1142 +

1212 +

1362 +

1452 +

1602 .

([Wm] 7 223, [S64] 148c).

4.7.17. Niech s ∈ N. Równanie 1x20= 1x21+ · · ·+ 1

x2sma nieskończenie wiele rozwiązań natu-

ralnych. ([S64] 150).

4.7.18. Niech s > 1. Równanie 1x20= 1x21+ · · · + 1

x2sma rozwiązanie w liczbach naturalnych

x0 < x1 < · · · < xs. ([S64] 148a).

F J. Sandor, On the equation 1x2 +1y2 =

1z2 in integers, [Sand] 59-61.

W. Sierpiński, Uwagi do pewnego zagadnienia P. Erdosa, [Wm] 7 221-228.oooooooooooooooooooooooooooooooooooooooooooooooooooooooooooooooooooooo4.8 Odwrotności liczb trójkątnychooooooooooooooooooooooooooooooooooooooooooooooooooooooooooooooooooooooLiczbą trójkątną nazywamy każdą liczbę naturalną postaci tn =

n(n+1)2 , gdzie n ∈ N.

4.8.1 (L. Moser). 1tn−1+ 1tn +

1tn+1+ · · ·+ 1

tn2−n−1= 12n . ([Mat] 5/49 47).

4.8.2. Rozważmy równanie 1tx+1+ 1tx+2+ · · ·+ 1

tx+n= 1ty, gdzie n ∈ N.

(1) Jeśli n nie jest liczbą kwadratową (lub n = 1), to równanie to ma nieskończenie wielerozwiązań naturalnych. ([Mat] 1/74 59).

(2) Równanie nie ma naturalnych rozwiązań dla n = 4 i n = 9. ([Mat] 1/76 56).

(3) Jeśli n jest liczbą kwadratową i n > 16, to równanie to ma skończoną (różną od zera)liczbę naturalnych rozwiązań. ([Mat] 1/76 56).

4.8.3. Dla każdej liczby naturalnej n liczba 1n jest sumą odwrotności n parami różnych liczbtrójkątnych. ([S64] 146a).

4.8.4. Dla każdej liczby naturalnej n > 1 istnieją liczby naturalne i < j takie, że

1n =

1i(i+1) +

1(i+1)(i+2) +

1(i+2)(i+3) + · · ·+

1j(j+1) .

([OM] Kanada 1973).

4.8.5. Dla każdej liczby naturalnej s 6= 2 równanie 1x1 +1x2+ · · ·+ 1xs = 1 ma rozwiązanie w

liczbach trójkątnych. ([S64] 144).

4.8.6. Znaleźć wszystkie trójki (m,n, r) ∈ N3 takie, że 1tm +1tn= 1tr .

([Mon] z.4666: 67(10)(1960) 1034-1035, 68(6)(1961) 578).

Page 40: Liczby wymierne

36 Liczby wymierne. 4. Odwrotności wyrazów pewnych ciągów

oooooooooooooooooooooooooooooooooooooooooooooooooooooooooooooooooooooo4.9 Odwrotności sześcianówoooooooooooooooooooooooooooooooooooooooooooooooooooooooooooooooooooooo

4.9.1. Równanie 1x3 +1y3 =

1z3 nie posiada naturalnych rozwiązań.

D. Przypuśćmy, że takie naturalne rozwiązanie (x, y, z) istnieje. Wtedy po pomnożeniu stronamiprzez (xyz)3 otrzymujemy równość (yz)3 + (xz)3 = (xy)3. Dobrze wiadomo jednak, że równaniex3 + y3 = z3 nie ma naturalnych rozwiązań. �

4.9.2. 123 +123 =

122 ,

1653 +

12603 =

15202 ,

1(4·13·61)3 +

1(9·13·61)3 =

1(8·27·13·61)2 .

4.9.3. Równanie 1x3 +1y3 =

1z2 ma nieskończenie wiele rozwiązań naturalnych. (Wynika z 4.6.2).

4.9.4. Jeśli x, y, z są liczbami naturalnymi takimi, że 1x3 +1y3 =

1z2 , to nwd(x, y) > 2.

4.9.5 (Maple). 193 +

1123 +

1723 =

183 . Zauważmy, że nwd(9, 12, 72) = 3 oraz 3 - 8. Inne

przykłady tego typu:

(1) 1953 +

11713 +

15703 =

1903 , nwd(95, 171, 570) = 19 oraz 19 - 90.

(2) 11403 +

11703 +

13403 =

11193 , nwd(140, 170, 340) = 10 oraz 10 - 119.

(3) 11203 +

12523 +

12663 =

11713 , nwd(120, 252, 266) = 2 oraz 2 - 171.

4.9.6. Równanie 1x3 +1y3 +

1z3 =

1t3 ma nieskończenie wiele naturalnych rozwiązań.

D. Z poprzednich przykładów wynika, że co najmniej jedno naturalne rozwiązanie (x, y, z, t)istnieje. Każda więc czwórka postaci (ax, ay, az, at), gdzie a ∈ N, też jest naturalnym rozwiązaniemrozpatrywanego równania. �

4.9.7. 1123 +

1153 +

1203 =

1103 . Równość tę otrzymujemy dzieląc obie strony znanej równości

33 + 43 + 53 = 63 przez 603. Istnieje więc naturalne rozwiązanie równania 1x3 +1y3 +

1z3 =

1t3

takie, że nwd(x, y, z) = 1. Czy istnieje inne tego typu naturalne rozwiązanie? Nie znamodpowiedzi na to pytanie. (15.02.2008).

4.9.8. Dla każdej liczby naturalnej s > 3 równanie 1x30= 1x31+ · · ·+ 1x3s ma nieskończenie wiele

rozwiązań naturalnych. ([S64] 151).

4.9.9. 113 +123 + · · ·+

1n3 <

54 . ([IMO] Longlist 1969, [OM] Grecja 2005).

4.9.10. 133 +143 + · · ·+

1n3 <

112 . ([OM] Irlandia 1990).

Page 41: Liczby wymierne

Liczby wymierne. 4. Odwrotności wyrazów pewnych ciągów 37

4.9.11.(1− 123

) (1− 133

) (1− 143

)· · ·(1− 1

n3

)> 12 . ([IMO] Longlist 1971).

oooooooooooooooooooooooooooooooooooooooooooooooooooooooooooooooooooooo4.10 Graniceoooooooooooooooooooooooooooooooooooooooooooooooooooooooooooooooooooooo

4.10.1. 11 −12 +

13 −

14 +

15 · · · = ln 2. ([Lion]).

4.10.2. 11 −13 +

15 −

17 + · · · =

π4 . (Leibniz, [Kw] 6/97 22).

4.10.3. 112 +122 +

132 + · · · =

π2

6 . ([Mon] 1/1934 s.29).

4.10.4. 112 +132 +

152 + · · · =

π2

8 . ([Mon] 22(1)(1915) 17-19).

4.10.5 (Apery 1978). Liczba 113 +123 +

133 + . . . jest niewymierna. ([Dlt] 7/1980).

4.10.6. Jeśli∞∑n=1

1n3 = s, to

∞∑n=1

1(2n−1)3 =

78s oraz

∞∑n=1

(−1)n−1n3 = 34s. ([MM] 32(3)(1959) z.345).

4.10.7. 114 +124 +

134 + · · · =

π4

90 . ([Dlt] 7/1980, [Mat] 4/88 218, [Ri97] 157).

4.10.8. 116 +126 +

136 + · · · =

π6

945 . ([Dlt] 7/1980).

4.10.9.∞∑n=1

1ns

(1 + 12s + · · ·+

1ns

)= 12

(ζ(s)2 + ζ(2s)

)dla s > 1, gdzie ζ(s) =

∞∑n=1

1ns

(funkcja zeta Riemanna). ([Cmj] 28(2)(1997) 149-150).

4.10.10.(112

)122 +

(112 +

122

)132 +

(112 +

122 +

132

)142 + · · · =

π4

120 . ([Mon] 41(1)(1934) s.29).

4.10.11. 11·2·3 +14·5·6 +

17·8·9 + · · · =

√3π12 −

ln 34 . ([S-kg] 38).

4.10.12. limn→∞

(12n+1 +

12n+3 +

12n+5 + · · ·+

14n−1

)= 12 ln 2. ([Crux] 1992 s.140 z.1640).

4.10.13.∞∑n=1

h(n)n2n =

π2

12 , gdzie h(n) = 1 +12 + · · ·+

1n . ([MM] 60(2)(1987) z.1240).

Page 42: Liczby wymierne

38 Liczby wymierne 4. Odwrotności wyrazów pewnych ciągów

4.10.14. Niech m, k ∈ N. Liczba∞∑n=1

1n(nm+k) jest wymierna wtedy i tylko wtedy, gdy m

dzieli k. ([Mon] 91(10)(1984) z.6424).

4.10.15. Niech (an) będzie ciągiem parami różnych liczb naturalnych, których rozwinięciadziesiętne nie zawierają cyfry 0. Wtedy

∑∞n=1

1an< 29. ([Br80] 89, [B-rs] 236).

4.10.16. Niech (an) będzie ciągiem parami różnych liczb naturalnych, których rozwinięciadziesiętne nie mają na początku cyfry 9. Wtedy szereg

∑∞n=1

1anjest zbieżny.

([MM] 21(2)(1947) s.112).

4.10.17. Niech (an) będzie ciągiem parami różnych liczb naturalnych, których rozwinięciadziesiętne nie zawierają cyfry 9. Wtedy szereg

∑∞n=1

1anjest zbieżny.

([Mon] 1914, [MM] 21(2)(1947) s.112).

4.10.18. Niech (an) będzie ciągiem parami różnych liczb naturalnych, których rozwinięciadziesiętne nie zawierają cyfry 9. Wtedy

∑∞n=1

1an< 28. ([Br83] s.77).

4.10.19 (Hipoteza Erdosa). Niech A będzie takim nieskończonym podzbiorem zbioru liczbnaturalnych, że

∑n∈A

1n = ∞. Wtedy dla każdego k ∈ N zbiór A zawiera pewien podciąg

arytmetyczny długości k. ([Dlt] 9/1999).

U. Nic nie wiadomo nawet dla k = 3 (z artykułu Pawła Strzeleckiego). �

F L. Bibiloni, P. Viader, J. Paradis, On series of Goldbach and Euler, [Mon] 113(3)(2006) 206-220.M. Kline, Euler and infinite series, [MM] 56(5)(1983) 307-314.P. Strzelecki, Sumowanie odwrotności, [Dlt] 9/1999 12-13.A. D. Wadhwa, Some convergent subseries of the harmonic series, [Mon] Oct./1978 661-663.P. Zarzycki, O ciekawych własnościach szeregu harmonicznego, [Dlt] 10/85 1-3.

Page 43: Liczby wymierne

����������������������������������������������������

5 Rozwinięcia dziesiętne liczb wymiernych����������������������������������������������������

oooooooooooooooooooooooooooooooooooooooooooooooooooooooooooooooooooooo5.1 Tablice rozwinięć dziesiętnych pewnych liczb wymiernychoooooooooooooooooooooooooooooooooooooooooooooooooooooooooooooooooooooo

5.1.1. Tabela przedstawia rozwinięcia dziesiętne, wraz z długościami okresów, dla liczb po-staci 1m , gdzie 2 6 m 6 100.

2 0, 5 03 0, (3) 14 0, 25 05 0, 2 06 0, 1(6) 17 0, (142857) 68 0, 125 09 0, (1) 110 0, 1 011 0, (09) 212 0, 08(3) 113 0, (076923) 614 0, 0(714285) 615 0, 0(6) 116 0, 0625 017 0, (0588235294117647) 1618 0, 0(5) 119 0, (052631578947368421) 1820 0, 05 021 0, (047619) 622 0, 0(45) 223 0, (0434782608695652173913) 2224 0, 041(6) 125 0, 04 026 0, 0(384615) 627 0, (037) 328 0, 03(571428) 629 0, (0344827586206896551724137931) 2830 0, 0(3) 131 0, (032258064516129) 1532 0, 03125 033 0, (03) 234 0, 0(2941176470588235) 1635 0, 0(285714) 636 0, 02(7) 137 0, (027) 338 0, 0(263157894736842105) 1839 0, (025641) 640 0, 025 041 0, (02439) 542 0, 0(238095) 643 0, (023255813953488372093) 2144 0, 02(27) 245 0, 0(2) 146 0, 0(2173913043478260869565) 2247 0, (0212765957446808510638297872340425531914893617) 4648 0, 0208(3) 149 0, (020408163265306122448979591836734693877551) 4250 0, 02 0

39

Page 44: Liczby wymierne

40 Liczby wymierne. 5. Rozwinięcia dziesiętne liczb wymiernych

51 0, (0196078431372549) 1652 0, 01(923076) 653 0, (0188679245283) 1354 0, 0(185) 355 0, 0(18) 256 0, 017(857142) 657 0, (017543859649122807) 1858 0, 0(1724137931034482758620689655) 2859 0, (0169491525423728813559322033898305084745762711864406779661) 5860 0, 01(6) 161 0, (016393442622950819672131147540983606557377049180327868852459) 6062 0, 0(161290322580645) 1563 0, (015873) 664 0, 015625 065 0, 0(153846) 666 0, 0(15) 267 0, (014925373134328358208955223880597) 3368 0, 01(4705882352941176) 1669 0, (0144927536231884057971) 2270 0, 0(142857) 671 0, (01408450704225352112676056338028169) 3572 0, 013(8) 173 0, (01369863) 874 0, 0(135) 375 0, 01(3) 176 0, 01(315789473684210526) 1877 0, (012987) 678 0, 0(128205) 679 0, (0126582278481) 1380 0, 0125 081 0, (012345679) 982 0, 0(12195) 583 0, (01204819277108433734939759036144578313253) 4184 0, 01(190476) 685 0, 0(1176470588235294) 1686 0, 0(116279069767441860465) 2187 0, (0114942528735632183908045977) 2888 0, 011(36) 289 0, (01123595505617977528089887640449438202247191) 4490 0, 0(1) 191 0, (010989) 692 0, 01(0869565217391304347826) 2293 0, (010752688172043) 1594 0, 0(1063829787234042553191489361702127659574468085) 4695 0, 0(105263157894736842) 1896 0, 01041(6) 197 0, (010309278350515463917525773195876288659793814432989690721649 96

484536082474226804123711340206185567)98 0, 0(102040816326530612244897959183673469387755) 4299 0, (01) 2100 0, 01 0

5.1.2. Liczby pierwsze p < 1000, dla których ułamki 1p mają rozwinięcia dziesiętne o okresachdługości p− 1.

7 17 19 23 29 47 59 61 97 109113 131 149 167 179 181 193 223 229 233257 263 269 313 337 367 379 383 389 419433 461 487 491 499 503 509 541 571 577593 619 647 659 701 709 727 743 811 821823 857 863 887 937 941 953 971 977 983

Page 45: Liczby wymierne

Liczby wymierne. 5. Rozwinięcia dziesiętne liczb wymiernych 41

oooooooooooooooooooooooooooooooooooooooooooooooooooooooooooooooooooooo5.2 Okresy rozwinięć dziesiętnych liczb wymiernychoooooooooooooooooooooooooooooooooooooooooooooooooooooooooooooooooooooo

5.2.1. Rozwinięcie dziesiętne liczby 17 jest równe 0, (142857). Rozważmy na płaszczyźniepunkty (1, 4), (4, 2), (2, 8), (8, 5), (5, 7) oraz (7, 1). Wszystkie te punkty leżą na elipsie orównaniu 19x2 + 36xy + 41y2 − 333x− 531y + 1638 = 0. ([MM] 60(4)(1987) 245).

5.2.2. Okres rozwinięcia liczby 17 przy podstawie 10 ma długość 6, a przy podstawie 2 madługość 3. Przy podstawie 2 mamy: 17 = 0, 001001001 . . . .

5.2.3. Dla każdej liczby pierwszej p istnieje podstawa q taka, że długość okresu rozwinięciaprzy podstawie q liczby 1p wynosi p− 1.

5.2.4. Niech a, b ∈ N, a < b, nwd(a, b) = 1. Załóżmy, że b = 2n5mc, gdzie 2 - c, 5 - c.Wówczas liczba cyfr po przecinku, stojących przed okresem rozwinięcia dziesiętnego ułamkaab , jest równa liczbie max(n,m). ([JaK] 52).

5.2.5. Okres liczby 1/1997 ma co najwyżej 200 siódemek. ([OM] S.Petersburg 1997).

5.2.6. Niech L(n) oznacza długość najmniejszego okresu rozwinięcia dziesiętnego ułamka 1n .

(1) Jeśli nwd(n, 10) = 1, to L(n) jest najmniejszą liczbą naturalną r taką, że n | 10r− 1.(2) Niech n = 2a5bk, gdzie nwd(k, 10) = 1. Wtedy L(n) = L(k).

(3) Jeśli nwd(a, b) = 1, to L(ab) = nww(L(a), L(b)).

(4) L(n) | ϕ(n).(5) Niech p ∈ P i niech k = vp

(10L(p) − 1

)(tzn. liczba 10L(p)−1 dzieli się pk i nie dzieli

się przez pk+1). Wtedy L(pk+m) = pmL(p) dla wszystkich m ∈ N.(6) L(3m+2) = 3m, L(7m+1) = 6 · 7m. ([Kw] 2000/2 25-29).

5.2.7. Niech p będzie liczbą pierwszą różną od 2 i 5. Niech 0 < n < p i niech d będzienajmniejszą liczbą naturalną taką, że p | 10d − 1.(1) Długość okresu rozwinięcia dziesiętnego liczby np jest równa d.

(2) Jeśli d jest parzyste, to okres rozwinięcia dziesiętnego liczby np można podzielić na

dwie połowy, których suma jest równa 10d/2 − 1. Przykład: 17 = 0, (142857) oraz 142 + 857 =999 = 103 − 1. (N.Sato).

F C. Hsia, Decimal expansion of fractions, [Crux] 1997 285-290.H. Rademacher, O. Toeplitz, Okresy rozwinięć dziesiętnych liczb wymiernych, [RaT] 182-200.K. S. Rao, A note on the recurring period. . . , [Mon] 62(7)(1955) 484-487.J. K. Schiller, A theorem in the decimal representation of rationals, [Mon] 66(9)(1959) 797-798.L. Semionowa, Ułamki okresowe (po rosyjsku), [Kw] 2000/2 25-29.

Page 46: Liczby wymierne

42 Liczby wymierne 5. Rozwinięcia dziesiętne liczb wymiernych

oooooooooooooooooooooooooooooooooooooooooooooooooooooooooooooooooooooo5.3 Różne zadania o rozwinięciach dziesiętnych liczb wymiernychoooooooooooooooooooooooooooooooooooooooooooooooooooooooooooooooooooooo

5.3.1. Nie ma liczby wymiernej ab takiej, że 0 < a < b < 101, w której rozwinięciu dziesiętnymwystępują cyfry 1, 6, 7 (w takiej kolejności). ([OM] W.Brytania 1978).

5.3.2. Nie ma liczby wymiernej ab takiej, że 0 < a < b < 126, w której rozwinięciu dziesiętnymwystępuje blok 143. ([Dlt] 2/2003 z.1016).

5.3.3.W rozwinięciu dziesiętnym liczby 13100 występuje blok cyfr 123456789. ([Dlt] 2/2003).

5.3.4. Rozważmy tablicę nieskończoną T , której każdy wiersz przedstawia rozwinięcie dzie-siętne pewnej liczby wymiernej z przedziału (0, 1). Załóżmy, że zapisaliśmy w ten sposóbwszystkie liczby wymierne przedziału (0, 1), a jeśli któraś ma podwójną reprezentację, uwzglę-dniliśmy obie. Tak więc na przykład pewien wiersz tablicy będzie miał postać 1 1 9 9 9 9 . . . ,a w innym wierszu pojawi się także 1 2 0 0 0 . . . . Będziemy pisali

T =

a11 a12 a13 . . .a21 a22 a23 . . .a31 a32 a33 . . .. . . . . . . . . . . .. . . . . . . . . . . .. . . . . . . . . . . .

,

gdzie każde aij jest jedną z cyfr 0, 1, 2, 3, 4, 5, 6, 7, 8, 9.Rozważmy liczbę rzeczywistą z przedziału (0, 1), której cyfry po przecinku są wzięte z prze-

kątnej tej tablicy, tzn., x = 0, a11a22a33 . . . . Wówczas:

(1) x jest liczbą niewymierną,

(2) x zawiera każdą cyfrę nieskończenie wiele razy. ([Dlt] 5/2004 12-13).

F A. Balfour, Rapid decimal expansions. . . , [MG] 87(509)(2003) 300-305.M. Mendes, Przekątna Cantora, [Dlt] 5/2004 12-13.

Page 47: Liczby wymierne

����������������������������������������������������

6 Przystawanie modulo m dla liczb wymiernych����������������������������������������������������

ooooooooooooooooooooooooooooooooooooooooooooooooooooooooooooooooooooo6.1 DefinicjeoooooooooooooooooooooooooooooooooooooooooooooooooooooooooooooooooooooNiech m ∈ N i x ∈ Q. Mówić będziemy, że x przystaje do 0 modulo m i zapisywać to

będziemy jakox ≡ 0 (mod m),

jeśli x = ab , gdzie a, b ∈ Z, nwd(a, b) = 1 oraz m dzieli a. Przykłady:

23 ≡ 0 (mod 2),

−3419 ≡ 0 (mod 17).

Jeśli x ∈ Z, to przystawanie do 0 w powyższym sensie pokrywa się z przystawaniem do 0 wsensie klasycznym.

6.1.1. Niech m ∈ N, x ∈ Q. Następujące dwa warunki są równoważne.(1) x ≡ 0 (mod m).(2) Dla każdych liczb całkowitych a i b takich, że x = ab zachodzi podzielność: m | a.

Niech m ∈ N i niech x, y ∈ Q. Mówić będziemy, że x przystaje do y modulo m i zapisywaćto będziemy jako

x ≡ y (mod m),

jeśli x− y ≡ 0 (mod m). Przykłady:

13 ≡ 1 (mod 2),

−3419 ≡

619 (mod 20).

Jeśli x, y ∈ Z, to przystawanie takie pokrywa się ze zwykłym przystawaniem modulo m.

6.1.2. Niech x = ab , y =cd , gdzie a, b, c, d ∈ Z.

(1) Jeśli x ≡ y (mod m), to ad ≡ bc (mod m).(2) Implikacja w odwrotnym kierunku nie musi być prawdziwa. Mamy na przykład:

5 · 6 ≡ 6 · 1 (mod 6), 56 6≡

16 (mod 6).

(3) Jeśli założymy dodatkowo, że nwd(b,m) = 1 i nwd(d,m) = 1, to

ad ≡ bc (mod m) ⇐⇒ x ≡ y (mod m).

6.1.3. Niech x, y ∈ Q, m1,m2 ∈ N, nwd(m1,m2) = 1. Wtedy

x ≡ y (mod m1m2) ⇐⇒ x ≡ y (mod m1) i x ≡ y (mod m2).

43

Page 48: Liczby wymierne

44 Liczby wymierne. 6. Przystawanie modulo m dla liczb wymiernych

ooooooooooooooooooooooooooooooooooooooooooooooooooooooooooooooooooooo6.2 Przystawanie i mianownikiooooooooooooooooooooooooooooooooooooooooooooooooooooooooooooooooooooo

6.2.1. Niech x = ab , y =cd , gdzie a, b, c, d ∈ Z, nwd(a, b) = 1, nwd(c, d) = 1. Jeśli x ≡ y

(mod m), to nwd(b,m) = nwd(d,m).

D. Niech x− y = ab −cd =

ad−bcbd =

αβ , gdzie α i β są względnie pierwszymi liczbami całkowitymi.

Wówczasad− bc = αs, bd = βs,

dla pewnej liczby całkowitej s. Ponadto m | α.Niech u = (b,m). Wtedy u | b, u | m i u | α. Z równości ad− bc = αs wynika więc, że u | ad. Ale

(u, a) = 1 (ponieważ (a, b) = 1 i u | b), więc u | d. Zatem u | m i u | d, a zatem u | (d,m). Pokazaliśmywięc, że (b,m) | (d,m). W ten sam sposób wykazujemy, że (d,m) | (b,m). �

6.2.2. Niech x = ab , y =cd , gdzie a, b, c, d ∈ Z, (a, b) = 1, (c, d) = 1. Jeśli x ≡ y (mod m),

to (b,m) > 1 ⇐⇒ (d,m) > 1. (Wynika z 6.2.1).

6.2.3. Załóżmy, że m = pk, k > 1, gdzie p jest liczbą pierwszą. Niech x = ab , y =

cd ,

a, b, c, d ∈ Z, (a, b) = 1, (c, d) = 1. Niech b = pib1, d = pjd1, gdzie i > 0, j > 0, p - b1, p - d1.Jeśli x ≡ y (mod pk), to i = j.

D. Jeśli i = 0, to (b,m) = 1 i wtedy (na mocy 6.2.2) (d,m) = 1 czyli j = 0, tzn. i = j. Możemywięc założyć, że i > 0. Wtedy (na mocy 6.2.2) również j > 0.Przypuśćmy, że i > j. Mamy wówczas:

x− y = ab −cd =

apib1− cpjd1= ad1−p

i−jcb1pib1d1

.

Stąd otrzymujemy równości: ad1 − pi−1cb1 = αs, pib1d1 = βs, gdzie s, α, β ∈ Z, (α, β) = 1 orazm = pk | α. Wtedy p | ad1. Ponieważ p - d1, więc p | a. Mamy zatem sprzeczność: p | (a, b) = 1.Podobną sprzeczność uzyskamy zakładając, że j > i. Zatem i = j. �

Niech m ∈ N i niech x ∈ Q. Mówić będziemy, że liczba x jest m-przedstawialna jeśliistnieją liczby całkowite a i b takie, że x = ab oraz (b,m) = 1.Liczba 16 jest 25-przedstawialna, a liczba

15 nie jest 25-przedstawialna.

6.2.4. Niech x = ab , a, b ∈ Z, (a, b) = 1. Liczba x jest m-przedstawialna wtedy i tylko wtedy,gdy (b,m) = 1.

6.2.5. Niech x, y ∈ Q i załóżmy, że x ≡ y (mod m). Wówczas liczba x jest m-przedstawialnawtedy i tylko wtedy, gdy liczba y jest m-przedstawialna. (Wynika z 6.2.1).

6.2.6. Niech m 6 2, a, b ∈ Z, (a,m) = 1, (b,m) = 1. Istnieje wtedy liczba całkowita c taka,że (m, c) = 1 oraz ab ≡ c (mod m).

D. Ponieważ (b,m) = 1, istnieją liczby całkowite u, v takie, że ub+ vm = 1. Niech c = ua. Wtedy(c,m) = 1 i mamy:

ab − c =

a−bcb =

avmb .

Jest jasne, że avmb ≡ 0 (mod m). �

Page 49: Liczby wymierne

Liczby wymierne. 6. Przystawanie modulo m dla liczb wymiernych 45

6.2.7. Niech m 6 2, q, a, b ∈ Z, (a,m) = 1, (b,m) = 1, (b, q) = 1, (q,m) > 1. Istnieje wtedyliczba całkowita c taka, że (m, c) = 1 oraz aqb ≡

cq (mod m).

D. Ponieważ (b, qm) = 1, istnieją liczby całkowite u, v takie, że ub + vqm = 1. Niech c = ua.Wtedy (c,m) = 1 i jest jasne, że aqb ≡

cq (mod m). �

ooooooooooooooooooooooooooooooooooooooooooooooooooooooooooooooooooooo6.3 Przystawanie i dodawanieooooooooooooooooooooooooooooooooooooooooooooooooooooooooooooooooooooo

6.3.1. Niech x, y ∈ Q, m ∈ N. Jeśli x ≡ 0 (mod m) i y ≡ 0 (mod m), to x+ y ≡ 0 (mod m).

D. Niech x = ab , a, b ∈ Z, (a, b) = 1, m | a, y = c

d , c, d ∈ Z, (c, d) = 1, m | c. Wtedyx+ y = ab +

cd =

ad+bcbd . Niech ad+ bc = αs, bd = βs, s, α, β ∈ Z, (α, β) = 1. Należy wykazać, że

m | α. Wiemy natomiast, że m | αs.Zauważmy, że (m, s) = 1. Przypuśćmy bowiem, że (m, s) > 1. Istnieje wtedy liczba pierwsza p

taka, że p | m i p | s. Wtedy p | a i p | c. Ponadto, p | bd (gdyż bd = βs), czyli p | b lub p | d. Jeśli p | b,to mamy sprzeczność: p | (a, b) = 1. Jeśli p | d, to mamy sprzeczność: p | (c, d) = 1. Zatem istotnie(m, s) = 1.Mamy teraz: m | αs i (m, s) = 1, czyli m | α. �

6.3.2. Niech x, y, x′, y′ ∈ Q, m ∈ N.Jeśli x ≡ x′ (mod m) i y ≡ y′ (mod m), to x+ y ≡ x′ + y′ (mod m).

D. Niech x ≡ x′ (mod m) i y ≡ y′ (mod m). Wtedy x − x′ ≡ 0 (mod m) i y − y′ ≡ 0 (mod m)więc, na mocy 6.3.1,

(x+ y)− (x′ + y′) = (x− x′) + (y − y′) ≡ 0 (mod m),

czyli x+ y ≡ x′ + y′ (mod m). �

ooooooooooooooooooooooooooooooooooooooooooooooooooooooooooooooooooooo6.4 Przystawanie jako relacja równoważnościooooooooooooooooooooooooooooooooooooooooooooooooooooooooooooooooooooo

6.4.1. Niech m ∈ N. Wtedy:(1) x ≡ x (mod m), dla x ∈ Q;(2) x ≡ y (mod m) =⇒ y ≡ x (mod m), dla x, y ∈ Q;(3) x ≡ y (mod m) i y ≡ z (mod m) =⇒ x ≡ z (mod m), dla x, y, z ∈ Q.

D. Własności (1) i (2) są oczywiste. Wykażemy (3). Niech x ≡ y (mod m) i y ≡ z (mod m).Wtedy x− y ≡ 0 (mod m) i y − z ≡ 0 (mod m). Z 6.3.1 otrzymujemy wtedy, że

x− z = (x− y) + (y − z) ≡ 0 (mod m),

tzn. x ≡ z (mod m). �

Page 50: Liczby wymierne

46 Liczby wymierne. 6. Przystawanie modulo m dla liczb wymiernych

ooooooooooooooooooooooooooooooooooooooooooooooooooooooooooooooooooooo

6.5 Przystawanie i mnożenieooooooooooooooooooooooooooooooooooooooooooooooooooooooooooooooooooooo

6.5.1. Jeśli x ≡ x′ (mod m) i y ≡ y′ (mod m), to nie musi być, że xy ≡ x′y′ (mod m).Przykład:

32 ≡ 0 (mod 3),

23 ≡

23 (mod 3), 1 =

32 ·23 6≡ 0 (mod 3).

Przypomnijmy, że liczba wymierna x jest m-przedstawialna, jeśli istnieją liczby całkowitea i b takie, że x = ab oraz (b,m) = 1.

6.5.2. Niech x, u ∈ Q i niech x ≡ 0 (mod m). Jeśli u jest m-przedstawialne, to xu ≡ 0(mod m).

D. Niech x = ab , a, b ∈ Z, (a, b) = 1, m | a, u = cd , (c, d) = 1, (d,m) = 1. Wtedy ux =acbd . Niech

ac = αs, bd = βs, s, α, β ∈ Z, (α, β) = 1.

Należy wykazać, że m | α. Wiemy natomiast, że m | αs. Wystarczy zatem wykazać, że (m, s) = 1. Tojest oczywiste. �

6.5.3. Niech x, y, u ∈ Q i niech x ≡ y (mod m). Jeśli u jest m-przedstawialne, to xu ≡ yu(mod m). (Wynika z 6.5.2).

6.5.4. Niech x, y, u, v ∈ Q. Niech x ≡ y (mod m) i u ≡ v (mod m). Jeśli liczby x i u (lubrównoważnie y i v) są m-przedstawialne, to xu ≡ yv (mod m).

D. Z 6.2.5 wiemy, że wszystkie liczby x, y, u, v są m-przedstawialne. Korzystając z wcześniejszychfaktów mamy kolejno: x−y ≡ 0 (mod m), (x−y)u ≡ 0 (mod m), (u−v)y ≡ 0 (mod m) i po dodaniu:

xu− vy = (x− y)u+ (u− v)y ≡ 0 (mod m),

czyli xu ≡ yv (mod m). �

ooooooooooooooooooooooooooooooooooooooooooooooooooooooooooooooooooooo

6.6 Przystawanie modulo 2ooooooooooooooooooooooooooooooooooooooooooooooooooooooooooooooooooooo

6.6.1. Jeśli a i b 6= 0 są całkowitymi liczbami nieparzystymi, to ab ≡ 1 (mod 2).

6.6.2. Jeśli a jest parzystą liczbą całkowitą i b 6= 0 jest nieparzystą liczbą całkowitą, to ab ≡ 0(mod 2).

6.6.3. Niech a, b będą nieparzystymi liczbami całkowitymi i niech n ∈ N. Istnieje wtedy nie-parzysta liczba całkowita c taka, że a2nb ≡

c2n (mod 2).

Page 51: Liczby wymierne

Liczby wymierne. 6. Przystawanie modulo m dla liczb wymiernych 47

D. Liczby b i 2n+1 są względnie pierwsze. Istnieją więc liczby całkowite u, v takie, że ub+v2n+1 = 1.Niech c = ua. Wtedy c jest liczbą nieparzystą i mamy:

a2nb −

c2n =

a−bc2nb =

a−uab2nb =

2vab .

Jest jasne, że 2vab ≡ 0 (mod 2). �

6.6.4. Niech a będzie nieparzystą liczbą całkowitą i niech n ∈ N. Istnieje wtedy nieparzystaliczba całkowita b taka, że a2n ≡

b2n (mod 2) oraz 1 6 b 6 2n+1 − 1.

D. Niech b będzie resztą z dzielenia liczby a przez 2n+1. Wtedy a = 2n+1+ b dla pewnego u ∈ Z,liczba b jest nieparzysta, 1 6 b 6 2n+1 − 1 oraz

a2n −

b2n =

a−b2n =

2n+1u2n =

2u1 ≡ 0 (mod 2),

czyli a2n ≡b2n (mod 2). �

6.6.5. Niech a, b będą nieparzystymi liczbami całkowitymi takimi, że 1 6 a, b 6 2n+1 − 1.Jeśli a2n ≡

b2n (mod 2), to a = b.

D. Ponieważ a−b2n ≡ 0 (mod 2), więc 2n+1 | a − b. Przypuśćmy, że a 6= b. Wtedy |a − b| > 2n+1,

gdy tymczasem |a− b| < 2n+1. �

6.6.6. Każda liczba wymierna przystaje modulo 2 do dokładnie jednej z liczb: 0, 1 lub 2k+12n ,gdzie n > 1, k = 0, 1, 2, . . . , 2n − 1. (Dla n = 1 mamy liczby 12 i

32 , a dla n = 2 liczby

14 ,34 ,54

i 74).

D. Wynika to z poprzednich faktów. �

ooooooooooooooooooooooooooooooooooooooooooooooooooooooooooooooooooooo

6.7 Przystawanie modulo pk

oooooooooooooooooooooooooooooooooooooooooooooooooooooooooooooooooooooZakładamy, że p jest liczbą pierwszą oraz, że k jest liczbą naturalną.

6.7.1. Niech a, b będą liczbami całkowitymi niepodzielnymi przez p i niech n ∈ N. Istniejewtedy liczba całkowita c niepodzielna przez p taka, że

apnb ≡

cpn (mod p

k).

D. Liczby b i pn+k są względnie pierwsze. Istnieją więc liczby całkowite u, v takie, że ub+vpn+k =1. Niech c = ua. Wtedy c jest liczbą niepodzielną przez p i mamy:

apnb −

cpn =

a−bcpnb =

a−uabpnb =

avpn+k

pnb =avpk

b .

Jest jasne, że avpk

b ≡ 0 (mod pk). �

Page 52: Liczby wymierne

48 Liczby wymierne 6. Przystawanie modulo m dla liczb wymiernych

6.7.2. Niech a będzie liczbą całkowitą niepodzielną przez p i niech n ∈ N. Istnieje wtedy liczbacałkowita b niepodzielna przez p taka, że

apn ≡

bpn (mod p

k)

oraz 1 6 b 6 pn+k − 1.

D. Niech b będzie resztą z dzielenia liczby a przez pn+k. Wtedy a = pn+k + b dla pewnego u ∈ Zoraz p - b, 1 6 b 6 pn+k − 1 oraz

apn −

bpn =

a−bpn =

pn+kupn = up

k

1 ≡ 0 (mod pk),

czyli apn ≡bpn (mod p

k). �

6.7.3. Niech a, b będą liczbami całkowitymi niepodzielnymi przez p takimi, że 1 6 a, b 6pn+k − 1. Jeśli apn ≡

bpn (mod p

k), to a = b.

D. Ponieważ a−bpn ≡ 0 (mod pk), więc pn+k | a− b. Przypuśćmy, że a 6= b. Wtedy |a− b| > pn+k,

gdy tymczasem |a− b| < pn+k. �

6.7.4. Niech a i b będą względnie pierwszymi liczbami całkowitymi niepodzielnymi przez p.Istnieje wtedy liczba całkowita c taka, że

p - c, 1 6 c < pk,a

b≡ c (mod pk).

D. Liczby b i pk są względnie pierwsze. Istnieją więc liczby całkowite u, v takie, że ub+ vpk = 1.Niech c będzie resztą z dzielenia liczby ua przez pk. Wtedy p - c, 1 6 c < pk oraz:

ab − c =

a−bcb =

a−uabb = avp

k

b .

Jest jasne, że avpk

b ≡ 0 (mod pk). �

U. Przy pomocy powyższych faktów łatwo opisać klasy abstrakcji rozważanej relacji przystawaniamodulo pk. �

Page 53: Liczby wymierne

����������������������������������������������������

7 Podzielność dla liczb wymiernych����������������������������������������������������

Oznaczenia: Q∗ := Q r {0}, Z∗ := Z r {0}.oooooooooooooooooooooooooooooooooooooooooooooooooooooooooooooooooooooo

7.1 Rozkład kanoniczny liczb wymiernychoooooooooooooooooooooooooooooooooooooooooooooooooooooooooooooooooooooo

7.1.1. Każda liczba wymierna q, różna od −1, 0, 1, ma przedstawienie w postaci

q = εpα11 pα22 · · · p

αss ,

gdzie ε ∈ {−1, 1}, p1, . . . , ps są parami różnymi liczbami pierwszymi i α1, . . . , αs są niezero-wymi liczbami całkowitymi. Przedstawienie to jest jednoznaczne z dokładnością do porządku.Przykład: 1528 = 2

−231517−1.

7.1.2. Każda niezerowa liczba wymierna q ma jednoznaczne przedstawienie w postaci

q = ε∏ppαp ,

gdzie ε = ±1, iloczyn przebiega zbiór wszystkich liczb pierwszych, każde αp jest liczbą całkowitąprzy czym αp = 0 dla prawie wszystkich liczb pierwszych p. Przedstawienie takie nazywaćbędziemy kanonicznym przedstawieniem liczby wymiernej q. W szczególności, q ∈ Z∗ ⇐⇒αp > 0 dla wszystkich liczb pierwszych p.

oooooooooooooooooooooooooooooooooooooooooooooooooooooooooooooooooooooo

7.2 Relacja podzielności w Q∗ooooooooooooooooooooooooooooooooooooooooooooooooooooooooooooooooooooooNiech a, b ∈ Q∗ i niech a = εa

∏pαp , b = εb

∏pβp będą rozkładami kanonicznymi. Mówić

będziemy, że a dzieli b i pisać a | b, jeśli ∀pαp 6 βp. W sytuacji, gdy a nie dzieli b pisać

będziemy a - b.Przyjmować będziemy również, że każde a ∈ Q∗ dzieli liczbę 0.

7.2.1. Liczba wymierna 16 dzieli liczbę wymierną 5, gdyż16 = 2

−13−150, 5 = 203051 oraz−1 < 0, −1 < 0 i 0 < 1.

7.2.2. 13 - 15 i15 - 13 .

7.2.3. Niech a ∈ Q. Wtedy 1 | a ⇐⇒ a ∈ Z.

7.2.4. Niech a, b ∈ Q, a 6= 0. Wtedy a | b ⇐⇒ ∃x∈Zb = xa.

49

Page 54: Liczby wymierne

50 Liczby wymierne. 7. Podzielność dla liczb wymiernych

7.2.5. Niech a, b, c ∈ Q przy czym a 6= 0 i b 6= 0. Wtedy:(1) a | a;(2) a | b i b | a =⇒ a = ±b;(3) a | b i b | c =⇒ a | c.

7.2.6. Niech a ∈ Q∗, b, c ∈ Q oraz x, y ∈ Z. Jeśli a | b i a | c, to a | xa+ yb.

Stąd w szczególności mamy:

7.2.7. Niech a ∈ Q∗, b, c ∈ Q. Jeśli a | b i a | c, to a | a± b. Jeśli a | b i x ∈ Z, to a | xb.

oooooooooooooooooooooooooooooooooooooooooooooooooooooooooooooooooooooo7.3 Nwd i nww dla liczb wymiernych. Definicje i przykładyooooooooooooooooooooooooooooooooooooooooooooooooooooooooooooooooooooooNiech A będzie skończonym niepustym zbiorem liczb wymiernych. Jeśli b ∈ Q∗, to piszemy

”b | A” w przypadku, gdy b | a dla wszystkich a ∈ A. Analogicznie, jeśli 0 6∈ A i b ∈ Q, topiszemy ”A | b” w przypadku, gdy a | b dla wszystkich a ∈ A.Mówimy, że liczba wymierna d jest największym wspólnym dzielnikiem zbioru A, jeśli

spełnione są następujące trzy warunki.

(0) d > 0;

(1) d | A;(2) ∀

e∈Q∗e | A =⇒ e | d.

Mówimy, że liczba wymierna w jest najmniejszą wspólną wielokrotnością zbioru A, jeślispełnione są następujące trzy warunki.

(0) w > 0;

(1) A | w;(2) ∀

u∈QA | u =⇒ w | u.

7.3.1. Niech A będzie niepustym skończonym zbiorem liczb wymiernych. Jeśli A 6= {0}, tonajwiększy wspólny dzielnik zbioru A istnieje i jest wyznaczony jednoznacznie. Jeśli 0 6∈ A,to najmniejsza wspólna wielokrotność zbioru A istnieje i jest wyznaczona jednoznacznie.

D. Niech A = {a1, . . . , an}. Załóżmy, że każda z liczb wymiernych a1, . . . , an jest różna od zera.Niech ai = εi

∏pα(i)p będzie rozkładem kanonicznym liczby ai, dla każdego i = 1, . . . , n. Dla każdej

liczby pierwszej p przyjmijmy:

γp := min(α(1)p , α

(2)p , . . . , α

(n)p

), δp := max

(α(1)p , α

(2)p , . . . , α

(n)p

)i niech d :=

∏pγp , w :=

∏pδp . Łatwo sprawdzić, że wtedy d jest największym wspólnym dzielnikiem

zbioru A oraz, że w jest najmniejszą wspólną wielokrotnością zbioru A. Jednoznaczność jest oczywista.Jeśli do zbioru A należy zero (w przypadku największego wspólnego dzielnika), to zamiast A

rozpatrujemy zbiór Ar {0}. �

Oznaczenia dla największego wspólnego dzielnika i najmniejszej wspólnej wielokrotnościliczb wymiernych stosujemy takie same, jak dla liczb całkowitych. W szczególności, stosowaćbędziemy też oznaczenia (a, b) := nwd(a, b), [a, b] := nww(a, b).

Page 55: Liczby wymierne

Liczby wymierne. 7. Podzielność dla liczb wymiernych 51

7.3.2.(12 ,13

)= 16 ;

[12 ,13

]= 1.

7.3.3. Niech a1, . . . , an ∈ Q∗. Jeśli nwd(a1, . . . , an) jest liczbą całkowitą, to wszystkie liczbya1, . . . , an są całkowite.

D. Niech d = nwd(a1, . . . , an) i załóżmy, że d ∈ Z. Wtedy d ∈ N. Istnieje więc liczba całkowita xtaka, że a1 = xd. Zatem a1 ∈ Z. Analogicznie a2, . . . , an ∈ Z. �

7.3.4. Jeśli a1, . . . , an ∈ Q∗, to nwd(1, a1, a2, . . . , an) = 1m , gdzie m jest najmniejszym wspól-nym mianownikiem liczb a1, . . . , an. ([Hass]).

oooooooooooooooooooooooooooooooooooooooooooooooooooooooooooooooooooooo7.4 Nwd i nww dla liczb wymiernych. Własnościoooooooooooooooooooooooooooooooooooooooooooooooooooooooooooooooooooooo

7.4.1. Niech a, b, c ∈ Q∗. Wtedy:(1) (a, b) = (b, a), [a, b] = [b, a],

(2) ((a, b), c) = (a, (b, c)) = (a, b, c), [[a, b], c) = [a, [b, c]] = [a, b, c],

(3) (ac, bc) = (a, b)|c|, [ac, bc] = [a, b]|c|.

D. Własności (1) i (2) są oczywiste. Udowodnimy (3). Niech a = εa∏pap , b = εb

∏pbp i c =

εc∏pcp będą rozkładami kanonicznymi liczb a, b, c. Wtedy ac = ±

∏pap+cp i bc = ±

∏pbp+cp są

rozkładami kanonicznymi liczb ac i bc. Mamy zatem: (a, b) =∏pdp , [a, b] =

∏pwp , (ac, bc) =

∏pd′p ,

[ac, bc] =∏pw′p , gdzie dp = min(ap, bp), wp = max(ap, bp), d′p = min(ap+ cp, bp+ cp), w

′p = max(ap+

cp, bp+cp). Ponieważ min(ap+cp, bp+cp) = min(ap, bp)+cp oraz max(ap+cp, bp+cp) = max(ap, bp)+cp,więc d′p = dp + cp i w

′p = wp + cp. Zatem (ac, bc) =

∏pd′p =∏pdp+cp =

(∏pdp)(∏pcp) = (a, b)|c| i

analogicznie [ac, bc] =∏pw′p =∏pwp+cp = (

∏pwp) (

∏pcp) = [a, b]|c|. �

7.4.2. ([a, b], c) = [(a, c), (b, c)), [(a, b), c] = ([a, c], [b, c]).

D. Wynika to z równości min(max(x, y), z) = max(min(x, z),min(y, z), max(min(x, y), z) =min(max(x, z),max(y, z), dla x, y, z ∈ R. �

7.4.3 ([Hass] 13). Jeśli a1, . . . , an ∈ Q∗, to

[a1, . . . , an] = 1(1a1,..., 1an

) , (a1, . . . , an) = 1[1a1,..., 1an

] .D. Niech ai = εi

∏pa(i)p będzie rozkładem kanonicznym liczby ai, dla i = 1, . . . , n. Wtedy a−1i =

εi∏p−a

(i)p jest rozkładem kanonicznym liczby a−1i (dla i = 1, . . . , n). Dla każdej liczby pierwszej p

oznaczmy: wp := max(a(1)p , . . . , a

(n)p

), dp := min

(−a(1)p , . . . ,−a(n)p

). Wtedy dp = −wp. Zatem(

1a1, . . . , 1an

)−1=(∏pdp)−1= (∏p−wp)−1 =

((∏pwp)−1

)−1=∏pwp = [a1, . . . , an].

Analogicznie wykazujemy drugą równość. �

Page 56: Liczby wymierne

52 Liczby wymierne 7. Podzielność dla liczb wymiernych

7.4.4. Niech a1, . . . , an ∈ Q∗ i niech b = |a1a2 · · · an|. Wtedy:

(a1, . . . , an)[ba1, . . . , ban

]= b, [a1, . . . , an]

(ba1, . . . , ban

)= b.

D. Wynika to z 7.4.1 i 7.4.3. �

7.4.5. (a, b)[a, b] = |ab|, dla a, b ∈ Q∗. (Wynika z 7.4.4 dla n = 2).

7.4.6. (a, b, c)[bc, ac, ab] = |abc|, [a, b, c](bc, ca, ab) = |abc|, dla a, b, c ∈ Q∗.(Wynika z 7.4.4 dla n = 3).

7.4.7. (a, b, c, d)[bcd, acd, abd, abc] = |abcd|, [a, b, c, d](bcd, acd, abd, abc) = |abcd|,dla a, b, c, d ∈ Q∗. (Wynika z 7.4.4 dla n = 4).

7.4.8. Niech a1, . . . , an ∈ Q∗ i niech d = nwd(a1, . . . , an). Wtedy a1 = a′1d, . . . , an = a′nd,gdzie a′1, . . . , a

′n są liczbami całkowitymi i przy tym nwd(a

′1, . . . , a

′n) = 1.

D. Z 7.4.1 wynika, że d = (a1, . . . , an) = (a′1d, . . . , a′nd) = (a′1, . . . , a′n)d i stąd (po podzieleniuprzez d) (a′1, . . . , a

′n) = 1. �

7.4.9. Niech a1, . . . , an ∈ Q∗ i niech d = nwd(a1, . . . , an). Istnieją wtedy liczby całkowitex1, . . . , xn takie, że d = x1a1 + · · ·+ xnan.

D. Korzystamy z 7.4.8. Niech a′1, . . . , a′n będą takimi liczbami całkowitymi, że ai = a′id dlai = 1, . . . , n. Wiemy, że wtedy nwd(a′1, . . . , a

′n) = 1. Na mocy klasycznego twierdzenia o największym

wspólnym dzielniku liczb całkowitych istnieją liczby całkowite x1, . . . , xn takie, że 1 = x1a′1+· · ·+xna′n.Mnożąc stronami tę równość przez d, otrzymujemy żądaną równość. �

7.4.10. Niech a1, . . . , an ∈ Q∗ i niech d = nwd(a1, . . . , an). Wówczas d należy do zbioru

{x1a1 + · · ·+ xnan; x1, . . . , xn ∈ Z} .Liczba d jest najmniejszą dodatnią liczbą wymierną należącą do tego zbioru. ([Hass] 16).

D. Fakt, że d należy do tego zbioru wynika z 7.4.9. Przypuśćmy, że q jest dodatnią liczbą wymiernąmniejszą od d i należącą do rozważanego zbioru. Niech q = y1a1+ · · ·+ynan, y1, . . . , yn ∈ Z. Ponieważd | a1, . . . , d | an, więc z równości q = y1a1 + · · ·+ ynan wynika, że d | q. Zatem q = md, gdzie m ∈ N.Stąd d 6 q i mamy sprzeczność. �

oooooooooooooooooooooooooooooooooooooooooooooooooooooooooooooooooooooo7.5 Względnie pierwsze liczby wymierneooooooooooooooooooooooooooooooooooooooooooooooooooooooooooooooooooooooMówimy, że dwie niezerowe liczby wymierne a, b są względnie pierwsze jeśli nie istnieje

żadna liczba pierwsza jednocześnie je dzieląca, tzn. jeśli p - nwd(a, b) dla każdej liczby pierw-szej p ([Hass] 13). Pojęcie to pokrywa się z klasyczną względnie pierwszością w przypadku,gdy liczby a i b są całkowite. Dla niecałkowitych liczb wymiernych to pojęcie nie ma specjal-nego sensu. Nie zachodzą żadne uogólnienia faktów znanych dla względnie pierwszych liczbcałkowitych. Dla przykładu, jeśli liczby a, b ∈ Q∗ są względnie pierwsze, to nww(a, b) nie musirównać się |ab|. Liczby 12 ,

13 są względnie pierwsze i

[12 ,13

]= 1 6= 12 ·

13 .

7.5.1. Niech a, b ∈ Q∗. Jeśli co najmniej jedna z tych liczb nie jest liczbą całkowitą, to liczbya i b są względnie pierwsze.

Page 57: Liczby wymierne

����������������������������������������������������

8 Twierdzenie Wolstenholme i jego uogólnienia����������������������������������������������������

oooooooooooooooooooooooooooooooooooooooooooooooooooooooooooooooooooooo8.1 Współczynniki AiooooooooooooooooooooooooooooooooooooooooooooooooooooooooooooooooooooooJeśli n jest liczbą naturalną, to przez Fn(x) oznaczamy wielomian stopnia n zmiennej x

zdefiniowany wzoremFn(x) = (x− 1)(x− 2) · · · (x− n).

Przyjmujemy ponadto oznaczenia:

Fn(x) = xn −A1xn−1 +A2xn−2 −A3xn−3 + · · ·+ (−1)nAn.

8.1.1. Przykłady:

F1 = x− 1

F2 = x2 − 3x+ 2

F3 = x3 − 6x2 + 11x− 6

F4 = x4 − 10x3 + 35x2 − 50x+ 24

F5 = x5 − 15x4 + 85x3 − 225x2 + 274x− 120

F6 = x6 − 21x5 + 175x4 − 735x3 + 1624x2 − 1764x+ 720

F7 = x7 − 28x6 + 322x5 − 1960x4 + 6769x3 − 13132x2 + 13068x− 5040

F8 = x8 − 36x7 + 546x6 − 4536x5 + 22449x4 − 67284x3 + 118124x2 − 109584x+ 40320

F9 = x9 − 45x8 + 870x7 − 9450x6 + 63273x5 − 269325x4 + 723680x3 − 1172700x2

+1026576x− 362880

F10 = x10 − 55x9 + 1320x8 − 18150x7 + 157773x6 − 902055x5 + 3416930x4 − 8409500x3

+12753576x2 − 10628640x+ 3628800

F11 = x11 − 66x10 + 1925x9 − 32670x8 + 357423x7 − 2637558x6 + 13339535x5 − 45995730x4

+105258076x3 − 150917976x2 + 120543840x− 39916800

F12 = x12 − 78x11 + 2717x10 − 55770x9 + 749463x8 − 6926634x7 + 44990231x6 − 206070150x5

+657206836x4 − 1414014888x3 + 1931559552x2 − 1486442880x+ 479001600

F13 = x13 − 91x12 + 3731x11 − 91091x10 + 1474473x9 − 16669653x8 + 135036473x7

−790943153x6 + 3336118786x5 − 9957703756x4 + 20313753096x3 − 26596717056x2

+19802759040x− 6227020800

F14 = x14 − 105x13 + 5005x12 − 143325x11 + 2749747x10 − 37312275x9 + 368411615x8

−2681453775x7 + 14409322928x6 − 56663366760x5 + 159721605680x4

−310989260400x3 + 392156797824x2 − 283465647360x+ 87178291200

53

Page 58: Liczby wymierne

54 Liczby wymierne. 8. Twierdzenie Wolstenholme i jego uogólnienia

8.1.2.

A1 =n∑i=1

i =n(n+ 1)2, A2 =

∑16i<j6n

ij, A3 =∑

16i<j<k6n

ijk, . . .

8.1.3.

An = n!, An−1 = n!n∑i=1

1i, An−2 = n!

∑16i<j6n

1ij, An−3 = n!

∑16i<j<k6n

1ijk, . . .

8.1.4. (x− 1)Fn(x− 1) = (x− n− 1)Fn(x).

8.1.5. (x− 1)(x− 2)Fn(x− 2) = (x− n− 1)(x− n− 2)Fn(x).

8.1.6 ([HW4] 87).

A1 =(n+12

)2A2 =

(n+13

)+A1

(n2

)3A3 =

(n+14

)+A1

(n3

)+A2

(n−12

)4A4 =

(n+15

)+A1

(n4

)+A2

(n−13

)+A3

(n−22

)...

(n− 1)An−1 =(n+1n

)+A1

( nn−1)+A2

(n−1n−2)+ · · ·An−2

(32

)nAn = 1 +A1 +A2 + · · ·An−1.

Dowód. Wynika to z równości 8.1.4.

8.1.7. Niech F (k)n (x) oznacza k-tą pochodną wielomianu Fn(x). Zachodzi równość:

An−k = (−1)n−k1k!F (k)n (0),

dla k = 0, 1, . . . n.

oooooooooooooooooooooooooooooooooooooooooooooooooooooooooooooooooooooo8.2 Współczynniki Ai dla liczb pierwszychooooooooooooooooooooooooooooooooooooooooooooooooooooooooooooooooooooooW tym podrozdziale zakładamy, że p jest nieparzystą liczbą pierwszą. Zajmować się bę-

dziemy współczynnikami A1, A2, . . . , An dla n = p− 1. Przypomnijmy:

(∗)Fp−1(x) = (x− 1)(x− 2) · · · (x− (p− 1))

= xp−1 −A1xp−2 +A2xp−3 −A3xp−4 + · · · −Ap−2x+Ap−1.

8.2.1. Liczby A1, A2, . . . , Ap−2 są podzielne przez p.

Page 59: Liczby wymierne

Liczby wymierne. 8. Twierdzenie Wolstenholme i jego uogólnienia 55

D. Sposób I. Ponieważ p jest liczbą pierwszą więc wszystkie liczby(p1

),(p2

), . . . ,(pp−1)są podzielne

przez p. Teza wynika zatem ze wzorów 8.1.6 (zastosowanych dla n = p− 1) i prostej indukcji.Sposób II. Z małego twierdzenia Fermata i twierdzenia Bezout’a wynika, że w pierścieniu wielo-

mianów Zp[x] zachodzi równość

xp−1 − 1 = (x− 1)(x− 2) · · · (x− (p− 1)) = Fp−1(x).

Oznacza to, ze liczby A1, A2, . . . , Ap−2 są równe zero modulo p, tzn., są podzielne przez p. �

8.2.2 (Twierdzenie Wilsona). Liczba (p− 1)! + 1 jest podzielna przez p.

D. Ze wzorów 8.1.6 (zastosowanych dla n = p − 1) mamy: (p − 1)Ap−1 = 1 + A1 + . . . Ap−2. Zatem(na mocy 8.2.1) (p− 1)Ap−1 ≡ 1 (mod p) i stąd (p− 1)! = Ap−1 ≡ −1 (mod p). �

8.2.3. Jeśli p > 5, to p2 | Ap−2.

D. ([HW4] 90). W równości (∗) podstawiamy x = p. Zauważmy, że Fp−1(p) = (p − 1)! = Ap−1.Mamy zatem: 0 = pp−1 −A1pp−2 +A2pp−3 −A3pp−4 + · · · −Ap−2p, czyli

Ap−2 = pp−2 −A1pp−3 +A2pp−4 −A3pp−5 + · · ·+Ap−3p.

Ponieważ liczby A1, A2, . . . , Ap−3 są podzielne przez p (na mocy 8.2.1), więc p2 | Ap−2. �

U. Powyższy fakt nie zachodzi dla p = 3. W tym przypadku bowiem Ap−2 = 3 nie jest podzielneprzez 9. �

8.2.4. Jeśli p > 5, to wszystkie liczby A3, A5, . . . , Ap−4, Ap−2 są podzielne przez p2.

D. Wykorzystamy 8.1.7 dla n = p−1. Niech m ∈ {0, 1, . . . , (p−5)/2}. Równość (∗) różniczkujemystronami 2m razy i do wyniku podstawiamy x = p. Wówczas (na mocy 8.1.7) otrzymujemy równośćpostaci

0 = u0pp−1−2m + u1A1pp−2−2m + u2A2pp−3−2m + · · ·+ up−2−2mAp−2−2mp,

gdzie u0, u1, . . . , up−2+2m są liczbami całkowitymi niepodzielnymi przez p. Ponieważ liczby A1, A2, . . . ,Ap−3−2m są podzielne przez p (na mocy 8.2.1), więc p2 | Ap−2−2m, dla wszystkich m = 0, 1, . . . , (p−5)/2. �

oooooooooooooooooooooooooooooooooooooooooooooooooooooooooooooooooooooo

8.3 Zastosowania dla liczb pierwszych i iloczynówoooooooooooooooooooooooooooooooooooooooooooooooooooooooooooooooooooooo

8.3.1. Jeśli p > 5 jest liczbą pierwszą, liczba

bp =p−1∑i=1

1 · 2 · · · i · · · (p− 1)

jest podzielna przez p2.

Page 60: Liczby wymierne

56 Liczby wymierne. 8. Twierdzenie Wolstenholme i jego uogólnienia

D. Liczba bp pokrywa się z liczbą Ap−2. Teza wynika zatem z 8.2.3. �

U. Rozkłady na czynniki pierwsze liczb postaci bn dla n niekoniecznie pierwszych.

n bn rozkład3 3 (3)4 11 (11)5 50 (2)(5)2

6 274 (2)(137)7 1764 (2)2(3)2(7)2

8 13068 (2)2(3)3(11)2

9 109584 (2)4(3)2(761)10 1026576 (2)4(3)2(7129)

n bn rozkład11 10628640 (2)5(3)2(5)(11)2(61)12 120543840 (2)5(3)2(5)(97)(863)13 1486442880 (2)7(3)3(5)(13)2(509)14 19802759040 (2)7(3)3(5)(29)(43)(919)15 283465647360 (2)8(3)3(5)(7)(1049)(1117)16 4339163001600 (2)8(3)4(5)2(7)(29)(41233)17 70734282393600 (2)11(3)4(5)2(7)(17)2(8431)18 1223405590579200 (2)11(3)4(5)2(7)(37)(1138979)19 22376988058521600 (2)12(3)7(5)2(7)(19)2(39541)20 431565146817638400 (2)12(3)7(5)2(7)(37)(7440427)

n bn rozkład21 8752948036761600000 (2)14(3)7(5)5(7)(11167027)22 186244810780170240000 (2)14(3)9(5)4(7)2(18858053)23 4148476779335454720000 (2)15(3)10(5)4(7)2(11)(23)2(53)(227)24 96538966652493066240000 (2)15(3)9(5)4(7)2(11)(761)(583859)25 2342787216398718566400000 (2)18(3)9(5)5(7)2(11)(577)(467183)26 59190128811701203599360000 (2)18(3)9(5)4(7)2(11)(109)(312408463)27 1554454559147562279567360000 (2)19(3)9(5)4(7)2(11)(13)(34395742267)28 42373564558110787183902720000 (2)19(3)10(5)4(7)2(11)(13)(521)(215087)(2789)29 1197348677077520393310044160000 (2)21(3)10(5)4(7)3(11)(13)(29)2(375035183)30 35027999979859805266492784640000 (2)21(3)10(5)4(7)3(11)(13)(43)2(4990290163)

8.3.2. Jeśli p > 5 jest liczbą pierwszą, to liczba

cp =∑

16i<j6p−11 · 2 · · · i · · · j · · · (p− 1)

jest podzielna przez p.

D. Liczba cp pokrywa się z liczbą Ap−3. Teza wynika zatem z 8.2.1. �

8.3.3. Jeśli p > 7 jest liczbą pierwszą, to liczba

dp =∑

16i<j<k6p−11 · 2 · · · i · · · j · · · k · · · (p− 1)

jest podzielna przez p2.

Page 61: Liczby wymierne

Liczby wymierne. 8. Twierdzenie Wolstenholme i jego uogólnienia 57

D. Liczba dp pokrywa się z liczbą Ap−4. Teza wynika zatem z 8.2.4. �

oooooooooooooooooooooooooooooooooooooooooooooooooooooooooooooooooooooo8.4 Sumy odwrotności iloczynówoooooooooooooooooooooooooooooooooooooooooooooooooooooooooooooooooooooo

8.4.1. Jeśli p > 5 jest liczbą pierwszą, to licznik ułamka

a

b=

∑16i<j6p−1

1ij

jest podzielny przez p.

D. Wynika to z równości

Ap−3 = (p− 1)!∑

16i<j6p−1

1ij

oraz faktu 8.2.1. �

8.4.2. Niech n > 1 będzie liczbą naturalną. Rozważmy wszystkie ułamki postaci 1pq , gdzie(p, q) = 1, 0 < p < q 6 n oraz p + q > n. Suma wszystkich takich ułamków jest równa 12 .([WaJ] 120(69), [B-zm] 71, [Dlt] 6/1999).

8.4.3. Suma wszystkich liczb postaci 1mn , gdzie m,n ∈ N, 1 6 m < n 6 1986, nie jest liczbącałkowitą. ([WaJ] 437(86)).

8.4.4. Jeśli p > 7 jest liczbą pierwszą, to licznik ułamka

a

b=

∑16i<j<k6p−1

1ijk

jest podzielny przez p2.

D. Wynika to z równości

Ap−4 = (p− 1)!∑

16i<j<k6p−1

1ijk

oraz faktu 8.2.4. �

8.4.5.n∑k=1

∑16i1<···<ik6n

1i1i2 · · · ik

= n.

([Bryn] 7.5).

D. 1 +∑nk=1

∑16i1<···<ik6n

1i1i2···ik =

(1 + 11

) (1 + 12

)· · ·(1 + 1n

)= 21 ·

32 ·43 · · ·

nn−1 = n. �

Page 62: Liczby wymierne

58 Liczby wymierne. 8. Twierdzenie Wolstenholme i jego uogólnienia

oooooooooooooooooooooooooooooooooooooooooooooooooooooooooooooooooooooo8.5 Odwrotności liczb względnie pierwszych: podstawowe faktyooooooooooooooooooooooooooooooooooooooooooooooooooooooooooooooooooooooNiech m, k ∈ N. W tym podrozdziale i dalszych podrozdziałach przez Um oznaczamy

zbiór wszystkich liczb naturalnych z przedziału [1,m] względnie pierwszych z m. Przykłady:U5 = {1, 2, 3, 4}, U10 = {1, 3, 7, 9}. Ponadto stosujemy następujące oznaczenie.

S(m, k) =∑i∈Um

1ik.

8.5.1. Dla każdej liczby naturalnej m 6= 2 licznik ułamka∑1 6 i 6 m(i,m) = 1

1i

jest podzielny przez m.

D. Jeśli m jest liczbą nieparzystą, to teza wynika z 8.6.3. Podamy dowód nie korzystający z tegofaktu.

Dla m = 1 twierdzenie jest oczywiste. Załóżmy, że m > 2. Jeśli m = 2n jest liczbą parzystą, toniech A = {i1, i2, . . . , is} będzie zbiorem wszystkich liczb naturalnych mniejszych od n i względniepierwszych z m. Jeśli natomiast m = 2n + 1 jest liczbą nieparzystą, to niech A = {i1, i2, . . . , is}będzie zbiorem wszystkich liczb naturalnych mniejszych od n+ 1 i względnie pierwszych z m. W obuprzypadkach mamy:

Um = {i1, i2, . . . , is, (m− i1), (m− i2), . . . , (m− is)} .

Zatem

S(m, 1) =∑i∈Um

1i=∑i∈A

(1i+1m− i

)=∑i∈A

m

i(m− i)= m∑i∈A

1i(m− i)

i stąd wynika teza. �

8.5.2. Jeśli k jest nieparzystą liczbą naturalną, to dla każdej liczby naturalnej m 6= 2 licznikułamka ∑

1 6 i 6 m(i,m) = 1

1ik

jest podzielny przez m.

D. Przy pewnym dodatkowym założeniu o k teza wynika z 8.6.3. Podamy dowód nie korzystającyz tego faktu.

Dla m = 1 twierdzenie jest oczywiste. Załóżmy, że m > 2. Jeśli m = 2n jest liczbą parzystą, toniech A = {i1, i2, . . . , is} będzie zbiorem wszystkich liczb naturalnych mniejszych od n i względniepierwszych z m. Jeśli natomiast m = 2n + 1 jest liczbą nieparzystą, to niech A = {i1, i2, . . . , is}będzie zbiorem wszystkich liczb naturalnych mniejszych od n+ 1 i względnie pierwszych z m. W obuprzypadkach mamy:

Um = {i1, i2, . . . , is, (m− i1), (m− i2), . . . , (m− is)} .

Page 63: Liczby wymierne

Liczby wymierne. 8. Twierdzenie Wolstenholme i jego uogólnienia 59

Zatem

S(m, k) =∑i∈Um

1ik=∑i∈A

(1ik+

1(m− i)k

)=∑i∈A

(m− i)k + ik

ik(m− i)k.

Ponieważ k jest nieparzyste, liczniki (m− i)k + ik są podzielne przez m i stąd wynika teza. �

8.5.3. Jeśli k jest parzystą liczbą naturalną i m ∈ N, to teza 8.5.2 nie musi zachodzić.

Przykłady. Liczby S(m, k) nie są podzielne przez m dla następujących k i m:

k m2 3, 4, 6, 8, 9, 12, 15, 16, 18, 24, 27, 30, 32, 33, 36, 45, 48, 50, 51, 54, 60, 64, 664 3, 4, 5, 6, 8, 9, 10, 12, 15, 16, 18, 20, 24, 25, 27, 30, 32, 33, 35, 36, 40, 45, 48, 506 3, 4, 6, 7, 8, 9, 12, 14, 15, 16, 18, 21, 24, 27, 28, 30, 32, 33, 35, 36, 42, 45, 48, 498 3, 4, 5, 6, 8, 9, 10, 12, 15, 16, 18, 20, 24, 25, 27, 30, 32, 33, 35, 36, 40, 45, 48, 50

8.5.4 (Hipoteza). Jeśli k jest liczbą parzystą i m jest potęgą trójki, to licznik liczby S(m, k)nie jest podzielny przez m.

8.5.5. Jeśli m jest liczbą parzystą, to licznik każdej z liczb S(3,m), S(4,m), S(8,m), S(9,m),S(16,m) nie jest podzielny przez m.

8.5.6.Wykaz pewnych par (m, k) takich, że liczniki liczb S(m, k) są podzielne przez m3.

k m1 39, 78, 155, 310, 465, 546, 793, 798, 9303 37, 55, 110, 111, 222, 310, 333, 355, 407, 666, 710, 814, 9305 5, 203, 4067 7, 21, 39, 63, 67, 78, 117, 133, 146, 201, 234, 259, 273, 399,438, 469, 518, 546, 603, 737, 777, 793, 798, 819, 876, 877

9 3, 5, 15, 30, 69, 178, 230, 345, 534, 690, 759, 89011 11, 68913 7, 13, 21, 39, 55, 59, 78, 91, 110, 133, 155, 182, 217, 231, 273,275, 310, 355, 385, 399, 413, 429, 462, 465, 507, 550, 607, 627,649, 651, 710, 715, 767, 770, 775, 793, 798, 806, 819, 858, 905, 930

1517 17, 34, 68, 149, 311, 401, 802, 95519 19, 39, 57, 78, 171, 203, 406, 546, 609, 674, 741, 793, 79821 7, 11, 21, 55, 77, 110, 231, 329, 385, 462, 770, 98723 23, 55, 110, 253, 275, 310, 355, 505, 550, 71025 13, 25, 65, 70, 78, 95, 117, 130, 146, 155, 182, 190, 210, 234, 259,260, 285, 310, 325, 327, 350, 351, 420, 438, 465, 475, 507, 518, 545,546, 570, 585, 630, 650, 654, 702, 730, 763, 775, 777, 780, 793, 798,855, 876, 910, 930, 950, 975, 981

27 7, 9, 21, 63, 177, 189, 413, 466, 53129 5, 29, 145, 290, 580

Page 64: Liczby wymierne

60 Liczby wymierne. 8. Twierdzenie Wolstenholme i jego uogólnienia

8.5.7.Wykaz pewnych par (m, k) takich, że liczniki liczb S(m, k) są podzielne przez m4.

k m13 54625 5, 39, 195, 39027 333 5549 5, 3553 55

oooooooooooooooooooooooooooooooooooooooooooooooooooooooooooooooooooooo8.6 Odwrotności liczb względnie pierwszych: Twierdzenia GesselaooooooooooooooooooooooooooooooooooooooooooooooooooooooooooooooooooooooPrzypominamy, że Um jest zbiorem wszystkich liczb naturalnych należących do przedziału

[1,m] względnie pierwszych z m. Ponadto:

S(m, k) =∑i∈Um

1ik.

8.6.1 ([Gess]). Jeśli a jest liczbą całkowitą względnie pierwszą z m, to licznik ułamka

(ak − 1)S(m, k)

jest podzielny przez m.

D. Zbiór {ai; i ∈ Um} zredukowany modulo m pokrywa się ze zbiorem Um. Stosując relacjęprzystawania modulom dla liczb wymiernych (w sensie takim, jak w jednym z poprzednich rozdziałów)mamy:

S(m, k) ≡∑i∈Um

1(ai)k

=1akS(m, k) (mod m)

i stąd wynika teza. �

8.6.2 ([Gess]). Jeśli k jest liczbą nieparzystą, to

2S(m, k) ≡ −mkS(m, k + 1) (mod m2).

D. Stosujemy relację przystawania modulom dla liczb wymiernych (w sensie takim, jak w jednymz poprzednich rozdziałów). Zauważmy najpierw, że

2S(m, k) =∑i∈Um

(1ik+

1(m− i)k

)=∑i∈Um

ik + (m− i)k

ik(m− i)k.

Ponieważ k jest nieparzyste, więc ik + (m− i)k ≡ kik−1m (mod m2). Stąd wynika, że

ik + (m− i)k

ik(m− i)k≡ kik−1m

ik(m− i)k= km

1i(m− i)k

(mod m2).

Ale (m − i)k ≡ −ik (mod m), więc 1(m−i)k ≡ −

1ik(mod m), stąd 1

i(m−i)k ≡ −1ik+1(mod m) czyli

km 1i(m−i)k ≡ −km

1ik+1(mod m2). Zatem:

2S(m, k) ≡∑i∈Um

kik−1m

ik(−1)k≡ −km

∑i∈Um

1ik+1

(mod m2)

i stąd wynika teza. �

Page 65: Liczby wymierne

Liczby wymierne. 8. Twierdzenie Wolstenholme i jego uogólnienia 61

8.6.3 ([Gess]). Załóżmy, że dla każdej liczby pierwszej p dzielącej m liczba k nie jest po-dzielna przez p− 1. Wtedy licznik ułamka S(m, k) jest podzielny przez m.

D. Z założenia wynika, że dla każdej liczby pierwszej p dzielącej m istnieje liczba całkowita aptaka, że p nie dzieli akp − 1. Istnieje (na mocy twierdzenia chińskiego o resztach) a ∈ Z takie, żea ≡ ap (mod p) dla wszystkich takich p. Wystarczy teraz zastosować 8.6.1. �

8.6.4 ([Gess]). Załóżmy, że k jest liczbą nieparzystą oraz, że dla każdej liczby pierwszej pdzielącej m liczba k + 1 nie jest podzielna przez p − 1. Wtedy licznik ułamka S(m, k) jestpodzielny przez m2.

D. Ponieważ k + 1 nie jest podzielne przez p − 1, więc p nie może być równe 2. Zatem m jestliczbą nieparzystą. Teza wynika zatem z 8.6.2 i 8.6.3. �

8.6.5 ([Gess]). Jeżeli k i n są liczbami naturalnymi, to licznik ułamka S(2n, k) jest podzielnyprzez 2n−1.

8.6.6 ([Gess]). Jeżeli k i n są liczbami naturalnymi i przy tym k jest nieparzyste, to licznikułamka S(2n, k) jest podzielny przez 22(n−1).

oooooooooooooooooooooooooooooooooooooooooooooooooooooooooooooooooooooo8.7 Twierdzenie Wolstenholme i inne twierdzeniaoooooooooooooooooooooooooooooooooooooooooooooooooooooooooooooooooooooo

8.7.1 (J. Wolstenholme 1862). Jeśli p > 5 jest liczbą pierwszą, to licznik ułamka

ab = 1 +

12 +

13 + · · ·+

1p−1

jest podzielny przez p2. ([Dic1] 96, [HW4] 88, [ShCY] 60, [WyKM] 733-78, [Ri97] 38, [Tao] 27).

D. Sposób 1. Niech d = nwd(a, b). Wtedy a = da1, b = db1, dla pewnych względnie pierwszychliczb naturalnych a1 i b1. Zauważmy, że

(p− 1)!ab = (p− 1)!(1 + 12 +

13 + · · ·+

1p−1

)= Ap−2,

czyli a1(p−1)! = b1Ap−2. Liczba Ap−2 jest podzielna przez p2 (na mocy 8.2.3), zatem a1 jest podzielneprzez p2 i konsekwentnie a jest podzielne przez p2.

Sposób 2. Wynika to z Twierdzenia Gessela 8.6.4 zastosowanego dla k = 1 i m = p. Możemy totwierdzenie zastosować gdyż liczba k jest nieparzysta oraz p− 1 - 2 = k + 1. �

U. Dla p = 3 powyższy fakt nie zachodzi: 1 + 12 =32 , 9 - 3. �

8.7.2 (P. Ossowski 2001). Jeśli p jest liczbą pierwszą, to licznik ułamka

ab = 1 +

12 +

13 + · · ·+

1(p−1)

(gdzie nwd(a, b) = 1) może być podzielny przez p3. Najmniejszą taką liczbą pierwszą jestp = 16843, liczby a i b mają wtedy odpowiednio 7309 i 7308 cyfr. (Maple).

Page 66: Liczby wymierne

62 Liczby wymierne. 8. Twierdzenie Wolstenholme i jego uogólnienia

8.7.3. Jeśli p > 5 jest liczbą pierwszą, to licznik ułamka

ab = 1 +

122 +

132 + · · ·+

1(p−1)2

jest podzielny przez p. ([HW4] 90, [ShCY] 60, [Ri97] 38).

D. Sposób 1. Oznaczmy ai = 1 · 2 · · · i · · · (p− 1). Wówczas dla i < j mamy:

aiaj = (p− 1)!1 · 2 · · · i · · · j · · · (p− 1).

Zatem1 + 1

22 +132 + · · ·+

1(p−1)2 =

1((p−1)!)2

∑p−1i=1 a

2i

= 1((p−1)!)2

((∑p−1i=1 ai

)2− 2∑i<j aiaj

)= 1

((p−1)!)2(A2p−2 − 2(p− 1)!Ap−3

)i teza wynika z 8.2.1.

Sposób 2. Wynika to z Twierdzenia Gessela 8.6.3 zastosowanego dla k = 2 i m = p. Możemy totwierdzenie zastosować gdyż p− 1 - 3 = k + 1. �

8.7.4 (P. Ossowski 2001). Jeśli p jest liczbą pierwszą, to licznik ułamka

ab = 1 +

122 +

132 + · · ·+

1(p−1)2

(gdzie nwd(a, b) = 1) może być podzielny przez p2. Najmniejszą taką liczbą pierwszą jestp = 16843, liczby a i b mają wtedy odpowiednio 14618 i 14617 cyfr. (Maple).

8.7.5. Niech p > 2 będzie liczbą pierwszą oraz niech

ab = 1 +

123 +

133 + · · ·+

1(p−1)3 ,

gdzie a i b są względnie pierwszymi liczbami naturalnymi. Wtedy a jest podzielne przez p.([Balt] 1994, wynika to z 8.5.2).

8.7.6. Niech p > 7 będzie liczbą pierwszą oraz niech

ab = 1 +

123 +

133 + · · ·+

1(p−1)3 ,

gdzie a i b są względnie pierwszymi liczbami naturalnymi. Wtedy a jest podzielne przez p2.

D. Stosujemy twierdzenie Gessela 8.6.4 dla k = 3 i m = p. �

8.7.7 (P. Ossowski 2001). Jeśli p jest liczbą pierwszą, to licznik ułamka

ab = 1 +

123 +

133 + · · ·+

1(p−1)3

(gdzie nwd(a, b) = 1) może być podzielny przez p3. Najmniejszą taką liczbą pierwszą jestp = 37, każda z liczb a i b ma wtedy 43 cyfry. (Maple).

Page 67: Liczby wymierne

Liczby wymierne. 8. Twierdzenie Wolstenholme i jego uogólnienia 63

8.7.8. Niech p > 2 będzie liczbą pierwszą. Niech k będzie liczbą naturalną oraz niech

ab = 1 +

12k +

13k + · · ·+

1(p−1)k ,

gdzie a i b są względnie pierwszymi liczbami naturalnymi. Wtedy:

(1) Jeśli p > k + 1, to p | a.(2) Jeśli k jest nieparzyste, to p | a.(3) Jeśli k jest nieparzyste i p > k + 2, to p2 | a.

D. (1). Twierdzenie Gessela 8.6.3 dla m = p.(2). 8.5.2.(3). Twierdzenie Gessela 8.6.4 dla m = p.

Podobne fakty udowodnił w 1866 roku P. Frost i w roku 1901 Glaisher ([Dic1] 96-100). �

8.7.9 (P. Ossowski 2001). Jeśli p jest liczbą pierwszą, to licznik ułamka

ab = 1 +

124 +

134 + · · ·+

1(p−1)4

(gdzie nwd(a, b) = 1) może być podzielny przez p2. Najmniejszą taką liczbą pierwszą jestp = 37, każda z liczb a i b ma wtedy 56 cyfr. (Maple).

8.7.10 (P. Ossowski 2001). Jeśli p jest liczbą pierwszą, to licznik ułamka

ab = 1 +

125 +

135 + · · ·+

1(p−1)5

(gdzie nwd(a, b) = 1) może być podzielny przez p3. Najmniejszą taką liczbą pierwszą jestp = 5, wtedy a = 532063, b = 21035. (Maple).

8.7.11. Jeśli p jest liczbą pierwszą nie należącą do zbioru {2, 3, 7, 13, 53, 79, 157}, to licznikułamka

11155 +

12155 + · · ·+

1(p−1)155 ,

jest podzielny przez p2. ([Zw] 2006).

8.7.12. Niech p > 2 będzie liczbą pierwszą. Niech k, n będą liczbami naturalnymi oraz niech

a

b=

∑1 6 i 6 pn

(i, p) = 1

1ik

gdzie a i b są względnie pierwszymi liczbami naturalnymi. Wtedy:

(1) Jeśli p > k + 1, to pn | a.(2) Jeśli k jest nieparzyste, to pn | a.(3) Jeśli k jest nieparzyste i p > k + 2, to p2n | m.

Page 68: Liczby wymierne

64 Liczby wymierne. 8. Twierdzenie Wolstenholme i jego uogólnienia

D. (1). Twierdzenie Gessela 8.6.3 dla m = p.(2). 8.5.2.(3). Twierdzenie Gessela 8.6.4 dla m = p. �

8.7.13. Niech p > 5 będzie liczbą pierwszą i niech 11p +12p +

13p + · · · +

1(p−1)p =

ab . Wtedy

p3 | a. ([Ibe] 2005).

F I. Śmieszek, Twierdzenie Wolstenholme i jego uogólnienia, pr. magisterska, UMK, Toruń 2001.oooooooooooooooooooooooooooooooooooooooooooooooooooooooooooooooooooooo8.8 Różne zadaniaoooooooooooooooooooooooooooooooooooooooooooooooooooooooooooooooooooooo

8.8.1. Niech ab = 1 +12 +

13 + · · ·+

1n , gdzie a, b ∈ N, nwd(a, b) = 1.

(1) Jeśli n = 20, to 5 | a. Czy 5 | a dla n = 100? ([WyKM] 827,831).(2) Liczba a jest podzielna przez 5 dla n = 4, 20, 24. Nie ma innych liczb naturalnych n

mniejszych od 10000 spełniających tę własność. (Maple).

(3) Liczba a jest podzielna przez 3 dla n = 2, 7, 22. Nie ma innych liczb naturalnych nmniejszych od 10000 spełniających tę własność. (Maple)

(4) Liczba a nie jest podzielna przez 3 dla n = 67. ([OM] Bułgaria 2004)

8.8.2. Licznik sumy 1 + 12 +13 + · · · +

1131 zapisanej w postaci ułamka nieskracalnego jest

liczbą złożoną. ([Zw] 2003).

8.8.3. Licznik każdej liczby postaci∑2nk=1

12k−1 , zapisanej w postaci ułamka nieskracalnego,

jest podzielny przez 22n. ([Cmj] 16(1)(1985) z.243).

8.8.4. Licznik każdej liczby postaci 1n+1n+1+ · · ·+

1n+m (gdzie n,m ∈ N), zapisanej w postaci

ułamka nieskracalnego, jest liczbą nieparzystą. ([Cmj] 29(3)(1998) z.601).

8.8.5 (Glaisher 1901, [Dic1] 100). Jeśli p > 3 będzie liczbą pierwszą, to

1 + 132n +

152n + · · ·+

1(p−2)2n ≡ 0 lub −

12 (mod p),

odpowiednio gdy p− 1 - 2n lub p− 1 | 2n.

8.8.6. Jeżeli n ∈ N, to przez pn, qn oznaczmy względnie pierwsze liczby naturalne takie, że

21 +

222 +

233 + · · ·+

2nn =

pnqn.

Wykazać, że jeżeli n > 3, to 8 | pn. ([Kw] 12/77 M 434).

Page 69: Liczby wymierne

Liczby wymierne 8. Twierdzenie Wolstenholme i jego uogólnienia 65

8.8.7. Jeżeli n ∈ N, to przez pn, qn oznaczmy względnie pierwsze liczby naturalne takie, że

21 +

222 +

233 + · · ·+

2nn =

pnqn.

Wykazać, że dla każdego k ∈ N istnieje n ∈ N takie, że liczby pn+1, pn+2, pn+3, . . . są podzielneprzez 2k. ([Ko01]).

8.8.8. Istnieje nieskończenie wiele liczb naturalnych n takich, że licznik ułamka nieskracal-nego 11 +

12 + · · ·+

1n nie jest potęgą liczby pierwszej. ([OM] Rosja 2002).

8.8.9. Dla każdej liczby pierwszej p licznik ułamkap−1∑k=0

(p−1k

)−2jest podzielny przez p.

([Zw] 2006).

F L. E. Dickson, Symmetric functions of 1, 2, . . . , p− 1 modulo p, [Dic1] 95-103.

Page 70: Liczby wymierne

����������������������������������������������������

9 Liczby postaci x1/x2 + x2/x3 + · · ·+ xs/x1����������������������������������������������������

Niech s ∈ N. Interesować nas będą dodatnie liczby wymierne postaci

x1x2+ x2x3 + · · ·+

xsx1,

gdzie x1, x2, . . . , xs są liczbami naturalnymi. Zbiór wszystkich takich dodatnich liczb wymier-nych oznaczać będziemy przez Bs. W szczególności interesować nas będą liczby naturalne tejpostaci. Zbiór wszystkich takich liczb naturalnych oznaczać będziemy przez As. Mamy więcAs = Bs ∩ N, A1 = B1 = {1} oraz

Bs =

{q ∈ Q+; ∃

x1,...,xs∈Nq = x1x2 +

x2x3+ · · ·+ xsx1

},

As =

{n ∈ N; ∃

x1,...,xs∈Nn = x1x2 +

x2x3+ · · ·+ xsx1

},

dla s > 2. Przez Q+ oznaczamy zbiór wszystkich liczb wymiernych większych od zera.oooooooooooooooooooooooooooooooooooooooooooooooooooooooooooooooooooooo9.1 Podstawowe własności zbiorów Bs i Asoooooooooooooooooooooooooooooooooooooooooooooooooooooooooooooooooooooo

9.1.1. Niech s ∈ N, q ∈ Q+. Jeśli q ∈ Bs, to q > s. W szczególności, jeśli liczba naturalnan należy do zbioru As, to n > s.

D. Niech q ∈ Bs. Wtedy q = x1x2+ x2x3 + · · · +

xsx1, dla pewnych x1, . . . , xs ∈ N. Z nierówności

pomiędzy średnią arytmetyczną i średnią geometryczną liczb x1x2 , . . . ,xsx1otrzymujemy:

q = s · 1s(x1x2+ x2x3 · · ·+

xsx1

)> s · s

√x1x2x2x3· · · xsx1 = s ·

s√1 = s.

Zatem q > s. Stąd oraz z faktu, że As = Bs ∩ N wynika, że jeśli n ∈ As, to n > s. �

9.1.2. Niech s ∈ N. Wtedy s ∈ As. Jeśli x1, . . . , xs są liczbami naturalnymi takimi, żes = x1x2 +

x2x3+ · · ·+ xsx1 , to x1 = x2 = · · · = xs.

D. Liczba s należy do As, gdyż s = x1x2 +x2x3+ · · ·+ xsx1 , dla x1 = x2 = · · · = xs = 1. Załóżmy teraz,

że s = x1x2 +x2x3+ · · ·+ xsx1 , gdzie x1, . . . , xs ∈ N. Wtedy średnia arytmetyczna liczb x1x2 ,

x2x3. . . . , xsx1 , jest

równa średniej geometrycznej tych liczb. Wszystkie więc te liczby są jednakowe. Niech a = x1x2 =x2x3=

· · · = xsx1 . Wtedy s = sa, więc a = 1 i stąd x1 = x2 = · · · = xs. �

9.1.3. Niech s ∈ N. Jeśli x1, . . . , xs są liczbami naturalnymi takimi, że s = x1x2 +x2x3+ · · ·+ xsx1

oraz nwd(x1, . . . , xs) = 1, to x1 = x2 = · · · = xs = 1. (Wynika z 9.1.2).

9.1.4 (Bondarenko 2000). Każda liczba naturalna n > 12 należy do zbioru A12. ([Bond]).

66

Page 71: Liczby wymierne

Liczby wymierne. 9. Liczby postaci x1/x2 + x2/x3 + · · ·+ xs/x1 67

9.1.5. Niech s > 2. Jeśli q jest dodatnią liczbą wymierną, to następujące warunki są równo-ważne.

(1) q ∈ Bs.(2) q = x1x2 +

x2x3+ · · ·+ xsx1 , dla pewnych x1, . . . , xs ∈ N takich, że nwd(x1, . . . , xs) = 1.

(3) q = y1y2 +y2y3+ · · ·+ ysy1 , dla pewnych y1, . . . , ys ∈ Q+.

(4) q = u1 + u2 + · · ·+ us, dla pewnych u1, . . . , us ∈ Q+ takich, że u1u2 · · ·us = 1.

D. (1) ⇐⇒ (2). Załóżmy, że q ∈ Bs. Niech q = a1a2 +a2a3+ · · ·+ asa1 , gdzie a1, . . . , as ∈ N. Niech d =

nwd(a1, . . . , as). Istnieją wtedy liczby naturalne x1, . . . , xs takie, że a1 = x1d, a2 = x2d, . . . , as = xsd.Wtedy nwd(x1, . . . , xn) = 1 oraz x1x2 +

x2x3+ · · ·+ xsx1 =

x1dx2d+ x2dx3d + · · ·+

xsdx1d= a1a2 +

a2a3+ · · ·+ asa1 = q.

Wykazaliśmy więc implikację (1) ⇒ (2). Implikacja (2) ⇒ (1) jest oczywista.(1) ⇐⇒ (3). Jest oczywiste, że zachodzi implikacja (1) ⇒ (3). Wykażemy implikację (3) ⇒ (1).

Niech q = y1y2 +y2y3+· · ·+ ysy1 , gdzie y1, . . . , ys ∈ Q+. Niech d będzie wspólnym mianownikiem wszystkich

liczb wymiernych y1, . . . , ys. Wtedy y1 = x1d , y2 =x2d , . . . , ys =

xsd , dla pewnych x1, . . . , xs ∈ N. Mamy

wtedy x1x2 +x2x3+ · · ·+ xsx1 =

x1/dx2/d+ x2/dx3/d

+ · · ·+ xs/dx1/d= y1y2 +

y2y3+ · · ·+ ysy1 = q. Zatem q ∈ Bs.

(3) ⇐⇒ (4). Załóżmy, że q = y1y2+y2y3+· · ·+ ysy1 , gdzie y1, . . . , ys ∈ Q+. Niech u1 = y1y2 , u2 =

y2y3, . . . ,

us =ysy1. Wtedy u1, . . . , us są dodatnimi liczbami wymiernymi, u1 · · ·us = 1 oraz q = u1 + · · · + us.

Wykazaliśmy więc implikację (1) ⇒ (4).Niech teraz q = u1 + · · · + us, gdzie u1, . . . , us są dodatnimi liczbami wymiernymi takimi, że

u1 · · ·us = 1. Niech

y1 = 1, y2 = 1u1, y3 = 1

u1u2, . . . , ys−1 = 1

u1u2···us−2 , ys =1

u1u2···us−1 .

Wtedy y1y2 +y2y3+ · · ·+ ysy1 = u1 + u2 + · · ·+ us = q. �

Jeśli w 9.1.5 założymy dodatkowo, że q jest liczbą naturalną, to otrzymamy następującetwierdzenie.

9.1.6. Niech s > 2. Jeśli n jest liczbą naturalną, to następujące warunki są równoważne.

(1) n ∈ As.(2) n = x1x2 +

x2x3+ · · ·+ xsx1 , dla pewnych x1, . . . , xs ∈ N takich, że nwd(x1, . . . , xs) = 1.

(3) n = y1y2 +y2y3+ · · ·+ ysy1 , dla pewnych y1, . . . , ys ∈ Q+.

(4) n = u1 + u2 + · · ·+ us, dla pewnych u1, . . . , us ∈ Q+ takich, że u1u2 · · ·us = 1.

Następne fakty są wnioskami z twierdzeń 9.1.5 i 9.1.6.

9.1.7. Każda liczba postaci xs1+x

s2+···+xss

x1x2···xs , gdzie x1, . . . , xs ∈ Q+, należy do zbioru Bs.

D. Oznaczmy: q = xs1+x

s2+···+x

ss

x1x2···xs , gdzie x1, . . . , xs ∈ Q+. Pokażemy, że q ∈ Bs.

(Sposób I). Niech ui =xsi

x1x2···xs , dla i = 1, . . . , s. Wtedy u1, . . . , us ∈ Q+, u1u2 · · ·us = 1 orazq = u1 + u2 + · · ·+ us. Teza wynika zatem z twierdenia 9.1.5.(Sposób II). Oznaczmy w = x1x2 · · ·xs i niech yi = 1

w

(xsixs−1i+1x

s−2i+2 · · ·x2i+s−2x1i+s−1

), dla i =

1, 2, . . . , s przy czym xs+j = xj dla j ∈ N. Wtedy y1, . . . , ys są liczbami naturalnymi oraz y1y2 +y2y3+

· · ·+ ysy1 =xs1+x

s2+···+x

ss

x1x2···xs = q. Zatem q ∈ Bs. �

Page 72: Liczby wymierne

68 Liczby wymierne. 9. Liczby postaci x1/x2 + x2/x3 + · · ·+ xs/x1

9.1.8. Niech s > 2. Każda liczba postacixs−11 x2+xs−12 x3+···+xs−1s−1xs+x

s−1s x1

x1x2···xs , gdzie x1, . . . , xs ∈Q+, należy do zbioru Bs.

D. Oznaczmy: q =xs−11 x2+x

s−12 x3+···+xs−1s−1xs+x

s−1s x1

x1x2···xs , gdzie x1, . . . , xs ∈ Q+. Niech ui =xs−1ixi+1

x1x2···xs ,dla i = 1, . . . , s, przy czym xs+1 = x1. Wtedy u1, . . . , us ∈ Q+, u1u2 · · ·us = 1 oraz q = u1 + u2 +· · ·+ us. Teza wynika zatem z twierdenia 9.1.5. �

9.1.9. Bn +Bm ⊆ Bn+m, dla n,m ∈ N.

D. Niech a ∈ Bn, b ∈ Bm. Pokażemy, że a+ b ∈ Bn+m.(Sposób I). Z twierdzenia 9.1.5 wynika, że istnieją dodatnie liczby wymierne u1, . . . , un oraz

v1, . . . , vm takie, że u1 · · ·un = 1, v1 · · · vm = 1, a = u1 + · · ·un i b = v1 + · · · + vm. Wtedyu1u2 · · ·unv1v2 · · · vm = 1 oraz a + b = u1 + · · · + un + v1 + · · · vm. Teza wynika więc z twierdze-nia 9.1.5.

(Sposób II). Istnieją liczby naturalne x1, . . . , xn, y1, . . . , ym takie, że a = x1x2+ · · · + xnx1 , b =y1

y2+ · · · ymy1 . Mamy wtedy

a+ b = x1y1x2y1+ x2y1x3y1

+ · · ·+ xny1x1y1+ y1x1y2x1

+ y2x1y3x1+ · · · ymx1y1x1

.

Zatem a+ b ∈ Bn+m. �

9.1.10. BmBn ⊆ Bmn, dla n,m ∈ N.

D. Niech a ∈ Bm, b ∈ Bn. Pokażemy, że ab ∈ Bnm.(Sposób I). Z twierdzenia 9.1.5 wynika, że istnieją dodatnie liczby wymierne u1, . . . , um oraz

v1, . . . , vn takie, że u1 · · ·um = 1, v1 · · · vn = 1, a = u1 + · · ·um i b = v1 + · · ·+ vn. Niech wij = uivj ,dla i = 1, . . . ,m, j = 1, . . . , n. Iloczyn wszystkich liczb postaci wij jest równy 1 i ich suma wynosi ab.Teza wynika więc z twierdzenia 9.1.5.

(Sposób II). Istnieją liczby naturalne x1, . . . , xm, y1, . . . , yn takie, że a = x1x2+ · · · + xmx1 , b =y1

y2+ · · · yny1 . Przyjmijmy:

z(p−1)n+i =(xn−i+1p xi−1p+1

)yi,

dla p = 1, 2, . . . ,m, i = 1, 2, . . . , n, przy czym xm+1 = x1. Mamy wtedy mn liczb naturalnychz1, z2, . . . , zmn. Zauważmy, że

z(p−1)n+iz(p−1)n+i+1

= xpxp+1

yiyi+1, dla p = 1, 2, . . . ,m oraz i < n. Ponadto,

z(p−1)n+nzpn+1

= xpxp+1

yny1, dla p = 1, 2, . . . ,m. Stąd wynika, że

z1z2+ z2z3 + · · ·+

zmnz1=(x1x2+ · · ·+ xmx1

)(y1y2+ · · ·+ yny1

)= ab.

Zatem ab ∈ Bmn. �

Z powyższych faktów wynika:

9.1.11. Am +An ⊆ Am+n, AmAn ⊆ Amn, dla m,n ∈ N.

9.1.12. Jeśli q ∈ Bs, to q + 1 ∈ Bs+1. Jeśli n ∈ As, to n+ 1 ∈ As+1.

Page 73: Liczby wymierne

Liczby wymierne. 9. Liczby postaci x1/x2 + x2/x3 + · · ·+ xs/x1 69

oooooooooooooooooooooooooooooooooooooooooooooooooooooooooooooooooooooo9.2 Zbiór B2oooooooooooooooooooooooooooooooooooooooooooooooooooooooooooooooooooooo

9.2.1. Jedyną liczbą naturalną n należącą do zbioru B2 jest n = 2. Innymi słowy: A2 = {2}.

D. Niech n ∈ A2 = B2∩N. Niech n = xy +yx , gdzie x, y ∈ N, nwd(x, y) = 1. Wtedy x2+y2 = nxy.

Przypuśćmy, że x > 2. Istnieje wtedy liczba pierwsza p taka, że p | x. Wtedy prawa strona równościx2+y2 = nxy jest podzielna przez p, więc (ponieważ p | x2 i p | x2+y2) liczba y również jest podzielnaprzez p. To jest jednak sprzecznością, gdyż nwd(x, y) = 1. Zatem x = 1 i analogicznie y = 1. Stądn = 11 +

11 = 2. �

9.2.2. Niech p będzie liczbą pierwszą i n liczbą naturalną. Liczba np należy do zbioru B2 wtedyi tylko wtedy, gdy n = 2p lub n = p2 + 1.

D. Oczywiście liczby 2pp = 2 =11 +

11 i

p2+1p =

p1 +

1p należą do B2. Pokażemy, że innych tego

typu liczb w zbiorze B2 nie ma.Załóżmy, że np ∈ B2. Niech

np =

xy +

yx =

x2+y2

xy , gdzie x, y ∈ N, nwd(x, y) = 1. Wtedy

p(x2 + y2) = nxy.

Załóżmy najpierw, że p | n. Niech n = pa, a ∈ N. Wtedy x2+y2 = axy, więc a = x2+y2

xy =xy +

yx ∈

A2. Ale A2 = {2}, więc a = 2. Jeśli więc p | n, to n = 2p.Załóżmy teraz, że p - n. Wtedy p | xy, więc p | x lub p | y. Dla ustalenia uwagi niech p | x. Wtedy

p - y, gdyż nwd(x, y) = 1. Niech x = pαa, a ∈ N, p - a, α > 1. Wtedy p2α+1a2 + py2 = npαay. Stądwynika, że α = 1 (bowiem gdy α > 2, to mamy sprzeczność z tym, że p - y). Zatem (pa)2 + y2 = nayPrzypuśćmy, że a > 2. Niech q będzie liczbą pierwszą dzielącą a. Wtedy z równości (pa)2 + y2 = naywynika, że q | y; wbrew temu, że nwd(x, y) = 1. Zatem a = 1, tzn. x = p. Mamy więc p2 + y2 = ny,p - n, p - y. Jeśli y > 2, to mamy oczywistą sprzeczność. Zatem y = 1 i stąd n = p2 + 1. �

9.2.3. Niech n ∈ N. Liczba n2 należy do zbioru B2 wtedy i tylko wtedy, gdy n = 4 lub n = 5.(Wynika z 9.2.2 dla p = 2).

9.2.4. Niech n ∈ N. Liczba n3 należy do zbioru B2 wtedy i tylko wtedy, gdy n = 6 lub n = 10.(Wynika z 9.2.2 dla p = 3).

9.2.5. Niech p będzie liczbą pierwszą oraz s, n ∈ N, p - n. Liczba nps należy do zbioru B2wtedy i tylko wtedy, gdy n = p2s + 1.

D. Jeśli n = p2s + 1, to nps ∈ B2, gdyż wtedynps =

p2s+1ps =

ps

1 +1ps . Załóżmy teraz, że

nps ∈ B2 i

niech x, y będą względnie pierwszymi liczbami naturalnymi takimi, że xy +yx =

nps . Wtedy

(1) ps(x2 + y2) = nxy.

Ponieważ p - n, więc p | x lub p | y. Zmieniając ewentualnie kolejność występowania liczb x, y, możemyzałożyć, że p | x. Wtedy ps | x oraz p - y. Niech x = ptu, u ∈ N, p - u, t > s. Jeśli t > s, to prawa

Page 74: Liczby wymierne

70 Liczby wymierne. 9. Liczby postaci x1/x2 + x2/x3 + · · ·+ xs/x1

strona równości (1) jest podzielna przez p i lewa strona tej równości nie jest podzielna przez p. Zatemt = s i mamy p2su2+ y2 = nuy. Jeśli u > 2, to istnieje liczba pierwsza q dzieląca u i wtedy z równościp2su2 + y2 = nuy wynika, że q | y wbrew temu, że nwd(x, y) = 1. Zatem u = 1, czyli x = ps. Mamywięc równość p2s + y2 = ny. Jeśli y > 2, to mamy oczywistą sprzeczność z tym, że nwd(x, y) = 1.Zatem y = 1. Ostatecznie n = p2s + 1. �

9.2.6. Niech a, b, c, d będą liczbami naturalnymi takimi, że a 6 b, c 6 d, nwd(a, b) = 1 oraznwd(c, d) = 1. Jeśli ab +

ba =

cd +

dc , to a = c i b = d.

D. Załóżmy, że ab +ba =

cd +

dc . Wtedy cd(a

2 + b2) = ab(c2 + d2). Ponieważ nwd(a, b) = 1 inwd(c, d) = 1, więc nwd(ab, a2 + b2) = 1 i nwd(cd, c2 + d2) = 1. Zatem ab | cd i cd | ab, czyli ab = cdi stąd a2 + b2 = c2 + d2. Stąd dalej mamy: (b− a)2 = b2 − 2ab+ a2 = d2 − 2cd+ c2 = (d− c)2, czylib− a = d− c = u, gdzie u > 0. Zatem b = a+ u, d = c+ u. Ale ab = cd, więc a2 + ua = c2 + cu i stąd(a− c)(a+ c+ u) = 0. Ponieważ a+ c+ u > 0, więc a = c i stąd wynika, że b = d. �

oooooooooooooooooooooooooooooooooooooooooooooooooooooooooooooooooooooo9.3 Zbiór B3 i liczby (a3 + b3 + c3)/abcooooooooooooooooooooooooooooooooooooooooooooooooooooooooooooooooooooooPrzypomnijmy, że B3 jest zbiorem wszystkich dodatnich liczb wymiernych xy+

yz+zx , gdzie

x, y, z ∈ N, natomiast A3 jest zbiorem wszystkich naturalnych liczb tej postaci. Wiemy już, żeA1 = {1}, A2 = {2}. Oczywiście 3 ∈ A3. Do zbioru A3 należą również inne liczby naturalne,na przykład 5 = 12 +

24 +

41 lub 6 =

212 +

129 +

92 . Wykażemy w następnym podrozdziale, że

zbiór A3 jest nieskończony. W tym podrozdziale wykażemy, że dodatnia liczba wymierna qnależy do zbioru B3 wtedy i tylko wtedy, gdy q = a

3+b3+c3abc dla pewnych liczb naturalnych

a, b, c.

9.3.1. Niech q będzie liczbą wymierną taką, że x2z + y2x + z2y = qxyz, dla pewnych liczbcałkowitych x, y, z. Niech

a = xyz(x+ y + z)(x2 + y2 + z2 − xz − yz − xy),b = (xz + yz + xy)(x2y2 + z2x2 + y2z2 − y2zx− yzx2 − z2yx),c = (x2y4 + y2z4 + z2x4)− xyz(x2y + y2z + z2x).

Wtedy a, b, c są liczbami całkowitymi takimi, że a3 + b3 + c3 = qabc. ([BrG], [Rus1]).

D. Standardowy rachunek; sprawdziłem to przy pomocy Maple. �

9.3.2. Zachodzi równość zbiorów:

B3 ={xy +

yz +

zx ; x, y, z ∈ N

}={a3+b3+c3abc ; a, b, c ∈ N

}.

([BrG], [Rus1]).

D. Oznaczmy zbiór po prawej stronie przez C3. Inkluzja C3 ⊆ B3 wynika z 9.1.7. Niech q ∈ B3,niech q = xy +

yz +

zx , gdzie x, y, z ∈ N. Możemy założyć, że nwd(x, y, z) = 1. Jeśli x = y = z = 1, to

q = 3 i wtedy 3 = 13+13+13

1·1·1 . Dalej możemy więc założyć, że (x, y, z) 6= (1, 1, 1). Z równości q =xy+yz+zx

wynika, że x2z + y2x + z2y = qxyz. Zatem a3 + b3 + c3 = qabc, gdzie liczby a, b, c są zdefiniowanew twierdzeniu 9.3.1. Korzystając z klasycznych nierówności łatwo stwierdzamy, że a, b, c ∈ N. Zatemq ∈ C3 i tym samym wykazaliśmy, że B3 ⊆ C3. Ostatecznie B3 = C3. �

Z powyższych faktów otrzymujemy:

Page 75: Liczby wymierne

Liczby wymierne. 9. Liczby postaci x1/x2 + x2/x3 + · · ·+ xs/x1 71

9.3.3 (Erdos, Niven 1946). Niech q będzie dodatnią liczbą wymierną. Następujące warunkisą równoważne.

(1) q ∈ B3 tzn. istnieją liczby naturalne x, y, z takie, że q = xy +yz +

zx .

(2) Istnieją liczby naturalne x, y, z takie, że q = xy +yz +

zx oraz nwd(x, y, z) = 1.

(3) Istnieją dodatnie liczby wymierne x, y, z takie, że q = xy +yz +

zx .

(4) Istnieją dodatnie liczby wymierne a, b, c takie, że q = a3+b3+c3abc .

(5) Istnieją liczby naturalne a, b, c takie, że q = a3+b3+c3abc .

(6) Istnieją liczby naturalne a, b, c takie, że q = a3+b3+c3abc oraz nwd(a, b, c) = 1.

([BrG], [Rus1]).

9.3.4. Niech n ∈ N. Następujące warunki są równoważne.(1) n ∈ A3 tzn. istnieją liczby naturalne x, y, z takie, że n = xy +

yz +

zx .

(2) Istnieją liczby naturalne x, y, z takie, że n = xy +yz +

zx oraz nwd(x, y, z) = 1.

(3) Istnieją dodatnie liczby wymierne x, y, z takie, że n = xy +yz +

zx .

(4) Istnieją dodatnie liczby wymierne a, b, c takie, że n = a3+b3+c3abc .

(5) Istnieją liczby naturalne a, b, c takie, że n = a3+b3+c3abc .

(6) Istnieją liczby naturalne a, b, c takie, że n = a3+b3+c3abc oraz nwd(a, b, c) = 1.

(7) Istnieją parami względnie pierwsze liczby naturalne a, b, c takie, że n = a3+b3+c3abc .

([Mon] 53(4)(1946) 223-224, [BrG], [Bond], [Rus1]).

9.3.5 (Erdos, Niven 1946). Niech x, y, z będą liczbami naturalnymi takimi, że liczba xy +yz +

zx jest naturalna oraz nwd(x, y, z) = 1. Niech a = nwd(x, y), b = nwd(y, z), c =

nwd(z, x). Wtedy liczby a, b, c są parami względnie pierwsze oraz x = c2a, y = a2b i z = b2c.([Mon] 53(4)(1946) 223-224).

D. (Erdos, Niven [Mon] 1946). Ponieważ nwd(x, y, z) = 1, więc jest oczywiste, że liczby a, b, c sąparami względnie pierwsze. Niech xy +

yz +

zx = m ∈ N. Mamy wtedy równość

(1) x2z + y2x+ z2y = mxyz.

Część I. Pokażemy najpierw, że a2 | y. Jeśli a = 1, to nie ma czego wykazywać. Załóżmy, że a > 2.Niech a = pr11 · · · prss będzie rozkładem kanonicznym liczby a. Niech i ∈ {1, 2, . . . , s} i oznaczmy p = pi,r = ri. Wówczas p | x oraz p | y (gdyż p | a = nwd(x, y)). Stąd p - z (bo nwd(x, y, z) = 1). Niechx = pαu, y = pβv, gdzie u, v ∈ N, p - u, p - v, α > r > 1, β > r > 1. Wstawiając to do równości (1)otrzymujemy równość

p2αu2z + p2β+αv2u+ pβz2v = mpα+βuvz,

z której jasno wynika, że β = 2α. Zatem a = nwd(x, y) = nwd(pαu, pβv) = nwd(pαu, p2αv) =pαnwd(u, pαv) = pαw, gdzie p - w. Stąd wnioskujemy, że α = r i stąd, że β = 2α = 2r. Zatem (pr)2

dzieli y. Dla każdego więc i ze zbioru {1, 2, . . . , s} mamy podzielność (prii )2 | y. To implikuje, że liczba

a2 =∏si=1 (p

rii )2 dzieli liczbę y. W ten sam sposób pokazujemy, że b2 | z, c2 | x. Zanotujmy:

(2) c2 | x, a2 | y, b2 | z.

Page 76: Liczby wymierne

72 Liczby wymierne. 9. Liczby postaci x1/x2 + x2/x3 + · · ·+ xs/x1

Część II. Ponieważ a | x, c2 | x oraz nwd(a, c2) = 1, więc ac2 | x. Analogicznie ba2 | y, cb2 | z.Zatem x = iac2, y = jba2, z = kcb2, dla pewnych i, j, k ∈ N. Pokażemy, że i = j = k = 1.Przypuśćmy, że i > 2. Istnieje wtedy liczba pierwsza p taka, że p | i. Wtedy p | x (bo x = iac2). Z

równości (1) wynika więc, że p | z2y czyli, że p | y lub p | z.Przypuśćmy, że p | y. Wtedy p - z (bo p | x, p | y oraz nwd(x, y, z) = 1). Ponadto, p | a = nwd(x, y).

Ale a = nwd(x, y) = nwd(iac2, jba2) = nwd(ic2, jba)a, więc nwd(ic2, jba) = 1. Tymczasem liczbanwd(ic2, jba) jest podzielna przez p (bo p | i oraz p | a). Sprzeczność ta implikuje, że p - y.Zatem p | z, p | x oraz p - y. Stąd wynika, że p | c = nwd(z, x). Niech x = pαu, z = pγw, gdzie

u,w ∈ N, p - u, p - w, α > 1, γ > 1. Wstawiając to do równości (1) otrzymujemy równość

p2α+γu2w + pαuy2 + p2γw2y = mpα+γuyw,

z której wynika, że α = 2γ. Stąd dalej mamy: p2γu = pαu = x = iac2 = iap2γw2 = p2γ+1r, dlapewnego r ∈ N. Zatem p | u wbrew temu, że p - u.Otrzymana sprzeczność implikuje, że i = 1. Analogicznie dowodzimy, że j = 1 oraz k = 1. Zatem

x = ac2, y = ba2, z = cb2 i to kończy dowód. �

9.3.6. Niech x, y, z ∈ N. Jeśli xy +yz +

zx jest liczbą naturalną, to xyz jest sześcianem.

([OM] Serbia-Czrnogóra 2004).

D. Niech d = nwd(x, y, z), x = dx1, y = dy1, z = dz1, gdzie x1, y1, z1 ∈ N, nwd(x1, y1, z1) = 1.Ponieważ x1y1 +

y1z1+ z1x1 =

x1dy1d+ y1dz1d +

z1dx1d= xy +

yz +

zx , więc

x1y1+ y1z1 +

z1x1jest liczbą naturalną. Z

twierdzenia 9.3.5 wynika więc, że istnieją liczby naturalne a, b, c takie, że x1 = ac2, y1 = ba2, z1 = cb2.Mamy zatem xyz = (dx1)(dy1)(dz1) = d3(ac2)(ba2)(cb2) = (abcd)3, czyli xyz jest sześcianem liczbynaturalnej. �

oooooooooooooooooooooooooooooooooooooooooooooooooooooooooooooooooooooo9.4 Nieskończoność zbioru A3ooooooooooooooooooooooooooooooooooooooooooooooooooooooooooooooooooooooWiemy (patrz 9.3.4), że zbiór A3 pokrywa się ze zbiorem wszystkich liczb naturalnych

postaci a3+b3+c3abc , gdzie a, b, c ∈ N. Możemy nawet założyć, że liczby a, b, c są parami względ-

nie pierwsze. Wykażemy teraz, że liczb naturalnych postaci a3+b3+c3abc jest nieskończenie wiele.

Wykażemy to nawet przy dodatkowym założeniu, że c = 1. Przedstawione tu fakty i ich dowo-dy pochodzą z rozwiązania zadania E682 z czasopima [Mon] 53(4)(1946) 223-224, podanegoprzez Erdosa i Nivena.

9.4.1 (Erdos, Niven 1946). Niech a, b będą liczbami naturalnymi takimi, że:

(a) nwd(a, b) = 1;

(b) a < b;

(c) ab | a3 + b3 + 1.Wtedy a | b3 + 1. Oznaczmy u = b3+1

a , m1 =a3+b3+1ab , m2 =

b3+u3+1bu . Wtedy u i m1 są

liczbami naturalnymi oraz:

(1) nwd(b, u) = 1;

(2) b < u;

(3) bu | b3 + u3 + 1, tzn. m2 jest liczbą naturalną;(4) m1 < m2. ([Mon] 53(4)(1946) 223-224).

Page 77: Liczby wymierne

Liczby wymierne. 9. Liczby postaci x1/x2 + x2/x3 + · · ·+ xs/x1 73

D. (1). Ponieważ au = b3 + 1, więc 1 = au+ (−b2)b, czyli nwd(b, u) = 1.(2). Przypuśćmy, że b > u. Wtedy: b3 > ab2 > ab > au = b3 + 1 i mamy sprzeczność: b3 > b3 + 1.

(3). Ponieważ u | b3 + 1, więc u | b3 + u3 + 1. Należy więc tylko pokazać, że b | b3 + u3 + 1czyli, że b | u3 + 1. Z założenia b dzieli a3 + 1. Niech a3 + 1 = vb, gdzie v ∈ N. Mamy wtedy:u3+1 = u3+(ua)3+1− (ua)3 = u3(1+ a3)+ 1− (b3+1)3 = u3vb− b9+3b6− 3b3, a zatem b | u3+1i ostatecznie bu | b3 + u3 + 1.(4). Zauważmy, że m1 = a

3+b3+1ab = a

3+uaab =

a2+ub , m2 =

b3+u3+1bu = ua+u

3

bu = a+u2

b . Należy więcpokazać, że a2 + u < a+ u2 czyli, że a(a− 1) < u(u− 1). Ale to jest oczywiste, gdyż 1 6 a < u. �

9.4.2. Istnieje nieskończenie wiele liczb naturalnych postaci a3+b3+c3abc , gdzie a, b, c ∈ N. Ist-

nieje nawet nieskończenie wiele liczb naturalnych tej postaci spełniających warunek c = 1.(Wynika to z twierdzenia 9.4.1).

9.4.3. Zbiór A3 jest nieskończony. Innymi słowy, istnieje nieskończenie wiele liczb natural-nych postaci xy +

yz +

zx , gdzie x, y, z ∈ N. (Jest to konsekwencja faktów 9.4.2 i 9.3.4).

9.4.4 (Dofs 1995). Niech t będzie dowolną liczbą naturalną i niech x = a2b, y = b2c, z = c2a,gdzie a = 2, b = t2 − t+ 1, c = t2 + t+ 1. Wtedy

xy +

yz +

zx = t

2 + 5.

Oznacza to, że każda liczba naturalna postaci t2 + 5, gdzie t ∈ N, należy do zbioru A3. Stądw szczególności wynika, że zbiór A3 jest nieskończony. ([Dofs], [Bond]).

9.4.5. Jeśli s > 3, to zbiór As jest nieskończony.

D. Wiemy (na mocy 9.4.3), że zbiór A3 jest nieskończony. Udowodniliśmy (patrz 9.1.12), że jeślin ∈ As, to n+ 1 ∈ As+1. Stąd wynika, że zbiór A4 jest nieskończony. Stąd dalej wynika, że zbiór A5jest nieskończony, itd. �

oooooooooooooooooooooooooooooooooooooooooooooooooooooooooooooooooooooo9.5 Przykłady liczb naturalnych należących do A3ooooooooooooooooooooooooooooooooooooooooooooooooooooooooooooooooooooooW 1964 roku Wacław Sierpiński ([S64] 136-138) napisał, że nie wiadomo czy czy liczba 4

należy do A3. Dzisiaj już wiadomo, że nie należy. Udowodnił to w 2000 roku A.V. Bondarenko([Bond]). On udowodnił nawet więcej:

9.5.1 (Bondarenko 2000). Każda liczba naturalna postaci 4m2, gdzie 3 - m, nie należy dozbioru A3. ([Bond], patrz 9.8.7).

W tym podrozdziale stosować będziemy następującą terminologię. Załóżmy, że liczbanaturalna n należy do zbioru A3. Istnieje wtedy trójka (x,y,z) liczb naturalnych takich, żen = xy+

yz+zx oraz nwd(x, y, z) = 1. Trójki (y, z, x) i (z, x, y) mają wówczas te same własności.

Z tych trzech trójek wybierzmy tę, która na pierwszym miejscu ma liczbę równą min{x, y, z}.Taką trójkę nazywać będziemy α-trójką liczby n.

Page 78: Liczby wymierne

74 Liczby wymierne. 9. Liczby postaci x1/x2 + x2/x3 + · · ·+ xs/x1

Wiemy, że jeśli n ∈ A3, to istnieje trójka (a, b, c) liczb naturalnych takich, że n = a3+b3+c3abc .

W tym przypadku możemy zakładać, że a 6 b 6 c oraz, że liczby a, b, c są parami względniepierwsze. Każdą trójkę o tych własnościach nazywać będziemy β-trójką liczby n. α-Trójkioznaczać będziemy przy pomocy zwykłych nawiasów. Natomiast β-trójki przy pomocy na-wiasów kwadratowych. Liczba naturalna może posiadać więcej niż jedną α-trójkę. Podobniejest z β-trójkami.

9.5.2. Jśli [a, b, c] jest β-trójką liczby naturalnej n, to trójki (ac2, ba2, cb2) i (ab2, ca2, bc2),po cyklicznym przestawieniu najmniejszej liczby na pierwsze miejsce, tworzą α-trójki licz-by n. Dla przykładu, z β-trójki [1, 2, 9] liczby 41 otrzymujemy dwie różne α-trójki liczby 41;mianowicie (2, 36, 81) i (4, 9, 162).

9.5.3 (Rusin 2003). Istnieje dokładnie 57 liczb naturalnych n mniejszych od 200, dla którychrównanie xy +

yz +

zx = n posiada rozwiązanie w zbiorze liczb naturalnych. Są to następujące

liczby:3, 5, 6, 9, 10, 13, 14, 17, 18, 19, 21, 26, 29, 30, 38,41, 51, 53, 54, 57, 66, 67, 69, 73, 74, 77,83, 86, 94, 101, 102, 105, 106, 110, 113, 117,122, 126, 129, 130, 133, 142, 145, 147, 149, 154, 158,161, 162, 166, 174, 177, 178, 181, 186, 195, 197.

Dla każdej z tych liczb, oprócz liczb 3 i 5, rozważane równanie posiada nieskończenie wielerozwiązań (x, y, z) takich, że nwd(x, y, z) = 1. To samo dotyczy równania x

3+y3+z3xyz = n.

([Rus1]).

9.5.4 (Maple). Pewne liczby naturalne n 6 200 należące do A3 wraz z ich pewnymi α-trójkami.

(3) (1, 1, 1);

(5) (1, 2, 4);

(6) (2, 12, 9), (3, 18, 4);

(9) (12, 63, 98), (18, 28, 147);

(10) (175, 882, 1620);

(14) (28, 637, 338), (52, 1183, 98);

(19) (5, 225, 81), (9, 405, 25);

(41) (2, 36, 81), (4, 9, 162), (5, 350, 196), (14, 980, 25);

(53) (28, 1323, 1458);

(66) (3, 126, 196), (9, 14, 588);

(106) (35, 66150, 2916), (64, 102060, 1225);

(149) (14, 8820, 2025), (45, 28350, 196);

(154) (52, 10647, 7938).

Page 79: Liczby wymierne

Liczby wymierne. 9. Liczby postaci x1/x2 + x2/x3 + · · ·+ xs/x1 75

9.5.5 (Rusin 2003, [Rus2]). Wszystkie liczby naturalne n 6 200 należące do A3 wraz z ichpewnymi β-trójkami.

(3) [1, 1, 1];

(5) [1, 1, 2];

(6) [1, 2, 3], [1817, 3258, 5275], [4904676969, 10840875082, 15051171563];

(9) [2, 3, 7], [970703, 2982043, 4461282];

(10) [5, 7, 18], [4192875343, 11021882957, 19765145610];

(13) [9, 13, 38], [2197345737653, 6384056084353, 12689495542854];

(14) [2, 7, 13], [279025573, 759054842, 1638591583];

(17) [5, 18, 37], [1932849997397, 7649960172210, 14857581287413];

(18) [13, 42, 95], [5902844861231317, 35013190193908290, 54059017558123943];

(19) [1, 5, 9], [728051, 1279935, 4135819];

(21) [2, 13, 21], [38304582498, 44899033717, 187979061005];

(26) [9, 38, 91], [2592527851712161, 16461714780091854, 31072284713059955];

(29) [27, 43, 182], [725188306504448123, 2863730199603918763, 7554216031389795222];

(30) [2, 21, 31], [907576024698, 2555537666039, 8213238158509];

(38) [70, 151, 629];

(41) [1, 2, 9], [1, 5, 14], [2, 31, 43], [61, 1133, 1314], [1541, 10690, 25029], [13547, 17314, 97663],[11441, 86425, 192834], [240322, 681959, 2567203], [193669, 2829857, 4119086];

(51) [9, 13, 77], [9496944543173, 28497283786885, 116604793962657];

(53) [2, 7, 27], [210121627, 5309015927, 5755076082];

(54) [2, 43, 57], [370030298454, 3412808117911, 7948993687541];

(57) [19, 91, 310], [278307036741995371, 5726573130751998070, 8251115886938879299];

(66) [1, 3, 14], [55075, 1201649, 1852326];

(67) [1133, 7525, 23517];

(69) [2, 57, 73], [42, 95, 523], [38808119, 45866266, 349822755],[907290117, 16844207218, 29911475693], [11708394650, 69802887831, 234380785219];

(73) [89200900157319, 1391526622949983, 2848691279889518];

(74) [133, 2502, 4607];

(77) [67, 630, 1763], [133, 1382, 3665], [40225, 221062, 819413], [401247, 1986038, 7768135],[8123011655, 138755312182, 277792875423], [321489851593, 5998665668870, 11380945916077];

(83) [5, 9, 61], [406164641531, 2343744686659, 8805786469335];

(86) [2, 73, 91], [5660399432462138, 114038591571428467, 220904967896959585];

(94) [27, 182, 673], [19, 746, 945], [20400692347, 64738300490, 351211722633],[180053104598, 478460823507, 2838538679977];

(101) [79, 1271, 3078];

(102) [459338480695732254, 3816006884967068935, 13212742329826830581];

(105) [2, 91, 111], [35, 1171, 1854], [4934775, 86738143, 204325982],[22107891903, 239901074434, 733520068619];

Page 80: Liczby wymierne

76 Liczby wymierne. 9. Liczby postaci x1/x2 + x2/x3 + · · ·+ xs/x1

(106) [1, 35, 54], [1342, 15929, 46683], [100054843, 4555645497, 5608864334],[327256085169, 710839851638, 4957711976947];

(110) [1147, 2745, 18578];

(113) [345842, 6313383, 15170275];

(117) [545, 1677, 10318];

(122) najmniejsza β-trójka jest bardzo długa;

(126) [2, 111, 133], [1093, 4199, 23982], [843543, 6610037, 26297374],[1437546238546, 8374810124997, 38751798984143];

(129) [31, 774, 1679], [70, 629, 2361], [11970393, 28883125, 210898982],[11235206, 80300179, 338895771];

(130) najmniejsza β-trójka jest bardzo długa;

(133) najmniejsza β-trójka jest bardzo długa;

(142) najmniejsza β-trójka jest bardzo długa;

(145) [44634584148027469, 157591646586434781, 1007950541819512850];

(147) [21, 925, 1529];

(149) [1, 14, 45], [2, 133, 157], [45257, 87913, 769298], [1261745, 32670622, 75361293],[3617906033, 3624015553, 44175121682], [155269296833, 1140528906910, 5108338064637];

(154) [2, 13, 63], [62, 1183, 3285], [94550101, 2427158214, 5731153295],[394133054, 847190695, 7164362061], [689430032438, 33597986722807, 53457471559053];

(158) [5642215349875, 7336556299898, 80828288788977];

(161) [11, 38, 259], [109, 3933, 7826], [146, 6517, 11349], [39927179, 179799907, 1072531846],[124939654, 330937307, 2577222931], [8233174563067629, 444973594136388818, 678169113751189021];

(162) [35, 1854, 2881];

(166) [9, 611, 790];

(174) [5, 7, 78], [2, 157, 183], [608242, 46497117, 55872983], [65441098, 3473800847, 5671311957],[455934805, 600648279, 6899701406], [28379531231, 73248432669, 600842282950];

(177) istnieje przypuszczenie, że najmniejsza β-trójka jest bardzo długa;

(178) [2, 27, 97], [14392834313297, 170840056879242, 655798399654747];

(181) [10672860536839861, 21088064331923949, 201705586625136962];

(186) [2269, 15938, 81711], [11403, 22774, 219641], [5246451, 376524257, 513247054],[145592437, 3886992711, 9968391914], [2334061450181786, 54431784269157829, 150189528932685207];

(195) [7, 15, 143], [39, 703, 2279], [12303811, 814494411, 1230936587],[2114540363, 8501821579, 59118924099], [109899928179, 4982656556915, 9696551744971];

(197) [127, 6278, 11655].

9.5.6 (Maple). Pewne liczby naturalne 200 < n 6 500 należące do A3 wraz z ich pewnymiα i β-trójkami.

(201) [2, 183, 211];

(209) [5, 254, 481];

(230) [2, 211, 241];

Page 81: Liczby wymierne

Liczby wymierne. 9. Liczby postaci x1/x2 + x2/x3 + · · ·+ xs/x1 77

(237) (65, 481650, 12996); [1, 65, 114];

(243) [9, 77, 409];

(250) (36, 5427, 8978); [2, 9, 67];

(261) (63, 3626, 16428); [2, 241, 273], [3, 7, 74], [18, 637, 1685];

(269) (14, 11956, 3721), (61, 52094, 196); [1, 14, 61];

(294) [2, 273, 307];

(323) [9, 49, 377], [13, 869, 1813];

(326) [5, 14, 151];

(329) [2, 307, 343];

(339) [7, 543, 1067];

(366) [2, 343, 381];

(405) [2, 381, 421];

(413) [62, 3285, 8953];

(446) [2, 421, 463];

(451) [23, 31, 567];

(478) [13, 23, 378];

(489) [2, 463, 507].

oooooooooooooooooooooooooooooooooooooooooooooooooooooooooooooooooooooo9.6 Występowanie danej liczby w rozkładach liczb ze zbioru A3oooooooooooooooooooooooooooooooooooooooooooooooooooooooooooooooooooooo

9.6.1. Jeśli r jest liczbą naturalną, to istnieje tylko skończenie wiele par (y, z) ∈ N2 takich,że ry +

yz +

zr jest liczbą naturalną.

D. Niech r będzie ustaloną liczbą naturalną. Załóżmy, że y, z są liczbami naturalnymi takimi, żery +

yz +

zr = n, gdzie n jest pewną liczbą naturalną. Mamy wtedy równość

(1) r2z + y2r + z2y = nryz,

z której wynika, że y | r2z oraz z | ry2. Niech r2z = ay, ry2 = bz, gdzie a, b ∈ N . Wtedy r3y2 =r2(ry2) = r2bz = bay, czyli r3y = ab. Stąd r5z = r3(r2z) = r3(ay) = a2b. Zatem y, z są liczbaminaturalnymi postaci

(2) y = abr3 , z =a2br5 .

Wstawiając to do (1) i mnożąc stronami przez r13

a2b , otrzymujemy równość

(3) r10 + r8b+ a3b2 = nr6ab.

Z tej równości wynika, że b jest podzielnikiem liczby r10. Takich podzielników jest oczywiście tylkoskończenie wiele. Niech r10 = ub. Mamy wtedy u+ r8 + a3b = nr6a. Liczba a jest więc podzielnikiemliczby u + r8. Takich liczb a jest więc też tylko skończenie wiele. Z równości (2) wynika zatem, żerozważanych par (y, z) jest tylko skończenie wiele. �

U. Z powyższego dowodu otrzymujemy algorytm na znajdowanie, dla danej liczby naturalnej r,wszystkich trójek (x, y, z) ∈ N3, w których występuje liczba r i liczba xy +

yz +

zx jest naturalna.

Page 82: Liczby wymierne

78 Liczby wymierne. 9. Liczby postaci x1/x2 + x2/x3 + · · ·+ xs/x1

Możemy przyjąć, że x = r. Wtedy liczby y, z otrzymujemy w następujący sposób. Niech a będziedowolnym podzielnikiem liczby r10. Wtedy r10 = ua, gdzie u ∈ N. Niech b będzie dowolnym podziel-nikiem liczby r8 + u. Mamy wtedy skończenie wiele liczb wymiernych y = abr3 , z =

a2br5 . Wybieramy

tylko te, które są liczbami naturalnymi i spełniają warunek ry +yz +

zr ∈ N. �

9.6.2. Niech n będzie liczbą naturalną i niech x, y, z będą liczbami naturalnymi takimi, żen = xy+

yz+zx . Jeśli min{x, y, z} = 1, to n = 3 lub 5. W tych przypadkach mamy: 3 =

11+11+11 ,

5 = 12 +24 +

41 .

D. Załóżmy, że x = 1.(Sposób I). Niech a = nwd(x, y), b = nwd(y, z), c = nwd(z, x). Wtedy a = 1, b ∈ N oraz c = 1. Z

twierdzenia 9.3.5 wiemy, że wtedy n = a3+b3+c3

abc = 2+b3

b =2b + b

2. Stąd wynika, że b = 1 lub b = 2.Jeśli b = 1, to n = 3. Jeśli b = 2, to n = 5.

(Sposób II). Z równości n = xy +yz +

zx wynika równość x

2z + y2x+ z2y = nxyz, która w naszymprzypadku ma postać

z + y2 + z2y = nyz.

Stąd ywnika, że y | z oraz z | y2. Niech z = uy, y2 = vz, gdzie u, v ∈ N. Wtedy y2 = vz = vuy istąd y = uv, z = u2v. Zatem u2v + u2v2 + u5v3 = nu3v3 i po podzieleniu stronami przez u2v mamy:1+ v+u3v2 = nuv i stąd v = 1. Zatem z = u2 = y2. Podstawiając to do równości z+ y2+ z2y = nyz,otrzymujemy równość 2+y3 = ny z której wynika, że y | 2, Jeśli y = 1, to n = 3. Jeśli y = 2, to n = 5.�

9.6.3. Niech n będzie liczbą naturalną i niech x, y, z będą liczbami naturalnymi takimi, żen = xy +

yz +

zx oraz nwd(x, y, z) = 1. Jeśli co najmniej jedna z liczb x, y, z jest równa 2, to

n = 5, 6 lub 41. W tych przypadkach mamy: 5 = 24+41+12 , 6 =

212+

129 +

92 , 41 =

236+

3681+

812 .

D. Załóżmy, że x = 2. Niech a = nwd(x, y), b = nwd(y, z), c = nwd(z, x). Wtedy a = 1 lub 2oraz c = 1 lub 2. Przypadek a = c = 2 odpada, gdyż nwd(x, y, z) = 1. Z twierdzenia 9.3.5 wiemy, żewtedy n = a

3+b3+c3

abc . Możliwe są więc tylko przypadki: n = 2+b3

b =2b + b

2 lub n = 9+b3

2b . W pierwszymprzypadku 2 | b, więc b = 1 lub b = 2. Jeśli b = 1, to x = y = z = 1 wbrew temu, że x = 2. Jeśli b = 2,to n = 5 = 24 +

41 +

12 . W drugim przypadku b = 1, 3 lub 9 i wtedy odpowiednio n = 5, 6 lub 41. �

Następne fakty otrzymano przy pomocy komputera i algorytmu opisanego w uwadze podowodzie twierdzenia 9.6.1.

9.6.4. Niech n będzie liczbą naturalną i niech x, y, z będą liczbami naturalnymi takimi, żen = xy +

yz +

zx oraz nwd(x, y, z) = 1. Jeśli co najmniej jedna z liczb x, y, z jest równa 3, to

n = 6 lub 66. W tych przypadkach mamy: 6 = 318 +

184 +

46 , 66 =

3126 +

1269 +

126196 . (Maple).

9.6.5. Niech n będzie liczbą naturalną i niech x, y, z będą liczbami naturalnymi takimi, żen = xy +

yz +

zx oraz nwd(x, y, z) = 1. Jeśli co najmniej jedna z liczb x, y, z jest równa 4, to

n = 5, 6 lub 41. W tych przypadkach mamy: 5 = 41+12+24 , 6 =

43+

318+

184 , 41 =

49+

9162+

1624 .

(Maple).

9.6.6. Niech n będzie liczbą naturalną i niech x, y, z będą liczbami naturalnymi takimi, żen = xy +

yz +

zx oraz nwd(x, y, z) = 1. Jeśli co najmniej jedna z liczb x, y, z jest równa 5, to

n = 19 lub 41. W tych przypadkach mamy: 19 = 5225 +

22581 +

815 , 41 =

5350 +

350196 +

1965 . (Maple).

Page 83: Liczby wymierne

Liczby wymierne. 9. Liczby postaci x1/x2 + x2/x3 + · · ·+ xs/x1 79

9.6.7. Niech n będzie liczbą naturalną i niech x, y, z będą liczbami naturalnymi takimi, żen = x

y +yz +

zx oraz nwd(x, y, z) = 1. Jeśli co najmniej jedna z liczb x, y, z jest równa 9,

to n = 6, 19, 41, 66, 2369 lub 14 803. W tych przypadkach mamy: 6 = 92 +

212 +

129 , 19 =

9405 +

40525 +

259 , 41 =

9162 +

1624 +

49 , 66 =

914 +

14588 +

5889 , 2369 =

911826 +

1182621316 +

213169 ,

14 803 = 929565 +

29565133225 +

1332259 . (Maple).

9.6.8. Niech x, y, z będą liczbami naturalnymi takimi, że xy +yz +

zx jest liczbą naturalną i

nwd(x, y, z) = 1. Wtedy każda z liczb x, y, z jest różna od 6, 7 i 9. (Maple).

9.6.9. Niech x, y, z będą liczbami naturalnymi takimi, że xy +yz +

zx jest liczbą naturalną i

nwd(x, y, z) = 1. Jeśli któraś z liczb x, y, z jest mniejsza od 77, to może ona być jedynie jednąz liczb: 1, 2, 3, 4, 5, 9, 12, 14, 18, 20, 25, 28, 35, 36, 45, 50, 52, 54, 61, 63, 65. (Maple).

oooooooooooooooooooooooooooooooooooooooooooooooooooooooooooooooooooooo9.7 Zbiór B3ooooooooooooooooooooooooooooooooooooooooooooooooooooooooooooooooooooooPrzypomnijmy, że

B3 ={xy +

yz +

zx ; x, y, z ∈ N

}={xy +

yz +

zx ; x, y, z ∈ Q+

}.

Oznaczmy:C3 =

{x3+y3+z3xyz ; x, y, z ∈ N

}={x3+y3+z3xyz ; x, y, z ∈ Q+

}.

Wiemy (patrz 9.3.2), że C3 = B3.

Niech q ∈ Q+. Podobnie jak w poprzednim podrozdziale mówić będziemy, że (x, y, z) jestα-trójką liczby q, jeśli: x, y, z ∈ N, nwd(x, y, z) = 1, x = min{x, y, z} oraz q = xy +

yz +

zx .

Każda dodatnia liczba wymierna należąca do B3 ma oczywiście co najmniej jedną α-trójkę.Mówić będziemy, że [a, b, c] jest β-trójką liczby q jeśli: a, b, c są liczbami naturalnymi,

a 6 b 6 c, nwd(a, b, c) = 1 oraz q = a3+b3+c3abc . Jeśli q posiada β-trójkę, to oczywiście posiada

α-trójkę i odwrotnie.

Niech [a, b, c] będzie β-trójką liczby wymiernej q. Wówczas nwd(a, b, c) = 1. W przypadku,gdy q jest liczbą naturalną, to stąd wynika, że liczby a, b, c są parami względnie pierwsze.Tak nie musi być jednak, gdy q nie jest liczbą naturalną. Dla przykładu [1, 2, 2] jest β-trójkąliczby 174 i liczby 1, 2, 2 nie są parami względnie pierwsze. Nie znam odpowiedzi na następującepytanie.

9.7.1. Załóżmy, że liczba wymierna q posiada β-trójkę. Czy wtedy istnieje taka β-trójka[a, b, c] liczby q, że liczby a, b, c są parami względnie pierwsze? (03.04.2007).

Podamy teraz przykłady pewnych liczb wymiernych należących do B3 wraz z ich α iβ-trójkami. Wszystkie te przykłady znaleziono przy pomocy Maple.

9.7.2 (Maple). Przykłady liczb wymiernych postaci n2 , gdzie n ∈ N oraz 2 - n, wraz z ich αlub β-trójkami.

(7) (1, 1, 2), (1, 2, 2); [5, 7, 8];

Page 84: Liczby wymierne

80 Liczby wymierne. 9. Liczby postaci x1/x2 + x2/x3 + · · ·+ xs/x1

(11) (2, 3, 9), (2, 6, 9), [629, 1204, 1737];

(19) (4, 80, 25), (5, 100, 16); [1, 4, 5];

(33) (1, 4, 16); [1, 1, 4];

(37) (1, 3, 18), (1, 6, 18), (3, 90, 50), (5, 150, 9); [13, 72, 119], [63, 551, 604];

(41) [27, 155, 268];

(45) (3, 72, 64), (8, 192, 9); [1, 3, 8], [4, 5, 21], [63, 412, 695];

(49) (22, 1815, 450), (30, 2475, 121); [20, 37, 133];

(51) (1, 10, 25), (2, 5, 50);

(57) [7, 93, 104];

(61) [7, 104, 109];

(73) [7, 8, 45];

(85) [8, 117, 175];

(87) (4, 208, 169), (13, 676, 16); [1, 4, 13];

(97) (8, 1216, 361), (9, 1620, 400), (19, 2888, 64), (20, 3600, 81); [1, 8, 19], [1, 9, 20].

9.7.3 (Maple). Przykłady liczb wymiernych postaci n3 , gdzie n ∈ N oraz 3 - n, wraz z ich αlub β-trójkami.(10) (2, 4, 3), (3, 6, 4); [62, 81, 91];

(13) (1, 1, 3), (1, 3, 3); [7, 13, 15];

(16) (12, 45, 50), (18, 20, 75); [2, 3, 5];

(17) (1, 6, 4), (1, 15, 25), (2, 12, 3); [627, 818, 1547];

(19) (3, 4, 16), (3, 12, 16);

(20) (5, 50, 12), (6, 60, 25);

(23) (3, 36, 16), (4, 48, 9); [1, 3, 4];

(29) (1, 3, 9); [1, 1, 3];

(38) (1, 2, 12), (1, 6, 12);

(40) (6, 252, 49), (7, 294, 36); [1, 6, 7];

(41) (8, 320, 75), (15, 600, 64);

(44) (36, 208, 507); [3, 4, 13];

(53) (3, 63, 49), (7, 147, 9), [1, 3, 7];

(56) (15, 400, 256);

(62) (6, 44, 121), (12, 33, 242);

(65) [31, 37, 156];

(70) [14, 61, 135];

(74) (7, 588, 144), (12, 1008, 49); [1, 7, 12];

(77) (3, 5, 75);

(79) [57, 527, 776];

(85) [5, 11, 39];

(89) (33, 3509, 841);

(92) [28, 67, 237].

Page 85: Liczby wymierne

Liczby wymierne. 9. Liczby postaci x1/x2 + x2/x3 + · · ·+ xs/x1 81

9.7.4 (Maple). Przykłady liczb wymiernych postaci n4 , gdzie n ∈ N oraz nwd(n, 4) = 1, wrazz ich α lub β-trójkami.

(17) (1, 4, 2); [1, 2, 2];

(21) (1, 1, 4), (1, 4, 4); [3, 7, 8];

(27) (12, 126, 49), (14, 147, 36);

(29) (2, 24, 9), (3, 36, 8), (4, 5, 25), (4, 20, 25); [7, 8, 19];

(35) (1, 2, 8), (1, 4, 8);

(45) [8, 19, 39];

(69) (6, 45, 100), (8, 576, 81), (9, 20, 150), (9, 648, 64); [1, 8, 9], [8, 39, 67];

(75) (1, 12, 18), (2, 3, 36).

9.7.5 (Maple). Przykłady liczb wymiernych postaci n5 , gdzie n ∈ N oraz 5 - n, wraz z ich αlub β-trójkami.

(18) (36, 80, 75); [3, 4, 5];

(19) (2, 4, 5), (3, 9, 5), (4, 5, 10), (5, 15, 9); [4, 5, 7], [5, 7, 9], [333, 551, 595], [385, 589, 698];

(28) (4, 15, 18), (10, 12, 45), (15, 72, 64); [13, 35, 36], [54, 133, 155];

(29) [9, 25, 26];

(31) (1, 1, 5), (1, 5, 5), (9, 20, 48), (15, 36, 80); [11, 31, 35], [14, 37, 45], [95, 189, 292];

(32) [14, 43, 45];

(33) (1, 10, 4), (2, 20, 5); [86, 175, 279];

(34) (20, 175, 98), (28, 245, 50); [2, 5, 7];

(36) [7, 9, 20];

(39) [7, 15, 26];

(41) (5, 6, 36), (5, 30, 36); [35, 36, 97];

(44) [37, 140, 171];

(51) (3, 45, 25), (5, 75, 9); [1, 3, 5];

(54) [40, 147, 221];

(56) (2, 60, 9), (3, 90, 20);

(57) (5, 150, 36), (6, 180, 25); [1, 5, 6], [9, 20, 43];

(59) (30, 612, 289);

(62) (4, 48, 45), (15, 180, 16);

(67) (2, 20, 25), (4, 5, 50), (5, 175, 49), (7, 245, 25); [1, 5, 7], [37, 55, 161], [40, 221, 283];

(72) [8, 19, 45], [15, 26, 73];

(83) (3, 180, 16), (4, 240, 45); [37, 209, 315];

(87) (10, 52, 169), (20, 65, 338);

(88) [7, 54, 65];

(89) [50, 91, 279];

(96) (7, 490, 100), (10, 700, 49); [1, 7, 10], [5, 11, 28].

Page 86: Liczby wymierne

82 Liczby wymierne. 9. Liczby postaci x1/x2 + x2/x3 + · · ·+ xs/x1

9.7.6 (Maple). Przykłady liczb wymiernych postaci n6 , gdzie n ∈ N oraz nwd(n, 6) = 1, wrazz ich α lub β-trójkami.

(19) (2, 2, 3), (2, 3, 3); [16, 19, 21];

(23) (1, 3, 2), (2, 6, 3); [79, 108, 143];

(25) (1, 2, 3), (2, 3, 6); [108, 143, 211];

(31) (1, 6, 2), (1, 6, 3), (6, 10, 25), (6, 15, 25), (36, 112, 147); [3, 4, 7], [11, 27, 28], [223, 380, 567];

(41) (1, 2, 6), (1, 3, 6); [380, 567, 1123];

(43) (1, 1, 6), (1, 6, 6); [13, 43, 48];

(47) (5, 75, 18), (6, 90, 25);

(55) (6, 7, 49), (6, 42, 49);

(59) (1, 6, 9), (2, 3, 18).

9.7.7 (Maple). Przykłady liczb wymiernych postaci n7 , gdzie n ∈ N oraz 7 - n, wraz z ich αlub β-trójkami.

(22) (9, 14, 12), (18, 28, 21); [7, 8, 9];

(23) (14, 20, 25), (28, 35, 50);

(30) (4, 16, 7), (7, 28, 16); [388, 629, 819];

(32) (2, 4, 7), (4, 7, 14);

(33) [3, 5, 7];

(38) (35, 150, 36);

(40) [19, 45, 56];

(41) (4, 21, 18), (12, 63, 14), (35, 275, 121); [5, 7, 13], [26, 31, 63];

(52) [26, 63, 97];

(53) (1, 14, 4), (2, 28, 7);

(57) (1, 1, 7), (1, 7, 7); [5, 19, 21];

(59) [45, 56, 139];

(60) (28, 441, 162), (36, 567, 98); [2, 7, 9], [12, 13, 35];

(71) (3, 18, 28), (7, 8, 64), (7, 56, 64), (9, 14, 84);

(72) [19, 21, 62];

(73) [7, 27, 38];

(83) [9, 26, 49];

(97) (2, 84, 9), (3, 126, 28), (5, 75, 63), (21, 315, 25); [99, 533, 721];

(99) [12, 35, 73].

9.7.8. Niech q = n2+n+1n , gdzie n ∈ N. Wtedy (1, 1, n) jest α-trójką liczby q oraz q = a3+b3+c3abc ,

gdzie a = 2n+ 1, b = n2 + n+ 1, c = n2 + 2n.

Page 87: Liczby wymierne

Liczby wymierne. 9. Liczby postaci x1/x2 + x2/x3 + · · ·+ xs/x1 83

9.7.9 (M.Klamkin, [Crux] 2001 s.78). Jeśli a, b, c są długościami boków trójkąta, to

3min{ab +

bc +

ca ,ac +

cb +

ba

}> (a+ b+ c)

(1a +

1b +

1c

).

oooooooooooooooooooooooooooooooooooooooooooooooooooooooooooooooooooooo9.8 Liczby postaci x/y + y/z + z/x, gdzie x, y, z są liczbami całkowitymiooooooooooooooooooooooooooooooooooooooooooooooooooooooooooooooooooooooPrzedstawione tu fakty pochodzą głównie z pracy [Rus1].

Niech n będzie liczbą naturalną. Interesować nas będzie problem istnienia rozwiązań rów-nania

(I) xy +

yz +

zx = n

w zbiorze niezerowych liczb całkowitych. Po pomnożeniu obu stron przez xyz, równanie toprzyjmuje postać

(II) x2z + y2x+ z2y = nxyz.

Wiemy (patrz 9.3.1), że problem istnienia rozwiązań równania (II) w zbiorze nieujemnychliczb całkowitych sprowadza się do analogicznego problemu dla równania

(III) x3 + y3 + z3 = nxyz.

Zajmiemy się więc problemem istnienia rozwiązań równania (III) w zbiorze niezerowychliczb całkowitych. Ponieważ równanie (III) jest jednorodne, więc wystarczy zbadać problemistnienia rozwiązań równania

(IV ) x3 + y3 + 1 = nxy

w zbiorze niezerowych liczb wymiernych.

9.8.1 (Rusin 2003). Niech x = −nX−36+Y2(3X+4n2) , y =−(nX+36+Y )2(3X+4n2) . Wtedy równanie (IV ) przyj-

muje postać

(V ) Y 2 = X3 + n2X2 − 72nX − 16(4n3 + 27).

Jeśli n 6= 3, to dane przekształcenie jest odwracalne. Przekształcenie odwrotne ma postaćX = −4(n

2(x+y)+9)(3(x+y)+n) , Y =

4(n3−27)(x−y)(3(x+y)+n) . ([Rus1]).

Przy pomocy powyższego faktu D. Rusin ([Rus1]) sprowadził problem istnienia rozwią-zań równia (I) w zbiorze niezerowych liczb całkowitych do badania struktury grupy krzywejeliptycznej zadanej równaniem (V ). Dzięki temu Rusin otrzymał następujące wyniki.

9.8.2 (Rusin 2003). Niech n będzie liczbą naturalną różną od 5. Jeśli równanie xy+yz+zx = n

posiada rozwiązanie w zbiorze nieujemnych liczb całkowitych, to posiada nieskończenie wieleprymitywnych takich rozwiązań tzn. z warunkiem nwd(x, y, z) = 1. ([Rus1]).

Page 88: Liczby wymierne

84 Liczby wymierne. 9. Liczby postaci x1/x2 + x2/x3 + · · ·+ xs/x1

9.8.3 (J.W.S. Cassels 1960). Równanie xy +yz +

zx = 1 nie posiada rozwiązań całkowitych.

([Mat] 2/61 68, [Rus1]).

Zanotujmy przy okazji:

9.8.4. Następujące warunki są równoważne.(1) Równanie xy +

yz +

zx = 1 ma rozwiązanie całkowite.

(2) Istnieją liczby wymierne u, v, w takie, że u+ v + w = uvw = 1.(3) Istnieją liczby całkowite a, b, c takie, że a3 + b3 + c3 = abc 6= 0.(4) Istnieją liczby całkowite a, b, c takie, że (a+ b+ c)3 = abc 6= 0.(5) Istnieje liczba wymierna a taka, że równanie x3−x2+ax−1 = 0 ma trzy pierwiastki

wymierne. ([Mat] 3/57 11-13, 1/58 57).

9.8.5. Równanie xy +yz +

zx = 2 nie posiada rozwiązań całkowitych. ([Rus1]).

9.8.6. Równanie xy +yz +

zx = 3 posiada nieskończenie wiele prymitywnych rozwiązań w

zbiorze niezerowych liczb całkowitych. Wśród tych rozwiązań tylko jedno jest w zbiorze liczbnaturalnych, mianowicie x = y = z = 1. ([Rus1]).

9.8.7 (Rusin 2003). Równanie xy +yz +

zx = 4 nie posiada rozwiązań całkowitych.

([Rus1], porównaj 9.5.1).

9.8.8 (Rusin 2003). Istnieje dokładnie 111 liczb naturalnych n mniejszych od 200, dla któ-rych równanie xy +

yz +

zx = n posiada rozwiązanie w zbiorze niezerowych liczb całkowitych.

Są to następujące liczby:

3, 5, 6, 9, 10, 13, 14, 15, 16, 17, 18, 19, 20, 21, 26, 29, 30, 31, 35, 36, 38, 40,41, 44, 47, 51, 53, 54, 57, 62, 63, 64, 66, 67, 69, 70, 71, 72, 73, 74, 76, 77,83, 84, 86, 87, 92, 94, 96, 98, 99,101, 102, 103, 105, 106, 107, 108, 109, 110, 112, 113, 116, 117, 119, 120,122, 123, 124, 126, 127, 128, 129, 130, 132, 133, 136,142, 143, 145, 147, 148, 149, 151, 154, 155, 156, 158, 159, 160,161, 162, 164, 166, 167, 172, 174, 175, 177, 178,181, 185, 186, 187, 189, 190, 191, 192, 195, 196, 197.

Dla każdej z tych liczb, oprócz liczby 5, rozważane równanie posiada nieskończenie wielerozwiązań prymitywnych. To samo dotyczy równania x

3+y3+z3xyz = n. ([Rus1]).

9.8.9 (Rusin 2003). Równanie xy +yz +

zx = 112 posiada rozwiązanie w zbiorze niezerowych

liczb całkowitych. Najprostszym rozwiązaniem jest

x = 444882220325179840803472420042062236091767720844845203037340381653808676781078204185344064777425

y = 1800010639340561476631947037621286947915240684971323481294582383858472523311320365128373281158

z = −13318091576854113300162838591657841686993519959959149070559988026538909081959649861205201860.

Występują tu liczby mające około 90 cyfr. Podobna sytuacja ma miejsce, gdy zamiast liczby112 rozpatrzymy liczby 122, 130, 133, 142, 164, 177, 187 i 190. ([Rus1]).

Page 89: Liczby wymierne

Liczby wymierne. 9. Liczby postaci x1/x2 + x2/x3 + · · ·+ xs/x1 85

Udowodniliśmy (patrz 9.3.6), że jeśli x, y, z są liczbami naturalnymi takimi, że liczbaxy+yz+zx jest naturalna, to xyz jest sześcianem liczby naturalnej. Co się stanie, gdy rozważymy

ten sam problem w przypadku, gdy x, y, z są niezerowymi liczbami całkowitymi? Zanotujmy:

9.8.10. Niech x, y, z będą niezerowymi liczbami całkowitymi takimi, że xy +yz +

zx jest liczbą

całkowitą. Czy wtedy xyz jest sześcianem liczby całkowitej? (27.03.2007; nie znam odpowiedzi).

9.8.11. Niech a, b, c ∈ Z r {0}. Jeśli ab +bc +

ca = 3, to abc jest sześcianem liczby całkowitej.

([OM] Bośnia-Hercegowina 2005).

9.8.12. Jeżeli liczby a, b, c, ab +bc +

ca ,ac +

cb +

ba są całkowite, to |a| = |b| = |c|. ([TTjs] 1995).

9.8.13. Jeśli x, y, z są niezerowymi liczbami całkowitymi takimi, że xy +yz +

zx , to żadna z

liczb xy ,yz ,zx nie jest całkowita. ([Mat] 4/59 214).

oooooooooooooooooooooooooooooooooooooooooooooooooooooooooooooooooooooo9.9 Zbiór A4ooooooooooooooooooooooooooooooooooooooooooooooooooooooooooooooooooooooPrzypomnijemy, że A4 jest zbiorem wszystkich liczb naturalnych postaci xy +

yz +

zt +

tx ,

gdzie x, y, z, t ∈ N. Wiemy (patrz 9.4.5), że zbiór A4 jest nieskończony.

9.9.1 (Maple). Przykłady pewnych liczb naturalnych n 6 30 postaci n = xy +yz +

zt +

tx ,

gdzie x, y, z, t są liczbami naturalnymi. Dla każdej takiej liczby n podano jej pewne czwórki(x, y, z, t) spełniające warunek nwd(x, y, z, t) = 1.

(4) (1, 1, 1, 1);

(5) (1, 2, 1, 2), (1, 2, 4, 2);

(6) (1, 1, 2, 4), (1, 2, 2, 4), (1, 6, 4, 3), (2, 10, 4, 5), (3, 18, 6, 4);

(7) (6, 45, 30, 25), (2, 2, 12, 9), (10, 12, 45, 50), (12, 45, 50, 60), (4, 48, 18, 9);

(9) (1, 3, 18, 4), (1, 6, 18, 4), (2, 6, 36, 9), (4, 40, 32, 5), (3, 18, 4, 12);

(10) (5, 150, 36, 9), (6, 12, 45, 50), (9, 18, 20, 75), (6, 180, 45, 25), (12, 45, 90, 100);

(11) (1, 2, 12, 9), (4, 48, 72, 9), (2, 60, 9, 5), (10, 372, 180, 31), (7, 294, 84, 36);

(12) (1, 35, 25, 7), (3, 45, 15, 25), (5, 175, 25, 7), (9, 63, 245, 75), (5, 75, 9, 15);

(13), (3, 36, 24, 32), (1, 10, 25, 10), (4, 40, 160, 25), (4, 16, 160, 25), (12, 18, 264, 121);

(14) (4, 5, 150, 36), (2, 10, 20, 25), (4, 9, 162, 18), (4, 20, 25, 50), (1, 18, 4, 9);

(15) (2, 105, 50, 21), (1, 10, 25, 2), (10, 145, 50, 116), (6, 468, 54, 13), (8, 160, 175, 98);

(17) (3, 126, 18, 28), (9, 22, 242, 132), (4, 9, 162, 12), (2, 36, 81, 6), (9, 22, 12, 132);

(18) (4, 60, 225, 54), (10, 350, 28, 49), (4, 140, 245, 50), (10, 150, 36, 135), (4, 15, 225, 54);

(19) (10, 42, 180, 175), (4, 240, 72, 5), (9, 14, 245, 150), (2, 396, 121, 18), (3, 180, 144, 10);

(20) (5, 5, 225, 81), (9, 405, 25, 25), (9, 405, 25, 9), (5, 225, 81, 5), (9, 405, 405, 25);

(21) (3, 198, 36, 44), (2, 132, 9, 11), (6, 252, 63, 98), (6, 396, 484, 33), (1, 4, 48, 18);

(22) (4, 39, 169, 78), (2, 60, 9, 30), (12, 20, 75, 250), (1, 30, 100, 15), (5, 450, 500, 36);

Page 90: Liczby wymierne

86 Liczby wymierne. 9. Liczby postaci x1/x2 + x2/x3 + · · ·+ xs/x1

(23) (3, 90, 15, 50), (5, 450, 324, 18), (4, 336, 18, 7), (1, 30, 100, 6), (10, 100, 375, 18);

(25) (4, 5, 150, 90), (6, 396, 99, 121), (4, 240, 18, 45), (3, 90, 4, 5), (6, 180, 225, 10);

(26) (5, 450, 150, 108), (4, 35, 294, 90), (5, 75, 18, 108), (5, 75, 450, 108), (3, 495, 450, 44);

(27) (4, 55, 242, 10), (4, 420, 147, 90), (7, 15, 126, 180), (7, 10, 84, 180), (2, 372, 36, 31);

(28) (1, 3, 9, 27), (3, 369, 27, 41);

(29) (1, 18, 12, 27), (3, 114, 36, 76), (2, 36, 81, 3), (4, 9, 162, 6), (4, 9, 162, 108);

(30) (2, 84, 252, 49), (4, 55, 242, 110), (10, 44, 20, 275), (4, 180, 25, 90), (2, 6, 252, 49).

9.9.2 (Maple). Liczby naturalne n 6 100 należące do zbioru A4.

4, 5, 6, 7, 9, 10, 11, 12, 13, 14, 15, 17, 18, 19,

20, 21, 22, 23, 25, 26, 27, 28, 29, 30, 31, 33, 35, 36, 37, 38, 39,

41, 42, 43, 45, 47, 49, 50, 51, 52, 54, 55, 57,

61, 62, 63, 67, 68, 70, 71, 73, 75, 76, 77,

82, 84, 90, 91, 93, 97, 98.

Nie wiem czy to są wszystkie liczby naturalne (mniejsze od 100) o tej własności.

Z obliczeń przy pomocy Maple nasuwają się następujące pytania, na które nie znamodpowiedzi.

9.9.3. Czy prawdą jest, że jeśli n ∈ A4, to 8 - n? (31.03.2007).

9.9.4.Wiadomo, że liczby 7, 10, 17, 18, 19, 20, 25, 26, 27, 30 są postaci xy +yz +

zt +

tx , gdzie

x, y, z, t ∈ N. Czy dla tych liczb istnieją takie czwórki (x, y, z, t), że co najmniej jedna z liczbx, y, z, t jest równa 1? (31.03.2007).

9.9.5. Rozpatrzmy równanie xy +yz +

zt +

tx = m.

(1) Jeśli m = 1, to równanie to nie ma naturalnych rozwiązań. Ma natomiast nieskoń-czenie wiele rozwiązań w zbiorze niezerowych liczb całkowitych.

(2) Dla m = 2 i m = 3 nie ma rozwiązań naturalnych.

(3) Jeśli m = 4, to każde naturalne rozwiązanie jest postaci (n, n, n, n), n ∈ N.([Mat] 3/57 13, [S64] 141-142).

Wiemy (patrz 9.3.4), że zbiór A3 pokrywa się ze zbiorem wszystkich liczb naturalnychpostaci a

3+b3+c3abc , gdzie a, b, c ∈ N. Czy coś podobnego zachodzi dla liczb naturalnych nale-

żących do zbioru A4? Wiemy (patrz 9.1.7), że każda liczba naturalna postaci a4+b4+c4+d4abcd ,

gdzie a, b, c, d ∈ N, należy do zbioru A4. Czy każdą liczbę naturalną ze zbioru A4 można takprzedstawić? Udowodnimy, że tak nie jest.

9.9.6. Liczba 5 należy do zbioru A4 i nie jest postaci a4+b4+c4+d4abcd , gdzie a, b, c, d ∈ N.

Page 91: Liczby wymierne

Liczby wymierne 9. Liczby postaci x1/x2 + x2/x3 + · · ·+ xs/x1 87

D. Ponieważ 5 = 12 +21 +

12 +

21 , więc 5 ∈ A4. Przypuśćmy, że istnieją liczby naturalne a, b, c, d

takie, że 5 = a4+b4+c4+d4

abcd . Skracając ewentualnie przez największy wspólny dzielnik, możemy założyć,że nwd(a, b, c, d) = 1. Mamy więc równość

a4 + b4 + c4 + d4 = 5abcd.

Jeśli liczba całkowita u nie jest podzielna przez 5, to (na mocy małego twierdzenia Fermata)

u4 ≡ 1 (mod 5).

W naszym przypadku a4 + b4 + c4 + d4 ≡ 0 (mod 5). Każda więc z liczb a, b, c, d musi być podzielnaprzez 5. Jest to jednak sprzeczne z tym, że nwd(a, b, c, d) = 1. �

Page 92: Liczby wymierne

88 Liczby wymierne Literatura

Literatura

[B-rs] J. Browkin, J. Rempała, S. Straszewicz, 25 lat Olimpiady Matematycznej, WSiP, Warszawa,1975.

[B-zm] V. I. Bernik, I. K. Żuk, O. W. Melnikow, Zbiór zadań olimpijskich z matematyki (po rosyjsku),Narodnaja Aswieta, Minsk, 1980.

[BaL] I. W. Baranowa, C. E. Lapin, Zadania z algebry (po rosyjsku), Leningrad, 1954.

[Balt] Zawody Matematyczne Państw Bałtyckich.

[Bedn] W. Bednarek, Zbiór zadań dla uczniów lubiących matematykę, Gdańskie Wydawnictwo Oświa-towe, Gdańsk, 1995.

[BoL] W. G. Bołtiański, W. G. Leman, Zbiór zadań moskiewskich olimpiad matematycznych (porosyjsku), Moskwa, 1965.

[Bond] A. V. Bondarenko, Investigation of a class of Diophantine equations, (po rosyjsku), UkrainMath. Zh. 52(6)(2000), 831-836.

[Br80] J. Browkin, Zadania z Olimpiad Matematycznych, tom 5, 21-25, 69/70 - 73/74, WSiP, War-szawa, 1980.

[Br83] J. Browkin, Zbiór Zadań z Olimpiad Matematycznych, tom 6, 26-30, 74/75 - 78/79, WSiP,Warszawa, 1983.

[BrG] A. Bremner, R. K. Guy, Two more representation problems, Proc. Edin. Math. Soc., 40(1997),1-17.

[Bryn] M. Bryński, Olimpiady Matematyczne, tom 7, 31-35, 79/80 - 83/84, WSiP, Warszawa, 1995.

[Cmj] College Mathematics Journal.

[Crux] Crux Mathematicorum, 1974-2008.

[Dic1] L. E. Dickson, History of the Theory of Numbers, Vol. I. Divisibility and primality, CarnegieInstitute of Washington, 1919. Reprinted by AMS Chelsea Publishing, New York, 1992.

[Dlt] Delta, Popularny Miesięcznik Matematyczno-Fizyczno-Astronomiczny.

[DoC] S. Doduniekow, K. Czakyrjan, Zadania z teorii liczb (po rosyjsku), Narodna Poswieta, Sofia,1985.

[Dofs] E. Dofs, Solutions of x3 + y3 + z3 = nxyz, Acta Mathematica, 73(3)(1995), 201-213.

[Fom] D. W. Fomin, Sankt-Petersburskie Olimpiady Matematyczne (po rosyjsku), Politechnika, Sankt-Petersburg, 1994.

[G-if] S. A. Genkin, I. W. Itenberg, D. W. Fomin, Leningradzkie Kółka Matematyczne (po rosyjsku),Kirow, ASA, 1994.

[GaT] G. A. Galpierin, A. K. Tołpygo, Moskiewskie Olimpiady Matematyczne (po rosyjsku), 1935-1985, Moskwa, 1986.

[GeG] S. I. Gelfand, M. L.Gerwer, A. A. Kiryłow, N. N. Konstantinow, A. G. Kusznirenko, Zadaniaz elementarnej matematyki, Ciągi, Kombinatoryka, Granice (po rosyjsku), Nauka, Moskwa,1965.

[Gess] Ira M. Gessel, Wolstenholme revisited, Amer. Math. Monthly 104(1998)(7), 657-658.

[Gy04] R. K. Guy, Unsolved Problems in Number Theory, Third edition, Springer-Verlag, New York,2004.

Page 93: Liczby wymierne

Liczby wymierne Literatura 89

[Hass] H. Hasse, Number Theory, Springer-Verlag, 1980.

[HW4] G. H. Hardy, E. M. Wright, An introduction to the theory of numbers, Fourth edition, Oxfordat the Clarendon Press, 1960.

[Ibe] Iberoamerican Mathematical Olympiad.

[IMO] Międzynarodowa Olimpiada Matematyczna.

[Isaa] I. M. Isaacs, Algebra, A Graduate Course, Brooks/Cole Publishing Company, Pacific Grove,California, 1994.

[JaK] R. Jajte, W. Krysicki, Z matematyką za pan brat, Iskry, Warszawa, 1985.

[Jedr] P. Jędrzejewicz, Bukiety Matematyczne dla Gimnazjum, Gdańskie Wydawnictwo Oświatowe,2002.

[Ko01] L. Kourliandtchik, Etiudy matematyczne, Tutor, Toruń, 2000.

[Ko04] L. Kourliandtchik, Słynne nierówności, Aksjomat, Toruń, 2002.

[KoM] KoMal, Kozepiskolai Matematikai Lapok, Węgierskie czasopismo matematyczne, 1894-2008.

[Kurs] J. Kurschak, Węgierskie Olimpiady Matematyczne (po rosyjsku), MIR, Moskwa, 1976.

[Kw] Kwant 1970-2008, czasopismo rosyjskie.

[LeH] H. Lee, Problems in Elementary Number Theory, Version 08, Internet 2003.

[Lion] F. Le Lionnais, Les nombres remarquables, Herman, Paris, 1983.

[M-sj] The Mathematics Student Journal.

[Mat] Matematyka, Czasopismo dla nauczycieli.

[Maza] W. Marzantowicz, P. Zarzycki, Elementarna Teoria Liczb, Wydawnictwo Naukowe PWN,Warszawa, 2006.

[MG] The Mathematical Gazette.

[MM] Mathematics Magazine.

[Mon] The American Mathematical Monthly.

[Nar86] W. Narkiewicz, Classical Problems in Number Theory, Monografie Matematyczne 62, War-szawa, 1986.

[OM] Olimpiada Matematyczna.

[OMm] Mała Olimpiada Matematyczna.

[Oss] Onatari Secondary School Mathematics Bulletin.

[Putn] Putnam (William Lowell) Mathematical Competition.

[RaT] H. Rademacher, O. Toeplitz, O liczbach i figurach, PWN, Warszawa, 1956.

[Ri97] P. Ribenboim, Mała księga wielkich liczb pierwszych, WNT, Warszawa, 1997.

[Rus1] D. Rusin, For which value of n is a/b+ b/c+ c/a = n solvable?, July, August 2003.http://www.math.niu.edu/∼rusin/reserach-math/abcn/.

[Rus2] D. Rusin, Small solutions for a/b+ b/c+ c/a = n, July, August 2003.http://www.math.niu.edu/∼rusin/reserach-math/abcn/smollsols,

[S-kg] W. A. Sadowniczij, A. A. Grigorjan, S. W. Konjagin, Zadania studenckich olimpiad matema-tycznych (po rosyjsku), Moskwa, 1987.

Page 94: Liczby wymierne

90 Liczby wymierne Literatura

[S50] W. Sierpiński, Teoria Liczb, Warszawa - Wrocław, 1950.

[S54] W. Sierpiński, Trójkąty pitagorejskie, PWN, Warszawa, 1954.

[S57a] W. Sierpiński, O rozkładach liczb wymiernych na ułamki proste, PWN, Warszawa, 1957.

[S59] W. Sierpiński, Teoria Liczb II, PWN, Warszawa, 1959.

[S64] W. Sierpiński, 200 zadań z elementarnej teorii liczb, Biblioteczka Matematyczna 17, PZWS,Warszawa, 1964.

[San4j] D. A. Santos, Junior Problem Seminar, Preprint, Internet 2004.

[Sand] J. Sandor, Geometric Theorems, Diophantine Equations, and Arithmetic Functions, AmericanResearch Press, Rehoboth, 2002.

[ShCY] D. O. Shklarsky, N. N. Chentzov, I. M. Yaglom, The USSR Olympiad Problem Book,W. F. Freeman and Company, San Francisco, London, 1962.

[Siw] I. H. Siwaszinskij, Nierówności w zadaniach (po rosyjsku), Nauka, Moskwa, 1967.

[Szn] L. B. Szneperman, Zbiór zadań z algebry i teorii liczb (po rosyjsku), Minsk, 1982.

[Szu87] M. Szurek, Opowieści matematyczne, WSiP, Warszawa, 1987.

[Tao] T. Tao, Solving Mathematical Problema, a personal perspective, Oxford, 2006.

[Tri] Ch. Trigg, Mathematical Quickies, McGraw-Hill Book Company, New York-London, 1967.Tłumaczenie rosyjskie: Moskwa, 2000.

[TT] Tournament of the Towns.

[TTjs] Tournament of the Towns, Junior, Spring.

[UsaT] USA Mathematical Talent Search.

[WaJ] N. B. Wasilew, A. A. Jegorow, Zadania Olimpiad Matematycznych Związku Radzieckiego (porosyjsku), 1961-1987, Moskwa, Nauka, 1988.

[Wino] I. Winogradow, Elementy teorii liczb, PWN, Warszawa, 1954.

[Wm] Wiadomości Matematyczne, Roczniki Polskiego Towarzystwa Matematycznego, 1956-2008.

[WyKM] W. A. Wyszenskij, I. W. Kartaszow, W. I. Michaiłowskij, M. I. Jadrenko, Zbiór zadańKijowskich Olimpiad Matematycznych (po rosyjsku), 1935-1983, Kijów, 1984.

[Zw] Zwardoń, Obóz Naukowy Olimpiady Matematycznej.

Uniwersytet M. Kopernika, Wydział Matematyki i Informatyki, ToruńOlsztyńska Wyższa Szkoła Informatyki i Zarządzania, Olsztyne-mail: [email protected]

Page 95: Liczby wymierne

SkorowidzAmerican Mathematical Monthly, 1Anning N., 5Apery, 37

Bonning A., 28Balfour A., 42Bibiloni L., 38bikwadrat liczby całkowitej, 5, 37, 63, 86, 87Boas R.P. Jr., 32boki trójkąta, 83Bondarenko A.V., 3, 66, 73

Cassels J.W.S., 84ciągarytmetyczny, 29, 30, 32, 38Fibonacciego, 31rekurencyjny, 12, 13

CoCoA, 1Cohen R., 28Crux Mathematicorum, 1cyfry, 5, 38, 41, 42, 62, 63, 85część całkowita, 2, 31część ułamkowa, 9czwórka liczb naturalnych, 34, 36, 85, 86

Delta, 1Derive, 1Derksen H., 10Dickson L.E., 65Dofs E., 73

elipsa, 41Erdos P., 71, 72

Frost P., 63funkcjaϕ, 2, 31, 41τ , 7, 22, 23z Q do Q, 10z Q×Q do Q, 10zeta Riemanna, 37

Gessel I.M., 3Glaisher, 63, 64granica ciągu, 37, 38Guy R.K., 28

Hagedorn T.R., 28hipoteza, 38, 59Hoke O.H., 15

Hsia C., 41

Iżboldin O.T., 16iloczyn kartezjański, 10indukcja matematyczna, 10, 12, 16, 30, 31, 55International Mathematical Olympiad, 1

kanoniczne przedstawienie, 49Klamkin M., 83Kline M., 38kolejne liczby naturalne, 5, 6, 9, 28–32, 34, 36–38,

53, 57, 61–65krzywa eliptyczna, 83Kurlandczyk L.D., 16Kwant, 1

Leibniz, 37liczbaπ, 37m-przedstawialna, 44bezkwadratowa, 17dzielników naturalnych, 7, 22, 23kwadratowa, 16, 18, 23, 28, 34, 35, 37, 62nieparzysta, 5, 7, 9, 15, 22, 28, 46, 47, 58, 60,61, 63, 64

niewymierna, 37, 42parzysta, 14, 15, 29, 32, 41, 46, 59pierwsza, 3, 8, 9, 22, 24, 32, 33, 40, 41, 49, 54,55, 57, 61–63, 65, 69

trójkątna, 35, 36wymierna, 3, 5zero-jedynkowa, 5

liczby względnie pierwsze, 9, 18, 20, 21, 23, 26–29,31–33, 44, 47, 48, 52, 58, 60–65, 69, 71,72, 74, 79

logarytm, 30, 37

maksymalny element, 21, 28Maple, 1Matematical Excalibur, 1Matematyka, 1Mathematica, 1Mathematical Gazette, 1Mendes M., 42Miksa F.L., 15Moser L., 35MuPad, 1

Nagara P.N., 15Narkiewicz W., 28

91

Page 96: Liczby wymierne

92 Liczby wymierne Skorowidz

nierówność, 12, 13, 17, 30, 31, 34, 36–38, 57, 66Niven I., 71, 72nwd, 2, 36, 44, 50–52, 71nww, 2, 31, 32, 41, 50–52

okres rozwinięcia dziesiętnego, 30, 39–41Ossowski P., 61–63

paraliczb całkowitych, 13liczb naturalnych, 7, 8, 13, 19–22, 26, 27, 33,59, 60, 77

liczb wymiernych, 10Paradis J., 38pochodna wielomianu, 54podpierścień, 11podzbiór, 9–11, 38potęgadwójki, 6, 9, 13, 21, 30, 32, 33, 36, 47, 61, 64,65

liczby pierwszej, 21, 22, 47, 65liczby wymiernej, 10trójki, 59

przedstawienie liczby naturalnej, 6, 7, 17przedstawienie liczby wymiernej, 9, 17–19przekrój zbiorów, 10, 66, 69Putnam Mathematical Competition, 1pytanie, 3, 36, 79, 86

równanie diofantycznen zmiennych, 12, 18, 33–36czterech zmiennych, 14, 24, 28, 34, 36, 86dwóch zmiennych, 7, 13, 19–24pięciu zmiennych, 14sześciu zmiennych, 14trzech zmiennych, 15, 21, 23, 25–28, 33, 35,36, 74, 83–85

Rademacher H., 41Rao K.S., 41relacja typu równoważności, 45rozkład kanoniczny, 22, 23, 49–51, 71rozwinięcie dziesiętne, 3, 30, 38–42Rusin D., 74, 75, 83, 84

Sandor J., 23, 28, 35Sato N., 41Schiller J.K., 41Schinzel A., 18Semionowa L., 41Sierpiński W., 25, 28, 35, 73silnia, 9, 13, 32, 54, 55, 57, 61, 62Simmons H.A., 28Smieszek I., 64

Sniady P.W., 11Strzelecki P., 38suma szeregu, 37, 38symbol Newtona, 54, 55, 65system numeracji, 5sześcian liczby całkowitej, 3, 5, 7, 8, 36, 37, 62, 63,

70–74, 78, 79, 82, 84–86sześcian liczby wymiernej, 10

Toeplitz O., 41trójkaliczb naturalnych, 13, 14, 17, 36, 73, 74, 76,77, 79–82

liczb wymiernych, 6twierdzenieBezouta, 55Bondarenko, 66, 73Erdosa-Nivena, 71, 72Gessela, 60–64małe Fermata, 55, 87Rusina, 84Wilsona, 55Wolstenholme, 3, 61

ułamek egipski, 3ułamek prosty, 3, 12

Viader P., 38

Wadhwa A.D., 38Wagutien W.N., 11warunki równoważne, 18, 22, 38, 43, 44, 67, 69–71,

84wielomian, 53–55Wilenkin N.J., 11Wolstenholme J., 3, 61Wróblewski J., 11Wrench J.W. Jr., 32

Zarzycki P., 38zbiór

N0, 1gęsty, 32liczb całkowitych, 1liczb naturalnych, 1liczb pierwszych, 1liczb rzeczywistych, 1liczb wymiernych, 1, 5, 9liczb zespolonych, 1, 7, 11multyplikatywny, 11